You are on page 1of 224

Ajay Tejwani

PGT Maths
KV AFS, Devlali
Conceptual Mathematics Workshop 2023
KV RHE Pune

Relation and Functions Case Study Questions

CASE STUDY 1:
A general election of Lok Sabha is a gigantic exercise. About 900 million
people were eligible to vote and voter turnout was about 75%, the highest
ever. Let I be the set of all citizens of India who were eligible to exercise
their voting right in general election held in 2019. A relation ‘R’ is defined
on I as follows:
R = {(𝑉1,2)∶ 𝑉1,𝑉2 ∈𝐼 and both use their voting right in general election –
2019}. Based on given information, answer the following questions
Q1) Is R a reflexive relation? Justify your answer. (1M)
Q2) Is R a symmetric relation? Justify your answer. (1M)
Q3) Is R a transitive relation? Justify your answer. (1M)
Q4) Is R an equivalence relation? Justify your answer. (1M)

CASE STUDY 2
Sherlin and Danju are playing Ludo at home
during Covid-19. While rolling the dice,
Sherlin’s sister Raji observed and noted the
possible outcomes of the throw every time
belongs to set {1,2,3,4,5,6}. Let A be the set
of players while B be the set of all possible
outcomes A i.e. A ={S, D} and B =
{1,2,3,4,5,6}, based on given information,
answer the following questions.
Q1) Let R : B ⟶ B be defined by R = {(x, y) :
y is divisible by x}. Is R an equivalence
relation? Justify your answer. (3M)
Q2 ) Raji wants to know the number of functions from A to B. How many number of functions are possible?
(1M)
CASE STUDY 3:
An organization conducted bike race under 2
different categories-boys and girls. Totally there
were 250 participants. Among all of them finally
three from Category 1 and two from Category 2
were selected for the final race. Ravi forms two
sets B and G with these participants for his college
project. Let G={g1,g2} and B = {b1,b2,b3} where B
represents the set of boys selected and G the set of
girls who were selected for the final race. based on given information, answer the following questions.
Q1) Ravi wishes to form all the relations possible from B to G. How many such relations are possible? (1M)
Q2) Ravi wants to know among those relations, how many functions can be formed from B to G? (1M)
Q3) Let R: B→B be defined by R = {(𝑥, 𝑦): 𝑥 and y are students of same sex}, then verify whether R is an
equivalence relation? (2M)

CASE STUDY 4:
Raji visited the Exhibition along with
her family. The Exhibition had a
huge swing, which attracted many
children. Raji found that the swing
traced the path of a Parabola as
given by 𝑦= 𝑥2. Answer the
following questions using the above
information.
Q1) Let 𝑓: 𝑅→𝑅 be defined by (𝑥)=
𝑥2, then verify whether 𝑓 is a
bijective function? (2M)

Q2) Let 𝑓: N→R be defined by (𝑥)=


𝑥2, then find the range of R? (1M)

Q3) Let 𝑓: Z→R be defined by (𝑥)=


𝑥2, Is f Injective function? Justify your answer? (1M)
Ajay Tejwani
PGT Maths
KV AFS, Devlali
Conceptual Mathematics Workshop 2023
KV RHE Pune

Relation and Functions MCQs


1) A relation R on set A = {1, 2, 3, 4, 5} is defined as R = {(1, 1), (2, 2), (3, 3), (4, 4), (5, 5)} then R is
_______________ relation
a) Reflexive b) Symmetric c) Transitive d) Equivalence.

2) Let R be the relation on set A = { x ∈ Z : x ≤ 20}, defined by R = {(a, b) : Ia - bI is a multiple of 3},


then [4], the equivalence class of 4, is
a) {0, 4, 8, 12, 16, 20} b) {1, 4, 7, 10, 13, 16, 19}
c) {0, 1, 4, 7, 10, 13, 16, 19} d) A

3) If a relation R on the set {a, b, c, d} is defined as R = {(a, b)}, then R is _______________ relation
a) Reflexive b) Symmetric c) Transitive d) simply a relation

4) The function 𝑓:R  Z, defined as 𝑓 (𝑥 ) = [𝑥] is (Z is set of integers)


a) neither one – one nor onto b) one – one but not onto
c) onto but not one – one d) one – one and onto

5) The function 𝑓(𝑥 ) = 5 − |sin(4𝑥)| has maximum value ‘a’ and minimum value ‘b’, then (a, b) =
a) (4, 5) b) (5, 4) c) (5, 6) d) (6, 5)

6) The function 𝑓: R  R, given by 𝑓 (𝑥) = |𝑥 | is


a) Surjective b) Injective c) Bijective d) neither surjective nor injective.

7) A relation R on set A = {a, b, c} is defined as R = {(a, b), (b, b)} then R will be _______________ relation when
(b, a) will be added
a) Reflexive b) Symmetric c) Transitive d) Equivalence

8) The function 𝑓: R  R, given by 𝑓 (𝑥) = 3𝑥 + 2 is


a) Surjective b) Injective c) Bijective d) neither surjective nor injective.

9) The maximum number of equivalence relation on the set A = {a, b, c} are


a) 2 b) 3 c) 5 d) 6

10) A relation R on set A = {a, b, c} is defined as R = {(a, b), (b, b), (c,c), (a, a)} then R will be _______________
relation when (b, a) will be added
a) Reflexive b) Symmetric c) Transitive d) Equivalence.

11) The function 𝑓:R  R, defined as 𝑓 (𝑥 ) = [𝑥] + x is


a) neither one – one nor onto b) one – one but not onto
c) onto but not one – one d) one – one and onto
Answers:

Q. No. Answer Q. No. Answer Q. No. Answer


1 d 5 b 9 d
2 b 6 d 10 d
3 c 7 b 11 d
4 c 8 c
INVERSE TRIGONOMETRIC FUNCTIONS

MULTIPLE CHOICE QUESTIONS


𝜋
1. If sin-1 x – cos-1 x = 6 , then x =

1 √3 −1 − √3
(a) (b) (c) (d)
2 2 2 2

2. If tan-1 (cot θ) = 2θ, then θ is equal to


𝜋 𝜋 𝜋
(a) (b) (c) (d) None of these
3 4 6

𝜋
3. cot( 4 – 2 cot-1 3) =

(a) 7 (b) 6 (c) 5 (d) None of these

4. The principal value of tan-1(tan 3π/5) is

(a) 2π/5 (b) -2π/5 (c) 3π/5 (d) -3π/5

5. sin[π/3 – sin-1(- ½)] is equal to:

(a) ½ (b) ⅓ (c) -1 (d) 1

6. The domain of sin–1(2x) is

(a) [0, 1] (b) [– 1, 1] (c) [-1/2, 1/2] (d) [–2, 2]

7. If sin–1 x + sin–1 y = π/2, then value of cos–1 x + cos–1 y is

(a) π/2 (b) π (c) 0 (d) 2π/3

8. The domain of y = cos–1 (x2 – 4) is

(a) [3, 5] (b) [0, π] (c) [-√5, -√3] ∩ [-√5, √3] (d) [-√5, -√3] ∪ [√3, √5]

9. The value of the expression sin [cot–1 (cos (tan–1 1))] is

(a) 0 (b) 1 (c) 1/√3 (d) √2/√3

10. Which of the following is the principal value branch of cos–1 x?

(a) [–π/2, π/2] (b) (0, π) (c) [0, π] (d) (0, π) – {π/2}

ANSWERS:

1 2 3 4 5 6 7 8 9 10
√3
(c)
𝜋 (b) - (c) [-1/2, (a) (d)
(b) (a) 7 (d) 1 (d) (c) [0, π]
2 6 2π/5 1/2] π/2 √2/√3
CASE BASED STUDY QUESTIONS

1. Read the following text and answer the following questions on the basis of the same:

In the school project Sheetal was asked to construct a triangle and name it as ABC. Two angles A and
B were given to be equal to tan-1(½) and tan-1(⅓) respectively.

a) The value of sin A is _______.

A. ½ B. ⅓ C. 1/√5 D. 2/√5

b) The third angle, ∠C = _______.

A. π/4 B. π/2 C. π/3 D. 3π/4

c) cos (A + B +C)

A. 1 B. 0 C. –1 D. 1/2

d) If A = sin-1 x , then the value of x is

A. 1/√10 B. 3/√10 C. 1/√5 D. 2/√5

e) If A = cos-1 x , then the value of x is

A. 1/√10 B. 3/√10 C. 1/√5 D. 2/√5

ANSWERS:
a) b) c) d) e)

1/√5 3π/4 –1 1/√5 3/√10

2. The Government of India is planning to fix a hoarding board at the face of a building on the road of a
busy market for awareness on COVID-19 protocol. Ram, Robert and Rahim are the three engineers
who are working on this project. ‘A’ is considered to be a person viewing the hoarding board 20
metres away from the building, standing at the edge of a pathway nearby, Ram Robert and Rahim
suggested to the film to place the hoarding board at three different locations namely C, D and E. ‘C’ is
at the height of 10 metres from the ground level. For the viewer ‘A’, the angle of elevation of ‘D’ is
double the angle of elevation of ‘C’. The angle of elevation of ‘E’ is triple the angle of elevation of ‘C’
for the same viewer.

Look at the figure given and based on the above information answer the following:
(i) Measure of ∠CAB =

(a) tan–1(2) (b) tan–1(1/2) (c) tan–1(1) (d) tan–1(3)

(ii) Measure of ∠DAB =

(a) tan–1(3/4) (b) tan–1(3) (c) tan–1(4/3) (d) tan–1(4)

(iii) Measure of ∠EAB

(a) tan–1(11) (b) tan–1(3) (c) tan–1(2/11) (d) tan–1(11/2)

(iv) A’ is another viewer standing on the same line of observation across the road. If the width of the
road is 5 meters, then the difference between ∠CAB and ∠CA’B is

(a) tan–1(1/12) (b) tan–1(1/8) (c) tan–1(2/5) (d) tan–1(11/21)

Answer:

(i) b (ii) c (iii) d (iv) a


Case Study Question CH 3: Matrices

Case Study Question – 1


Two schools P and Q want to award their selected students on the values of Tolerance, Kindness,
and Leadership. The school P wants to award Rs x each, Rs y each and Rs z each for the three
respective values to 3, 2 and 1 students respectively with total award money of Rs. 2200.
School Q wants to spend Rs 3100 to award its 4, 1 and 3 students on the respective values (by
giving the same award money to the three values as school P). If the total amount of award for
one prize on each value is Rs1200, using matrices, find the following:

1. What is award money for Tolerance?


1. 350
2. 300
3. 500
4. 400
2. What is the award money for Leadership?
1. 300
2. 280
3. 450
4. 500
3. What is the award money for Kindness?
1. 500
2. 400
3. 300
4. 550
4. If a matrix A is both symmetric and skew-symmetric, then
1. A is a diagonal matrix
2. A is a scalar matrix
3. A is a zero matrix
4. A is a square matrix
5. If A and B are two matrices such that AB = B and BA = A, then B 2 is equal to

1. B
2. A
3. 1
4. 0
Case Study Question - 2
Read the case study carefully and answer any four out of the following questions:
Three friends Ravi, Raju and Rohit were buying and selling stationery items in a market. The price
of per dozen of Pen, notebooks and toys are Rupees x, y and z respectively.Ravi purchases 4
dozen of notebooks and sells 2 dozen pens and 5 dozen toys. Raju purchases 2 dozen toys and
sells 3 dozen pens and 1 dozen of notebooks. Rohit purchases one dozen of pens and sells 3
dozen notebooks and one dozen toys.In the process, Ravi, Raju and Rohit earn ₹ 1500, ₹ 100 and
₹400 respectively.

Answer the following questions using the matrix method:

1. What is the price of one dozen of pens?


1. ₹ 100
2. ₹ 200
3. ₹ 300
4. ₹ 400
2. What is the total price of one dozen of pens and one dozen of notebooks?
1. ₹ 100
2. ₹ 200
3. ₹ 300
4. ₹ 400
3. What is the sale amount of Ravi?
1. ₹ 1000
2. ₹ 1100
3. ₹ 1300
4. ₹ 1200
4. What is the amount of purchases made by all three friends?
1. ₹ 1200
2. ₹ 1500
3. ₹ 1300
4. ₹ 1400
5. What is the price of sales made by all three friends?
1. ₹ 3000
2. ₹ 2500
3. ₹ 2700
4. ₹ 2400
Answer Key:
1. (a) ₹ 100
2. (c) ₹ 300
3. (d) ₹ 1200
4. (b) ₹ 1500
5. (c) ₹ 2700
Case Study Questions Matrices – 3
Read the case study carefully and answer any four out of the following questions:
Once a mathematics teacher drew a triangle ABC on the blackboard. Now he asked Jose,” If I
increase AB by 11 cm and decrease the side BC by 11 cm, then what type of triangle it would be?”
Jose said, “It will become an equilateral triangle.”

Again teacher asked Suraj,” If I multiply the side AB by 4 then what will be the relation of this with
side AC?”
Suraj said it will be 10 cm more than the three times AC.
Find the sides of the triangle using the matrix method and answer the following questions:
1. What is the length of the smallest side?
1. 54 cm
2. 43 cm
3. 30 cm
4. 35 cm
2. What is the length of the largest side?
1. 54 cm
2. 43 cm
3. 65 cm
4. 35 cm
3. What is the perimeter of the triangle?
1. 150 cm
2. 160 cm
3. 165 cm
4. 162cm
4. What is the side of the equilateral triangle formed?
1. 54 cm
2. 43 cm
3. 30 cm
4. 35 cm
1. What is the order of the matrix formed?
1. 3 × 3
2. 2 × 3
3. 3 × 2
4. 2 × 2
Answer Key:
1. (b) 43 cm 2. (c) 65 cm 3. (d) 162 cm 4. (a) 54 cm 5. (a) 3 × 3
Prepared by
Ajaykumar Patole
PGT-MATHS
K.V.NO.1,Ahmednagar
NAME: BHANU PRATAP MISHRA DESIGNATION: PGT-MATHS
NAME OF KV: K V AMBARNATH (MUMBAI REGION)

TOPIC: DETERMINANTS.
SECTION A
MULTIPLE CHOICE TYPE QUESTIONS:

𝑥 1 1 1
1. The value of 𝑥 if | |=| |, is
1 𝑥 1 25
A) 1 B) 2 C) 5 D) 25
2. If 𝑨 is a square matrix of order 3 and |𝑨| = 5 then the value of |𝟑𝑨| is
A) 45 B) 75 C) 135 D) 125
3. 𝑨 is a matrix of order 4 and |𝑨|=10 then the value of |𝒂𝒅𝒋𝑨| is
A) 40 B) 400 C) 1,000 D) 10,000
4. The number of possible matrices of order 2x2 with each entry 3 or 5 is
A) 4 B) 8 C) 16 D) 32
5. If A is a matrix of order 3 and |𝐴| = 2 then the value of |2 𝑎𝑑𝑗 𝐴| is
A) 32 B) 16 C) 8 D) 81
0 𝑎 1
6. If 𝐴 = [−1 𝑏 1] is a skew symmetric matrix, then the value of
−1 𝑐 0
(𝑎 + 𝑏 + 𝑐)2 is
A) 1 B) 0 C) 4 D) -2
7. The |𝐴| where matrix A is a scalar matrix of order 4 with every diagonal
element equal to 3 is
A) 64 B) 81 C) 12 D) 7
1 −2
8. The matrix 𝐴 such that 𝐴. [ ] = [17 1], is
2 1
A) [3 7] B) [7 1] C) [17 1] D) [7 3]
9. If |𝑎𝑑𝑗𝐴|=|2𝐴| where A is a non-singular matrix of order 3X3, then
Value of |𝐴| is
A) 32 B) 16 C) 8 D) 81
4 −3
10. Inverse of the Matrix [ ] is
5 −4
4 −3 −4 3 −4 −3 4 3
A) [ ] B) [ ] C) [ ] D) [ ]
5 −4 −5 4 5 4 −5 −4
SECTION B
ASSERTION REASON TYPE QUESTIONS:
Choose the correct option for the following questions:
(A) Both statements I & II are true and Statement II is the correct
Explanation of Statement I.
(B) Both statements I & II are true and Statement II is not the correct
Explanation of Statement I.
(C) Statement I is true and Statement II is false.
(D) Statement I is false and Statement II is true.
(E) Both the Statements I & II are false.
1. Let 𝐴 and 𝐵 be square matrices of same order.
Statement I: 𝐴. (𝑎𝑑𝑗𝐴) = |𝐴|𝐼
Statement II: 𝑎𝑑𝑗(𝐴𝐵 ) = (𝑎𝑑𝑗𝐴). (𝑎𝑑𝑗𝐵).
1 0 1
2. Statement I: If matrix 𝐴 = [0 1 2] then |3𝐴| = 108.
0 0 4
Statement II: If 𝐴 is a square matrix of order n then |𝑘𝐴| = 𝑘 𝑛 |𝐴|.
3. Let 𝐴, 𝐵 and 𝐶 are three square matrices of the same order.
Statement I: 𝐴𝐵 = 𝟎  𝐴 = 𝟎 or, 𝐵 = 𝟎.
Statement II: 𝐴 + 𝐵 = 𝐴 + 𝐶  𝐵 = 𝐶.
4. Statement I: If 𝐴 is a square matrix then 𝐴2 is asymmetric matrix.
Statement II: (𝐴2)′ = 𝐴2.
5. Statement I: Every square matrix is associated with a unique
determinant.
2 1
Statement II: The square matrix [ ] is associated with the
5 6
2 1 3 −5
determinants | | and | | .
5 6 −1 4

Answers:
SECTION A: 1. (C) 2. (C) 3. (C) 4. (C) 5. (A)
6. (B) 7. (B) 8. (A) 9. (C) 10. (A).
SECTION B: 1. (C) 2. (A) 3. (D) 4. (E) 5. (A).
Topic: MCQ on Ch 3.Matrices

Q1. If A is a square matrix such that A2 = A, then (I – A)3 + A is equal to


(a) I (b) 0 (c) I – A (d) I + A
Correct option: (a) I
Solution: Given that, A is a square matrix and A2 = A.
Consider (I + A)3, where I is the identity matrix.
Using the identity of (a + b)3 = a3 + b3 + 3ab (a + b), we get, (I + A)3 = I3 + A3 + 3A2I + 3AI2
= I + A2(A) + 3AI + 3A
= I + A2 + 3A + 3A
= 7A + I {since it is given that A2 = A}
So, (I + A)3 = 7A + I….(1)
Now,
(I + A)3 – 7A = 7A + I – 7A [From (1)]
=I
Q2. If A = [aij] is a square matrix of order 2 such that aij = 1, when i ≠ j and aij = 0, when i = j, then A2 is

(a) 1 1 (b) 1 0 (c) 1 0 (d) 0 0


0 0 0 1 1 0 1 1
Correct option (b)
Solution: A = [aij] is a square matrix of order 2 such that aij = 1, when i ≠ j and aij = 0, when i = j.
So, a11 = 0, a12 = 1, a21 = 1 and a22 = 0.
A2 = A.A

= 0 1 0 1 = 1 0
1 0 1 0 0 1

Q3. Total number of possible matrices of order 3 × 3 with each entry 2 or 0 is


(a) 9 (b) 27 (c) 81 (d) 512
Correct option: (d) 512
Solution: We know that a matrix 3× 3 contains 9 elements.
Given that each entry of this 3× 3 matrix is either 0 or 2.
Thus, by simple counting principle, we can calculate the total number of possible matrices as:
Total number of possible matrices = Total number of ways in which 9 elements can take possible values = 29 = 512
Q4. If A and B are two matrices of the order 3 × m and 3 × n, respectively, and m = n, then the order of
matrix (5A – 2B) is
(a) m × 3 (b) 3 × 3 (c) m × n (d) 3 × n
Correct option: (d) 3 × n
Solution: Given that, the order of matrix A is 3 × m, and the order of B is 3 × n.
Also, m = n. So, the order of matrix A and B is the same, i.e. 3 × m.
Thus, subtraction of matrices is possible and (5A – 3B) also has the same order, i.e. 3 × n.

Q5. If 2p + q p–2q = 4 -3 , then the value of p + q – 2r + s is

5r – s 4r + 3s 11 24

(a) 8 (b) 10 (c) 4 (d) -8


Correct option: (a) 8
Solution: Now, by equating the corresponding elements of these two matrices, we get;
2p + q = 4….(1)
p – 2q = -3….(2)
5r – s = 11….(3)
4r + 3s = 24….(4) By eq (1) × 2 + eq (2), we get;
4p + 2q + p – 2q = 8 – 3
5p = 5 Therefore p = 1
Substituting p = 1 in (1), 2 + q = 4 and q = 4 – 2 = 2
By eq (3) × 3 + eq (4), we get;
15r – 3s + 4r + 3s = 33 + 24
19r = 57 therefore r = 3
Substituting r = 3 in (3),
15 – s = 11 and s = 15 – 11 = 4
Now, p + q – r + 2s = 1 + 2 – 3 + 2(4) = 8
1 0 0
Q6. The matrix 0 2 0 is
0 0 4

(a) identity matrix (b) symmetric matrix (c) skew symmetric matrix (d) none of these
Correct option: (b) symmetric matrix
Q7. For any two matrices A and B, we have
(a) AB = BA (b) AB ≠ BA (c) AB = O (d) None of the above
Correct option: (d) None of the above
Solution: For any two matrices A and B,
AB = BA and AB ≠ BA are not valid unless they follow the condition of matrix multiplication.
Also, AB = O is not true in all cases.
Q8. If A and B are symmetric matrices of the same order, then (AB′ –BA′) is a

(a) Skew symmetric matrix (b) Null matrix (c) Symmetric matrix (d) None of these
Correct option: (a) Skew symmetric matrix
Solution: Given that A and B are symmetric matrices of the same order.
Let’s find the transpose of (AB′ –BA′).
(AB′ –BA′)′ = (AB′)′ – (BA′)′ = (BA′ – AB′) = – (AB′ –BA′)
As (AB′ –BA′)′ = – (AB′ –BA′), the matrix (AB′ –BA′) is skew symmetric.
Q9. If A is a skew-symmetric matrix, then A2 is a
(a) Skew symmetric matrix (b) Symmetric matrix (c) Null matrix (d) Cannot be determined
Correct option: (b) Symmetric matrix
Solution: Given that A is a skew-symmetric matrix, so A′ =-A.
Consider the transpose of A2.
(A2)′ = (AA)′ = A′A′ = (-A)(-A) = A2 ⇒ (A2)′ = A2
Therefore, A2 is a symmetric matrix.
Q10.If A = 0 2 and KA = 0 3a , then the values of k, a and b respectively are

3 -4 2b 2
(a) -6, -12, -18 (b) -6, -4, -9 (c) -6, 4, 9 (d) -6, 12, 18
Correct option: (b) -6, -4, -9

Prepared by

Ajaykumar Patole
PGT-MATHS
K.V.NO.1,Ahmednagar
Ajay Tejwani
PGT Maths
KV AFS, Devlali

Conceptual Mathematics Workshop 2023


KV RHE Pune

DETERMINANTS CASE STUDY QUESTIONS


1. Rani wants to donate a rectangular plot of land for a Hospital in
her village. When she was asked to give dimensions of the plot, she
told that if its length is decreased by 50 m and breadth is increased
by 50m, then its area will remain same, but if length is decreased
by 10m and breadth is decreased by 20m, then its area will
decrease by 5300 m2. Based on the information answer the
following questions.
(a) If x represents length and y represents breadth, then find the two equations in x and y. (1)
(b) Write the matrix form of the equations. (1)
(c) Find x and y. (2)

2. Amar, Akbar and Anthony the three friends went to a stationary shop to purchase some articles. Amar
purchased 1 pencil, 2 erasers and 2 sharpeners and paid Rs. 20 for it. Akbar purchased 2 pencil, 1 erasers
and 2 sharpeners and paid Rs. 28 for it. Anthony purchased 2 pencil, 2 erasers and 1 sharpeners and paid Rs.
27 for it. If cost of a pencil, eraser and sharpener is Rs. x, y and z respectively, then based on the above
information answer the following questions.
(a) Write the three equations. (1)
(b) Using Matrices find the values of x, y and z. (3)

3) Three friends Seeta, Geeta and Reeta are standing at the points S, G
and R respectively and talking on a walky talky. Based on the
information answer the following questions.
(a) Identify the points at which all three are standing? (1.5 M)
(b) Using determinants find the area of Δ SGR (2.5 M)

4) Sunita ask a puzzle to her sister Anita. The puzzle is about three numbers. The numbers are such that their
sum is 6. If 3rd number is multiplied by 2 and added to 1st number then the result is 7. The result is 12 on
adding the 2nd and 3rd number to thrice the 1st number. If the three numbers are x, y and z, then based on the
above information answer the following questions.
(a) Write the three equations. (1)
(b) Using Matrices find the values of x, y and z. (3)
5) Sravan is a nutritionist. He wants to create a mixture of orange juice, beetroot juice and kiwi juice that can
provide 1860 mg of vitamin C, 22 mg of iron and 760 mg of calcium. The quantity of each nutrient per litre of
juice is shown in table below.
Juice Vitamin C (mg/litre) Iron (mg/litre) Calcium (mg/litre)
Orange 500 2 100
Beetroot 20 5 120
Kiwi 800 3 200
If he mixes x, y and z litres of Orange, Beetroot and Kiwi juices, then based on given information, answer the following
questions.
(a) Write the three equations. (1)
(b) Using Matrices find the values of x, y and z. (3)
Ajay Tejwani
PGT Maths
KV AFS, Devlali
Conceptual Mathematics Workshop 2023
KV RHE Pune

DETERMINANTS MCQ
1) Find the value of k < 0, so that the area of triangle with vertices (1, 0), (5, 0) and (1, k) is 6 sq.units.
a) 3 b) -3 c) ± 3 d) -10
2) Find the value of k, so that the three points (1, 1), (3, 4) and (7, k) are collinear
a) 9 b) 10 c) -9 d) -10
3) Given that A is a square a matrix of order 3, such that IAI = λ > 0 and Iadj(A)I = 16, then I5AI =_____
a) 4 b) 20 c) 80 d) 500
4) Given that A is a square a matrix of order 3, and Iadj(A)I = 64, then IAI =_____
a) 8 b) -8 c) 4 d) ±8
5) If A is a square matrix of order 3, such that A(adj A) = 3 I, then Iadj AI =
a) 3 b) 9 c) 27 d) -3
2 12
6) If A = [ ], then the matrix A(adj A) is
1 6
6 −12 −6 12 1 0 0 0
a) [ ] b) [ ] c) [ ] d) [ ]
−1 2 1 −2 0 1 0 0
0 0 −1
7) If A−1 = [ 0 −2 0 ], then |2 A| is a) -16 b) 16 c) 64 d) -64
−1 1 −1
4 5 −7
8) If A = [0 −1 5 ], then |A−1 | is a) 4 b) 16 c) 64 d) 0.25
0 0 −1
𝑥 −2 − 6 −2
9) If | |=| |, then the possible value of x is
2 2𝑥 − 2 2 −10
a) – 5 b) – 6 c) ±8 d) Both a and b
1 2 3
10) The value of determinant |3 4 5| is a) 1 b) – 1 c) 0 d) 5
3 5 6
11) If for a square matrix A of order 3, |4A| = 192, then |𝑎𝑑𝑗 𝐴| =
a) 3 b) 9 c) -3 d) -9
−5 0 0
12) If A is a square matrix of order 3 and A(adj A) = [ 0 −5 0 ], then det.(A-1) =
0 0 −5
a) -5 b) 25 c) -625 d) -0.2
13) If A is a square matrix of order 3 and A(adj A) = 10I, then A =
a) 10 b) 100 c) 1000 d) 0.1
Answers:

Q. No. Answer Q. No. Answer Q. No. Answer


1 C 5 b 9 a
2 B 6 d 10 b
3 d 7 a 11 b
4 d 8 d 12 d
MCQ MATRICES

V S MESHRAM KV VSN NAGPUR

1. If A is a square matrix such that A2 = A, then (I – A)3 + A is equal to

(a) I

(b) 0

(c) I – A

(d) I + A

Correct option: (a) I

Solution:

Given that, A is a square matrix and A2 = A.

Consider (I + A)3, where I is the identity matrix.

Using the identity of (a + b)3 = a3 + b3 + 3ab (a + b), we get;

(I + A)3 = I3 + A3 + 3A2I + 3AI2

= I + A2(A) + 3AI + 3A

= I + A2 + 3A + 3A

= 7A + I {since it is given that A2 = A}

So, (I + A)3 = 7A + I….(1)

Now,

(I + A)3 – 7A = 7A + I – 7A [From (1)]

=I

2 If A and B are two matrices of the order 3 × m and 3 × n, respectively, and m =


n, then the order of matrix (5A – 2B) is

(a) m × 3

(b) 3 × 3

(c) m × n

(d) 3 × n
Correct option: (d) 3 × n

Solution:

Given that, the order of matrix A is 3 × m, and the order of B is 3 × n.

Also, m = n.

So, the order of matrix A and B is the same, i.e. 3 × m.

Thus, subtraction of matrices is possible and (5A – 3B) also has the same order,

i.e. 3 × n.

3) If A and B are symmetric matrices of the same order, then (AB′ –BA′) is a

(a) Skew symmetric matrix

(b) Null matrix

(c) Symmetric matrix

(d) None of these

Correct option: (a) Skew symmetric matrix

Solution:

Given that A and B are symmetric matrices of the same order.

Let’s find the transpose of (AB′ –BA′).

(AB′ –BA′)′ = (AB′)′ – (BA′)′

= (BA′ – AB′)

= – (AB′ –BA′)

As (AB′ –BA′)′ = – (AB′ –BA′), the matrix (AB′ –BA′) is skew symmetric.

4)If A is a skew-symmetric matrix, then A2 is a

(a) Skew symmetric matrix

(b) Symmetric matrix

(c) Null matrix

(d) Cannot be determined

Correct option: (b) Symmetric matrix

Solution:
Given that A is a skew-symmetric matrix, so A′ =-A.

Consider the transpose of A2.

(A2)′ = (AA)′

= A′A′

=(-A)(-A)

= A2

⇒ (A2)′ = A2

Therefore, A2 is a symmetric matrix.

5 For any two matrices A and B, we have

(a) AB = BA

(b) AB ≠ BA

(c) AB = O

(d) None of the above

Correct option: (d) None of the above

Solution:

For any two matrices A and B,

AB = BA and AB ≠ BA are not valid unless they follow the condition of matrix
multiplication.

Also, AB = O is not true in all cases.

Q6)The number of all possible matrices of order 3x3 with each entry 0 and 1 is
a)27 b)18 c)81 d)512
Q6)If A is square matrix such that A2=A then (I+A)3-7A is
a) 0 b)1 c)1/A d) A2

Q7) If A is invertible matrix of order 3 and |A| =5 find |adjA|


a)3 b)5 c)15 d) 25
4 𝑥+2
Q8) If [ ] is symmetric matrix then x is
2𝑥 − 3 𝑥+1
a) 0 b) 3 c)2 d) 5
𝑥−𝑦 2 2 2
Q9)If [ ] = [ ] , then the value of y
𝑥 5 3 5
a)1 b)3 c)2 d)5
Q10)Let A be a square matrix of order 3x3, then |kA| is equal to
a)k|A| b)k2|A| c)3k|A| d) k3|A|
Q11)If A is an m × n matrix such that AB and BA are both defined, then B is a
(a) m × n matrix
(b) n × m matrix
(c) n × n matrix
(d) m × n matrix
Answer:
(b) n × m matrix

Q12) If A2 – A + I = O, then the inverse of A is


(a) I – A
(b) A – I
(c) A
(d) A + I
Answer:
(a) I – A
MCQ OF MATRICES
Q1. If a matrix has 8 elements then the total number of different orders of writing
the matrices.
A) 1 B)2 C)3 D)4
Q2. Let A and B are two matrices and A+B and AB both exist, then
A) A and B are square matrices B) A and B are mXn matrices.
C) A and B are square matrices of same order. D)None
Q3. The number of all possible matrices of order 3x3 with each entry 1 or 2 is
A) 27 B)18 C)81 D)512
Q4. From the following, Identify the wrong statement.
A) Matrix multiplication satisfies associative property.
B) Matrix multiplication is distributive over addition.
C) Matrix multiplication satisfies commutative property.
D) For every non-singular square matrix, inverse exists.
Q5. If A and B are symmetric matrices of same order, then AB - BA is a
A) Skew Symmetric matrix B) Zero matrix
C) Identity matrix D) Symmetric matrix
Q6. The principal diagonal elements of a skew symmetric matrix are
A) 1 B) 0 C) 0 or 1 D) None of these
Q7. The number of all possible matrices of order 3 × 3 with each entry 0 or 1 or 2
is:
A) 33 B)35 C) 38 D) 39
Q8. If A and B are symmetric matrices of same order, then
A) Skew symmetric matrix B) Null matrix
C) Symmetric matrix D) None of these
Q9. If A and B are matrices of same order, then (AB′–BA′) is a
A) skew symmetric matrix B) null matrix
C) symmetric matrix D) unit matrix
Q10. If A is a 2 × 3 matrix and AB is a 2 × 5 matrix, then B must be a
A) 3 × 5 matrix B) 5 × 3 matrix
C) 3 × 2 matrix D) 5 × 2 matrix

ANSWERS
1.D 2.C 3.D 4.C 5.A 6.B 7.D 8.A 9.A 10.A
1). If for matrix A, |A| = 3, where matrix A is of order 2 × 2, then |5 A| is ____
a) 9 b) 75 c) 15 d)
2
2). If the points A (3, -2), B(k,2) and C (8,8) are collinear, then the value of k is:
a) 2 b) -3 c) 5 d)
-4
3). Find the area of the triangle whose vertices are (3, 8), (– 4, 2) and (5, 2)
a) 18 b) 34 c) 27 d)
61

4). The value of is:


1 √3
a) 1 b) c) d)
2 2

0
5). If A is a square matrix such that A2 = I, then A-1 is equal to
a) 2A b) O c) A d)
A+I
6). If area of triangle is 35 sq units with vertices (2, – 6), (5, 4) and (k, 4). Then k is
a) 12 b) -2 c) -12, -2 d)
12, -2
7). A square matrix A is said to be singular if IAI =
a) 1 b) -1 c) 0 d)
None of these

8). If Δ = and Aij is Cofactors of aij, then value of Δ is given by:

a) a11 A31+ a12 A32 + a13 A33 c) a11 A11+ a12 A21 + a13 A31
b) a21 A11+ a22 A12 + a23 A13 d) a11 A11+ a21 A21 + a31 A31
9.
then x is equal to:
a) 6 b) ±6 c) -1
d) -6
10 Given that A is a square matrix of order 3 and | A | = - 4, then | adj A | is equal to:
. a) 4 b) -4 c) 16 d)
-16
11
. Given that A = and A2 = 3I, then:
a) 1 + 𝛼2 + 𝛽𝛾 = 0 b) 1 − 𝛼2 − 𝛽𝛾 = 0
c) 3 − 𝛼2 − 𝛽𝛾 = 0 d) 3 + 𝛼2 + 𝛽𝛾 = 0
12
. Find the minor of the element 7 in the determinant

a)23 b) -23 c) 24 d)
0
13
. If A, B and C are angles of a triangle, then the
determinant

a) 0 b) -1 c) 1 d)
2
14
. Find the minor of the element of second row and third column in
the following det
a) 13 b) 4 c) 5 d) 0
15 If A(3,4), B(-7,2) and C(x,y) are collinear, then:
.
a) x+5y+17=0 b) x+5y+13=0 c) x-5y+17=0 d)
none of these
16
.

17 If the points (a1, b1), (a2, b2) and (a1 + a2, b1 + b2) are collinear, then
. a) a1b2 = a2b1 b)a1 + a2 = b1 + b2 c)a2b2 = a1b1 d)a1 +
b1 = a2 + b2
18 18). Compute (AB)-1 if:
.

19 The area of a triangle with vertices (-3, 0), (3, 0) and (0, k) is 9 sq. units. then k=
. a) 9 b) 3 c) -9 d) 6
20
.
21 Let A be a non-singular square matrix of order 3 × 3. Then |adj A| is equal to:
. a) | A | b) | A |2 c) | A |3 d)
3|A|

22
.

23 If A is a square matrix of order 4 such that |adj A| = 125, then |A| is ____
. a) 25 b) 5 c) 15 d)
625
24 Which of the following is a correct statement?
. a) Determinant is a square matrix
b) Determinant is a number associated to a matrix
c) Determinant is a number associated with the order of the matrix
d) Determinant is a number associated to a square matrix
25
.

a) −6, −12, −18 b) −6, −4, −9 c) −6, 4, 9


d) −6, 12, 1

Answers
1.B 2.C 3.C 4.C 5.C 6.D 7.C 8.D 9.A 10.C

11.C 12.B 13.A 14.A 15.C 16.A 17.A 18.A 19.B 20.B

21.B 22.B 23.B 24.D 25.B

ASSERTION AND REASONING TYPE QUESTIONS

1. Assertion The value of x for which |


3 𝑥
| =|
3 2
| is ± 2√2
𝑥 1 4 1
(A)
Reason(R) The determinant of a matrix A order 2x2, A= [
𝑎 𝑏]
𝑐 𝑑
is = ad – bc
A Both A and R are true and R is the correct explanation of A
B Both A and R are true but R is NOT the correct explanation of A.
C A is true but R is false
D A is false but R is true
E Both A and R are false

2. Assertion The value of x for which |


𝑥 2
| =|
6 2
| is ± 6
18 𝑥 18 6
(A)
Reason(R) The determinant of a matrix A order 2 x 2 , A= [
𝑎 𝑏]
𝑐 𝑑
is = ab–dc
A Both A and R are true and R is the correct explanation of A
B Both A and R are true but R is NOT the correct explanation of A.
C A is true but R is false
D A is false but R is true
E Both A and R are false

3. Assertion 1 0 1
If A= [0 1 2] then |3𝐴| =9|𝐴|
(A) 0 0 4
Reason(R) If A is a square matrix of order n then |𝑘𝐴| =kn|𝐴|
A Both A and R are true and R is the correct explanation of A
B Both A and R are true but R is NOT the correct explanation of A.
C A is true but R is false
D A is false but R is true
E Both A and R are false

4. Assertion If A is a non singular square matrix of order 3x3


(A) and|𝐴| =5 then |𝑎𝑑𝑗𝐴| is equal to 125
Reason(R) |𝑎𝑑𝑗𝐴| =(|𝐴| )n-1where n is order of A.
A Both A and R are true and R is the correct explanation of A
B Both A and R are true but R is NOT the correct explanation of A.
C A is true but R is false
D A is false but R is true
E Both A and R are false

5. Assertion Let A-1= [


5 −7]and -1 [7
B =
6
]then (AB) -1
−2 3 8 7
(A)
23 31
=[ ]
26 35
Reason(R) (AB) -1
= A-1B-1
A Both A and R are true and R is the correct explanation of A
B Both A and R are true but R is NOT the correct explanation of A.
C A is true but R is false
D A is false but R is true
E Both A and R are false

6. Assertion 1 2 0
Value of x for which the matrix [ 0 1 2] is singular
(A) −1 2 𝑥
is 5
Reason(R) A square matrix is singular if |𝐴| =0
A Both A and R are true and R is the correct explanation of A
B Both A and R are true but R is NOT the correct explanation of A.
C A is true but R is false
D A is false but R is true
E Both A and R are false

7. Assertion 2 3 1
The minor of the element 3 in the matrix [0 −2 4]is
(A) 2 1 5
8.
Reason(R) : Minor of an element aij of a matrix is the
determinant obtained by deleting its jth row and i th
column
A Both A and R are true and R is the correct explanation of A
B Both A and R are true but R is NOT the correct explanation of A.
C A is true but R is false
D A is false but R is true
E Both A and R are false

8. Assertion For two matrices A and B of order 3, |𝐴|=2|𝐵| = -3


(A) then if|2𝐴𝐵| is -48.
Reason(R) For a square matrix A, A(adj A)=(adj A)A=|𝐴| I
A Both A and R are true and R is the correct explanation of A
B Both A and R are true but R is NOT the correct explanation of A.
C A is true but R is false
D A is false but R is true
E Both A and R are false
9. Assertion Values of k for which area of the triangle with
(A) vertices (2, -6), (5,4) and (k,4) is 35 sq units are 12,
2.
Reason(R) Area of a triangle with vertices A (x 1, y1),B (x2, y2)
𝑥1 𝑦1 1
1
and C (x3, y3) is |𝑥2 𝑦2 1|
2
𝑥3 𝑦3 1
A Both A and R are true and R is the correct explanation of A
B Both A and R are true but R is NOT the correct explanation of A.
C A is true but R is false
D A is false but R is true
E Both A and R are false

10. Assertion The points A(a, b+c), B(b, c+a) and C(c, a+b) are
(A) collinear.
Reason(R) Three points A (x1, y1) , B(x2, y2) and C(x3, y3) are
collinear if area of a triangle ABC is zero.
A Both A and R are true and R is the correct explanation of A
B Both A and R are true but R is NOT the correct explanation of A.
C A is true but R is false
D A is false but R is true
E Both A and R are false

11. Assertion 1 −1 2
(A) Inverse of the matrix [0 2 −3] is the
3 −2 4
−2 0 1
matrix[ 9 2 −3]
6 1 −2
Reason(R) : Inverse of a square matrix A, if it exits is given by
1
A-1 =𝐼𝐴𝐼 adjA
A Both A and R are true and R is the correct explanation of A
B Both A and R are true but R is NOT the correct explanation of A.
C A is true but R is false
D A is false but R is true
E Both A and R are false
12. Assertion For a matrix A=[
2 −1
], A. adj A =[
4 0
]
−3 4 0 4
(A)
Reason(R) For a square matrix A , A( adj A) = (adj A)A= |𝐴|
I
A Both A and R are true and R is the correct explanation of A
B Both A and R are true but R is NOT the correct explanation of A.
C A is true but R is false
D A is false but R is true
E Both A and R are false
13. Assertion In a square matrix of order 3 the minor of an element
(A) a22 is 6 then cofactor of a22is -6.
Reason(R) Cofactor an element aij = AIJ= ( -1)i+jMij
A Both A and R are true and R is the correct explanation of A
B Both A and R are true but R is NOT the correct explanation of A.
C A is true but R is false
D A is false but R is true
E Both A and R are false

14. Assertion Inverse of a matrix A =[


2 3
] is the matrix A-1
1 2
(A)
2 −3
=[ ]
−1 2
Reason(R) : Inverse of a square matrix (
𝑎 𝑏
)is (
𝑑 −𝑏
).
𝑐 𝑑 −𝑐 𝑎
A Both A and R are true and R is the correct explanation of A
B Both A and R are true but R is NOT the correct explanation of A.
C A is true but R is false
D A is false but R is true
E Both A and R are false
15. Assertion If A is an invertible matrix of order 2, and det A= 3
(A) then det( A-1)is equal to
1
3

Reason(R) If A is an invertible matrix of order 2 then det (A -1)


= det 𝐴
A Both A and R are true and R is the correct explanation of A
B Both A and R are true but R is NOT the correct explanation of A.
C A is true but R is false
D A is false but R is true
E Both A and R are false

16. Assertion The equation of the line joining (1,2) and (3,6)
(A) using determinants is y= 3x.
Reason(R) The area of ∆PAB is zero if P(x, y) is a point on the
line joining a A and B.
A Both A and R are true and R is the correct explanation of A
B Both A and R are true but R is NOT the correct explanation of A.
C A is true but R is false
D A is false but R is true
E Both A and R are false

CASE STUDY TYPE QUESTIONS


Case Study-1

Three shopkeepers Ujjwal, Lohith, and Kundan are using polythene bags,
handmade bags and newspaper's envelope as carry bags. It is found that the
shopkeepers Ujjwal, Lohith, and Kundan are using (20, 30, 40), (30, 40, 20),
and (40, 20, 30) polythene bags, handmade bags, and newspapers envelopes
respectively. They spent ₹250, ₹270, and ₹200 on these carry bags
respectively. Let the cost of polythene bag, handmade bag and newspaper
envelope costs are x,y and z respectively.
1. What is the Linear equation representing amount spent by Lohith on carry
bags?

a. 20x + 30 y + 40 z = 250 b. 30x + 40 y + 20 z = 270


c. 40x + 20 y + 30 z = 270 d. 250x + 270 y + 200
z=0

2. What is the Linear equation representing amount spent by Lohith on carry


bags?

a. 20x + 30 y + 40 z = 250 b. 30x + 40 y + 20 z = 270


c. 40x + 20 y + 30 z = 270 d. 250x + 270 y + 200 z = 0

3. 2 3 4
Adjoint of [3 4 2] =
4 2 3
8000 −1000 −10000 8 −1 −10
a. [ −1000 −10000 8000 ] b. [ −1 −10 8 ]
−10000 8000 −1000 −10 8 −1
20 30 40 4 3 2
c. [30 40 20] d. [2 4 3]
40 20 30 3 2 4

4. What is the cost of one newspaper bag?

a. ₹ 1 b. ₹ 2 c. ₹ 3 d. ₹ 5

5. Find the total amount spent by ujjwal for handmade bags ?

a. 100 b. 200 c. 150 d. 250

Case Study-2

Each triangular face of the square pyramid of Peace in


Kazakhstan is made up of 25 smaller equilateral triangles as
shown in the figure.
Using the above information and concept of determinants,
answer the following questions.
1. If the vertices of one of the smaller equilateral triangles are (0, 0),
(3, √3) and (3, - √3), then the area of such triangle is
a. √3sq. units b. 2 √3 sq. units c. 3√3 sq. units d. none
of these
2. The lateral surface area of the Pyramid is
a. 300√3 sq. unit b. 75 sq. unit c. 75 √3 sq. unit d.
300 sq. unit
3. The length of each altitude of a smaller equilateral triangle is
a. 2 units b. 3 units c. 2 √3 units d. 4
units
4. If (2, 4), (2, 6) are two vertices of a smaller equilateral triangle, then the
third vertex is
a. (2 ± √3, 5) b. (2 ± √3, ±5)c. (2 ± √5, 3) d. (2 ± √5, ±3)
5. Let A (a, 0), B (0, b) and C (1, 1) be three points. If 1 + 1 = 1, then
𝑎 𝑏
the three points are
a. vertices of an equilateral triangle b. vertices of a right-angled
triangle
c. collinear d. vertices of an isosceles
triangle

Case Study-3

Area of a triangle whose vertices are (x1, y1), (x2, y2) and (x3, y3) is given by
𝑥1 𝑦1 1
the determinant Δ = 1/2[𝑥2 𝑦2 1]
𝑥3 𝑦3 1

Since, area is a positive quantity, so we always take the absolute value of the
determinant A. Also, the area of the triangle formed by three collinear points is
zero.
Based on the above information, answer the following questions.
1. Find the area of the triangle whose vertices are (-2, 6), (3, -6), and (1,
5).
a. 30 sq. units b. 35 sq. units c. 40 sq. units d. 15.5 sq. units

2. If the points (2, -3), (k, -1) and (0, 4) are collinear, then find the value of
4k.
7 40
a. 4 b. c. 47 d.
140 7
3. If the area of a triangle ABC, with vertices A (1, 3), B (0, 0) and C (k, 0)
is 3 sq. units, then a value of k is
a. 2 b. 3 c. 4 d. 5
4. Using determinants, find the equation of the line joining the points
A(1,2) & B(3,6).
y = 2x b. x = 3y c. y = x d. 4x - y = 5

5. If A is (11, 7), B is(5, 5) and C is (-1, 3), then


a)△ABC is scalene triangle c. △ABC is equilateral triangle
b) A, B and C are collinear d. None of these

Answers
ASSERTION AND REASONING
1 A 2 C 3 D 4 D 5 E

6 D 7 E 8 B 9 D 10 A

11 A 12 D 13 D 14 C 15 C

16 D 17 B 18 C 19 D 20 B

CASE STUDY
CS-1 I)b II) d III) b iv) b V) c

CS-2 I)c II) a III) b iv) a V) c

CS-3 I)d II) d III) a iv) a V) b


Prepared by

Devendra Malvi (Resource Person)


PGT (Mathematics)
Kendriya Vidyalaya O F Chanda
Contact No. 9424948510
Email ID : malvi.baba1960@gmail.com

Work Shop on capacity building


Venue: KV RHE Pune
Course Director: Sh. Dilip Singh Bhati, Principal KV Ambarnath
CONTINNUITY AND DIFFERENTIABILITY
Content:
I Multiple Choice Questions
II Assertion and Reason based questions
III Case Based Questions
I Multiple Choice Questions
1
𝑥𝑠𝑖𝑛 (𝑥) 𝑤ℎ𝑒𝑛 𝑥≠0
1. Let 𝑓 (𝑥 ) = { . Then,
0 𝑤ℎ𝑒𝑛 𝑥 = 0
(a) 𝑓(𝑥 ) does not defined at 𝑥 ≠ 0.
(b) lim 𝑓 (𝑥) = 1
𝑥→0
(c) lim 𝑓 (𝑥) does not exist
𝑥→0
(d) 𝑓(𝑥) is continuous at 𝑥 = 0
2. The function defined by 𝑔(𝑥 ) = 𝑥 − [𝑥 ] is discontinuous at:
(a) All rational point
(b) All irrational points
(c) All integral points
(d) None of the above
1
3. Given 𝑓 (𝑥 ) = 1+𝑡𝑎𝑛𝑥 , then:
(a) 𝑓(𝑥) is a continuous, real valued function for all 𝑥 ∈ (−∞, ∞)
3𝜋
(b) Is discontinuous only at 𝑥 = .
4
(c) Has only finitely many discontinuities on (−∞, ∞).
(d) Has infinitely many discontinuities on (−∞, ∞).
𝑥
, 𝑥≠0
4. If a function 𝑓(𝑥) is defined as 𝑓 (𝑥 ) = {√𝑥 2 then:
0, 𝑥=0
(a) 𝑓(𝑥 ) is continuous at 𝑥 = 0 but not differentiable at 𝑥 = 0.
(b) 𝑓(𝑥 ) is continuous as well as differentiable at 𝑥 = 0.
(c) 𝑓(𝑥 ) is discontinuous at 𝑥 = 0.
(d) None of these.
log(1+𝑎𝑥)−log(1−𝑏𝑥)
5. Let 𝑓 (𝑥 ) = , 𝑥 ≠ 0. If 𝑓(𝑥 ) is continuous at 𝑥 = 0 then 𝑓(0) =
𝑥
(a) 𝑎 − 𝑏 (b) 𝑎 + 𝑏 (c) 𝑏 − 𝑎 (d) 𝑙𝑜𝑔𝑎 + 𝑙𝑜𝑔𝑏
[𝑥]−1
, 𝑥≠1
6. If 𝑓(𝑥 ) = { 𝑥−1 then 𝑓 (𝑥 ) is:
0, 𝑥=1
(a) Continuous as well as differentiable at 𝑥 = 1.
(b) Differentiable but not continuous at 𝑥 = 1.
(c) Continuous but not differentiable at 𝑥 = 1.
(d) Neither continuous nor differentiable at 𝑥 = 1.
1
𝑥 𝑘 𝑐𝑜𝑠 (𝑥) , 𝑥 ≠ 0
7. If 𝑓(𝑥 ) = { is continuous at 𝑥 = 0, then
0, 𝑥=0
(a) 𝑘 < 0 (b) 𝑘 > 0 (c) 𝑘 = 0 (d) 𝑘 ≥ 0
8. If 𝑓(𝑥 ) = |𝑠𝑖𝑛𝑥|, then
(a) f is everywhere differentiable
(b) f is everywhere continuous but not differentiable at 𝑥 = 𝑛𝜋, 𝑛 ∈ ℤ.
𝜋
(c) f is everywhere continuous but not differentiable at 𝑥 = (2𝑛 + 1) 2 , 𝑛 ∈ ℤ.
(d) None of the above.
𝑥+2
, 𝑖𝑓 𝑥 ∈ ℝ − {−1, −2}
𝑥 2 +3𝑥+2
9. If 𝑓: ℝ → ℝ is defined by 𝑓 (𝑥 ) = { −1, 𝑖𝑓 𝑥 = −2 then 𝑓 is continuous on:
0, 𝑖𝑓 𝑥 = −1
(a) ℝ (b) ℝ − {−2} (c) ℝ − {2} (d) ℝ − {−1, −2}
|𝑥| 2
10. If 𝑓(𝑥 ) = 𝑎𝑒 + 𝑏|𝑥 | , 𝑎, 𝑏 ∈ ℝ and 𝑓(𝑥) is differentiable at 𝑥 = 0. Then, 𝑎 and 𝑏 are:
(a) 𝑎 = 0, 𝑏 ∈ ℝ (b) 𝑎 = 1, 𝑏 = 2 (c) 𝑏 = 0, 𝑎 ∈ ℝ (d) 𝑎 = 4, 𝑏 = 5
𝜋
11. If 𝑓(𝑥 ) = |𝑐𝑜𝑠𝑥 − 𝑠𝑖𝑛𝑥 |, then 𝑓′ ( 3 ) is equal to:
2 √3+1
(a) √3+1 (b) (c) (d) none of these
√3+1 2
𝑥3
12. The derivative of 𝑒 with respect to 𝑙𝑜𝑔𝑥 is:
3 3 3 3
(a) 𝑒 𝑥 (b) 3𝑥 2 𝑒 𝑥 (c) 3𝑥 3 𝑒 𝑥 (d) 3𝑥 3 𝑒 𝑥 + 3𝑥 2
𝑑
13. If 2𝑓(𝑠𝑖𝑛𝑥 ) + 𝑓 (𝑐𝑜𝑠𝑥 ) = 𝑥, then 𝑓(𝑥) is: 𝑑𝑥
1
(a) 𝑠𝑖𝑛𝑥 + 𝑐𝑜𝑠𝑥 (b) 2 (c) √1−𝑥 2 (d) none of these
14. Which of the following is/are true?
𝑑𝑦 𝜃
Statement I: If 𝑥 = 𝑎(𝜃 − 𝑠𝑖𝑛𝜃), 𝑦 = 𝑎(1 + 𝑐𝑜𝑠𝜃) then 𝑑𝑥 = −𝑐𝑜𝑡 (2 )
𝑠𝑖𝑛 3 𝑡 𝑐𝑜𝑠 3𝑡
Statement II: If = ,𝑦 = , then derivative of 𝑦 with respect to 𝑥 is −𝑐𝑜𝑡3𝑡.
√ 𝑐𝑜𝑠2𝑡 √𝑐𝑜𝑠2𝑡
(a) Only I is true (b) Only II is true (c) Both I and II are true (d) Neither I nor II is true
1
15. If 𝑓(𝑥 ) = 𝑥 − 1for all positive 𝑥 ≠ 1 and if 𝑓 is continuous at 1, then 𝑥 equals:
𝑥
1
(a) 0 (b) 𝑒 (c) 𝑒 (d) 𝑒 2
𝑑𝑦
16. If 𝑦 = 𝑠𝑒𝑐(𝑡𝑎𝑛−1 𝑥 ), then 𝑑𝑥 at 𝑥 = 1 is equal to:
1 1
(a) √2 (b) (c) 1 (d) 2
√2
𝑘𝑐𝑜𝑠𝑥 𝜋
, 𝑖𝑓 𝑥≠ 2
𝜋−2𝑥
17. The value of k for which the function f given by 𝑓(𝑥 ) = { 𝜋 is continuous at 𝑥 =
5, 𝑖𝑓 𝑥 = 2
𝜋
, is:
2
5
(a) 6 (b) 5 (c) 2 (d) 10
18. If 𝑥 = 𝑎𝑐𝑜𝑠𝜃 + 𝑏𝑠𝑖𝑛𝜃, 𝑦 = 𝑎𝑠𝑖𝑛𝜃 − 𝑏𝑐𝑜𝑠𝜃 then which one of the following is true?
𝑑2 𝑦 𝑑𝑦 𝑑2 𝑦 𝑑𝑦
(a) 𝑦 2 𝑑𝑥 2 − 𝑥 𝑑𝑥 + 𝑦 = 0 (b) 𝑦 2 𝑑𝑥 2 + 𝑥 𝑑𝑥 + 𝑦 = 0
𝑑2 𝑦 𝑑𝑦 𝑑2 𝑦 𝑑𝑦
(c) 𝑦 2 𝑑𝑥 2 + 𝑥 𝑑𝑥 − 𝑦 = 0 (d) 𝑦 2 𝑑𝑥 2 − 𝑥 𝑑𝑥 − 𝑦 = 0
𝑥
19. Derivative of 𝑡𝑎𝑛−1 (√1−𝑥 2 ) with respect to 𝑠𝑖𝑛−1 (2𝑥√1 − 𝑥 2 ) is:
1 1 1
(a) − 4 (b) 2 (c) 2 (d) − 2
𝜋 𝑥 𝑑𝑦
20. If 𝑦 = 𝑙𝑜𝑔 [𝑡𝑎𝑛 ( 4 + 2)], then 𝑑𝑥 is:
(a) 𝑠𝑒𝑐𝑥 (b) 𝑐𝑜𝑠𝑒𝑐𝑥 (c) 𝑡𝑎𝑛𝑥 (d) 𝑠𝑒𝑐𝑥 ∙ 𝑐𝑜𝑠𝑒𝑐𝑥
II. Assertion and Reason Questions
2𝑥 1−𝑥 2
21. Assertion (A): The derivative of 𝑠𝑖𝑛−1 ( ) with respect to 𝑐𝑜𝑠 −1 ( ) is 1.
1+𝑥 2 1+𝑥 2
𝑓′(𝑥)
Reason (R): If 𝑢 = 𝑓(𝑥) and 𝑣 = 𝑔(𝑥) the derivative of 𝑢 with respect to 𝑣 is 𝑔′(𝑥). (a)
𝑑𝑦
22. Assertion (A): If = 𝑙𝑜𝑔√𝑡𝑎𝑛𝑥 , then the value of (𝑑𝑥 ) 𝜋 = 1.
𝑥=
4
Reason (R) : The value of 𝑙𝑜𝑔1 is not defined. (c)
𝑑2 𝑦 −1
23. Assertion (A) : If 𝑥 = 𝑎𝑡 2 and 𝑦 = 2𝑎𝑡, then (𝑑𝑥 2 ) = 16𝑎
𝑡=2
𝑑2 𝑦 𝑑𝑦 2 𝑑𝑥 2
Reason (R) : (𝑑𝑥 2 ) = ( 𝑑𝑡 ) × ( 𝑑𝑡 ) (c)
𝑑𝑦 𝜋
24. Assertion (A) : If 𝑦 = 𝑠𝑒𝑐𝑥 𝑜 , then 𝑑𝑥 = 180𝑜 𝑠𝑒𝑐𝑥 𝑜 𝑡𝑎𝑛𝑥 𝑜.
Reason (R) : To find whether a function is continuous or differentiable at a point the angle should
be in radian measure only. (a)
𝑑𝑦 𝑙𝑜𝑔10 𝑒 𝑦
25. Assertion (A) : If 𝑦 = 𝑙𝑜𝑔10 𝑥 + 𝑙𝑜𝑔𝑒 𝑦 then 𝑑𝑥 = (𝑦−1).
𝑥
𝑑 𝑙𝑜𝑔𝑥
Reason (R) : 𝑑𝑥 (𝑙𝑜𝑔𝑎 𝑥) = 𝑙𝑜𝑔𝑎 (a)
III Case Based Questions:
𝑓(𝑎−ℎ)−𝑓(𝑎)
1. Let 𝑓(𝑥) be a real valued function, then its Left Hand Derivative is𝐿𝑓 ′ (𝑎) = lim
ℎ→0 −ℎ
′( 𝑓(𝑎+ℎ)−𝑓(𝑎)
and Right hand derivative is 𝑅𝑓 𝑎) = lim . Also a function 𝑓(𝑥) is said to be
ℎ→0 ℎ
differentiable at 𝑥 = 𝑎 if its LHD and RHD at 𝑥 = 𝑎 exist and are equal.
5𝑥 − 4 𝑖𝑓 0 < 𝑥 < 1
For the function 𝑓 (𝑥 ) = { 4𝑥 2 − 3𝑥 𝑖𝑓 1 ≤ 𝑥 < 2
3𝑥 + 4 𝑖𝑓 𝑥 ≥ 2
Based on above information answer the following questions:
(a) Find RHD of 𝑓(𝑥) at 𝑥 = 2. (3)
(b) Find LHD of 𝑓(𝑥) at 𝑥 = 2. (13)
(c) Find the point where 𝑓(𝑥) is not differentiable (𝑥 = 2)
3
(d) Find the value of 𝑓′ (2). (9)
𝑑𝑦 𝑑𝑦 𝑑𝑡
2. Let 𝑥 = 𝑓(𝑡) and 𝑦 = 𝑔(𝑡) be parametric forms with 𝑡 as a parameter, then 𝑑𝑥 = × 𝑑𝑥 =
𝑑𝑡
𝑔′(𝑡)
where 𝑓′(𝑡) ≠ 0.
𝑓′(𝑡)
On the basis of above information, solve the following questions.
𝜋
(1) Find the derivative of 𝑓(𝑐𝑜𝑡𝑥) with respect to 𝑔(𝑐𝑜𝑠𝑒𝑐𝑥) at 𝑥 = , where 𝑓 ′ (1) = 2 and
4
′( 1
𝑔 √2) = 4. ( 2)

(2) Find the derivative of 𝑠𝑖𝑛−1 𝑥 with respect to 𝑐𝑜𝑠 −1 𝑥. (−1)
1 2 𝑑𝑦 2
(3) If 𝑦 = 4 𝑟 4 and 𝑟 = 3 𝑥 3 + 5, then find 𝑑𝑥 . (27 𝑥 2 (2𝑥 3 + 15)3 ).
Case Study Questions - Class 12 - Mathematics
Continuity and Differentiability

Q.1. Let 𝑓(𝑥) be a real valued function, then its


𝑓(𝑎−ℎ)−𝑓(𝑎)
• Left Hand Derivative (L.H.D.) : 𝐿𝑓 ′ (𝑎) = lim
h→0 −ℎ
𝑓(𝑎+ℎ)−𝑓(𝑎)
• Right Hand Derivative (R.H.D.) : 𝑅𝑓 ′ (𝑎) = lim
h→0 ℎ
Also, a function 𝑓(𝑥) is said to be differentiable at 𝑥 = 𝑎 if its L.H.D. and
R.H.D. at 𝑥 = 𝑎 exist and are equal.
|𝑥 − 3|, 𝑥≥1
For the function (𝑥) = {𝑥 2 3𝑥 13 , answer the following questions.
− + , 𝑥<1
4 2 4
(i) R.H.D. of 𝑓(𝑥) at 𝑥 = 1 is
(a) 1 (b) −1 (c) 0 (d) 2
(ii) L.H.D. of 𝑓(𝑥) at 𝑥 = 1 is
(a) 1 (b) −1 (c) 0 (d) 2
(iii) 𝑓(𝑥) is non-differentiable at
(a) 𝑥 = 1 (b) 𝑥 = 2 (c) 𝑥 = 3 (d) 𝑥 = 4
(iv) Find the value of 𝑓 ′ (2).
(a) 1 (b) 2 (c) 3 (d) −1
(v) The value of 𝑓′(−1) is
(a) 2 (b) 1 (c) −2 (d) −1

Q.2. Let 𝑥 = 𝑓(𝑡) and 𝑦 = 𝑔(𝑡) be parametric forms with 𝑡 as a parameter, then
𝑑𝑦 𝑑𝑦 𝑑𝑡 𝑔′ (𝑡)
= × = , where 𝑓′(𝑡) ≠ 0.
𝑑𝑥 𝑑𝑡 𝑑𝑥 𝑓′ (𝑡)
On the basis of above information, answer the following questions.
𝜋
(i) The derivative of 𝑓(tan 𝑥) w.r.t. 𝑔(sec 𝑥) at 𝑥 = , where 𝑓 ′ (1) = 2 and
4
𝑔′(√2) = 4, is
1
(a) (b) √2 (c) 1 (d) 0
√2
2𝑥 1−𝑥 2
(ii) The derivative of sin−1 ( ) with respect to cos −1 (1+𝑥 2 ) is
1+𝑥 2
(a) −1 (b) 1 (c) 2 (d) 4
𝑥3
(iii) The derivative of 𝑒 with respect to log 𝑥 is
𝑥3 3 3 3
(a) 𝑒 (b) 3𝑥 2 2𝑒 𝑥 (c) 3𝑥 3 𝑒 𝑥 (d) 3𝑥 2 𝑒 𝑥 + 3𝑥
(iv) The derivative of cos −1 (2𝑥 2 − 1) w.r.t. cos −1 𝑥 is
−1 2
(a) 2 (b) (c) (d) 1 − 𝑥 2
2√1−𝑥 2 𝑥
1 2 𝑑𝑦
(v) If 𝑦 = 𝑢4 and 𝑢 = 𝑥 3 + 5, then =
4 3 𝑑𝑥
2 2 2 2
(a) 𝑥 (2𝑥 3 + 15)3 (b) 𝑥 (2𝑥 3 + 15)3
27 7
2 2
(c) 𝑥(2𝑥 3 + 5)3 (d) (2𝑥 3 + 15)3
27 7

Q.3. The function 𝑓(𝑥) will be discontinuous at 𝑥 = 𝑎 if 𝑓(𝑥) has


• Discontinuity of first kind : lim 𝑓(𝑎 − ℎ) and lim 𝑓(𝑎 + ℎ) both exist but
h→0 h→0
are not equal. If is also known as irremovable discontinuity.
• Discontinuity of second kind : If none of the limits lim 𝑓(𝑎 − ℎ) and lim
h→0 h→0

𝑓(𝑎 + ℎ) exist.
• Removable discontinuity : lim 𝑓(𝑎 − ℎ) and lim 𝑓(𝑎 + ℎ) both exist and
h→0 h→0

equal but not equal to 𝑓(𝑎).


Based on the above information, answer the following questions.
𝑥 2 −9
, for 𝑥 ≠ 3
(i) If 𝑓(𝑥) = { 𝑥−3 , then at 𝑥 = 3
4, for 𝑥 = 3
(a) 𝑓 has removable discontinuity
(b) 𝑓 is continuous
(c) 𝑓 has irremovable discontinuity
(d) None of these
𝑥 + 2 , if 𝑥 ≤ 4
(ii) If 𝑓(𝑥) = { , then at 𝑥 = 4
𝑥 + 4, if 𝑥 > 4
(a) 𝑓 is continuous
(b) 𝑓 has removable discontinuity
(c) 𝑓 has irremovable discontinuity
(d) None of these
𝑥 2 −4
, for 𝑥 ≠ 2
(iii) Consider the function 𝑓(𝑥) defined as 𝑓(𝑥) = { 𝑥−2 , then at
5, for 𝑥 = 2
𝑥=2

(a) 𝑓 has removable discontinuity


(b) 𝑓 has irremovable discontinuity
(c) 𝑓 is continuous
(d) 𝑓 is continuous if 𝑓(2) = 3
𝑥−|𝑥|
, 𝑥≠0
(iv) If 𝑓(𝑥) = { 𝑥 , then at 𝑥 = 0
2, 𝑥=0

(a) 𝑓 is continuous
(b) 𝑓 has removable discontinuity
(c) 𝑓 has irremovable discontinuity
(d) None of these
𝑒 𝑥 −1
, if 𝑥 ≠ 0
(v) If 𝑓(𝑥) = {log(1+2𝑥) , then at 𝑥 = 0
7, if 𝑥 = 0
(a) 𝑓 is continuous if 𝑓(0) = 2
(b) 𝑓 is continuous
(c) 𝑓 has irremovable discontinuity
(d) 𝑓 has removable discontinuity

Q.4. If 𝑦 = 𝑓(𝑢) is a differentiable function of 𝑢 and 𝑢 = 𝑔(𝑥) is a differentiable


𝑑𝑦 𝑑𝑦 𝑑𝑢
function of 𝑥, then 𝑦 = 𝑓(𝑔(𝑥)] is a differentiable function of 𝑥 and = ×
𝑑𝑥 𝑑𝑢 𝑑𝑥
. This rule is also known as CHAIN RULE.
Based on the above information, find the derivative of functions w.r.t. 𝑥 in the
following questions.
(i) cos √𝑥
− sin √𝑥 sin √𝑥
(a) (b) (c) sin √𝑥 (d) − sin √𝑥
2√𝑥 2√𝑥
1
𝑥+
(ii) 7 𝑥
1 1
𝑥 2 −1 𝑥 2 +1
(a) (
𝑥2
) ∙ 7𝑥+𝑥 ∙ log 7 (b) (
𝑥2
) ∙ 7𝑥+𝑥 ∙ log 7
1 1
𝑥 2 −1 𝑥− 𝑥 2 +1
(c) (
𝑥2
)∙7 𝑥 ∙ log 7 (d) (
𝑥2
) ∙ 7𝑥−𝑥 ∙ log 7
1−cos 𝑥
(iii) √
1+cos 𝑥
1 2𝑥 1 𝑥 𝑥 𝑥
(a) sec (b) − sec 2 (c) sec 2 (d) −sec 2
2 2 2 2 2 2
1 𝑥 1 𝑥
(iv) tan−1 ( ) + tan−1 ( )
𝑏 𝑏 𝑎 𝑎
−1 1 1 1
(a) + (b) +
𝑥 2 +𝑏 2 𝑥 2 +𝑎2 𝑥 2 +𝑏 2 𝑥 2 +𝑎2
1 1
(c) − (d) None of these
𝑥 2 +𝑏2 𝑥 2 +𝑎2
−1 −1 𝑥
(v) sec 𝑥 + 𝑐𝑜𝑠𝑒𝑐
√𝑥 2 −1
2 −2 1 2
(vi) (a) (b) (c) (d)
√𝑥 2 −1 √𝑥 2 −1 |𝑥|√𝑥 2 −1 |𝑥|√𝑥 2 −1
Q5: Read the following passage and answer the questions given below.

A potter made a mud vessel, where the shape of the pot is based on f(x) = |x-3|
+ |x-2|, where f(x) represents height of the pot

i. When x>4 what will be the height in terms of x ?


a. x-2 b. x-3 c. 2x-5 d. 5-2x

ii. What is dy/dx at x=3 ?


a. 2 b. -2 c. 1 d. Function is not differentiable
iii. If the potter is trying to make a pot using the function f(x)= [x] will
he get a pot or not ?
a. Yes, because it is a continuous function
b. Yes, because it is not a continuous function
c. No, because it is a continuous function
d. No, because it is not a continuous function

iv. Will the slope vary with x value.


a. Yes b. No c. Can’t say d. incomplete data

v. when the x value lies between (2,3) then the function is


a. 5 b. 1 c. 2x-5 d. 5-2x
Ans. Key:- Q.1. (i) b, (ii) b, (iii) c, (iv) d, (v) c
Q.2. (i) a, (ii) b, (iii) c, (iv) a, (v) a
Q.3. (i) a, (ii) c, (iii) a, (iv) c, (v) d
Q.4. (i) a, (ii) a, (iii) a, (iv) b, (v) d
Q.5. (i) c, (ii) c, (iii) d, (iv) a, (v) b
CASE STUDY QUESTIONS 2023-24
V S MESHRAM KV VSN NAGPUR
1
1) A company produces x units of output at a total cost c=3x3-18x2+160x.average cost is the
cost per unit and the rate of change is the change with respect to x from the above
information find
𝑐 1
a) Average cost ( Ans: 𝑥 =3x2-18x+160 )

b) Marginal cost ( Ans : x2-36x+160 )

c) Find the output when average cost equals to marginal cost (x=27)

d) Find the output when marginal cost is minimum (x=18)

e) Find the output when average cost is minimum (x=27)

2) A telephone company in a town has500 subscribers on list and collect fixed charge of rs 300 per
Subscriber. The company proposes to increase the annual subscription and it is believe that every
Increase of Rs 1, one customer discontinues the service then find
a) If annual subscription increased by x find the revenue of the company
(Ans: (500-x)(300+x)
500
b) Find average revenue (Ans: ( 𝑥
− 1)(300+x)

c) Find marginal revenue (Ans: 2(100-x))

d) What is the subscription when revenue is maximum? (Ans: x=100)

e) Find maximum annual revenue (Ans: 160 000)

3) A toy manufacturer wants to cut 28 metres long wire in to two pieces .one of the two pieces is to
bend in the form of square of perimeter 4x and a circle of radius y find

a) Relation between x and y (Ans:2x +πy=14)


1
b) Find combined area (Ans: 𝜋 {(π+4)x2-56x+196})

c) show combined area is max (Ans: show d2A/dx2 is negative)


28
d) At what value of x area will be maximum (Ans: x=𝜋+4 )
196
e) Find area (Ans: 𝜋+4
)

4) An open tank with rectangular base and vertical sides to be constructed so as to hold water
8000 litres of water. The tank is 2meter deep and material cost is Rs 1800/m2 and for vertical faces
900 Rs/m2 find
a) If x and y are length and breadth. find cost of base (Ans: 180xy)
b) Find cost of construction of four walls (Ans: 3600(x+y) )
c) Find total cost in terms of x (Ans: 3600x8/x3 )
d) Show that cost is minimum (Ans: show d2C/dx2 is positive)
e) Find cost (Ans: 21600)
5) Engine displacement is the measure of the cylinder swept by all pistons .The piston moves
inside the cylinder base .The cylinder is the form of open cylinder made from the sheet of
75𝜋cm2 find
𝜋
a) Find the volume of cylinder (Ans: (75r-r3) )
2
3𝜋
b) Find dv/dr (Ans: 2
(25-r2)
c) Show that volume is maximum (Ans: show d2v/dr2 is negative)
d) h>r is true or false (Ans: false)
OBJECTIVE QUESTIONS (2023-24)
(V S MESHRAM)

VSN NAGPUR

Q1) If y=(logx)2 then find dy/dx


a) 2 logx b)2/logx c)1/x d) 2log x(1/x)

Q2) If A is invertible matrix of order 3 and |A| =4 find |adjA|


a)3 b)4 c)16 d) 12
Q3) if y=tan-1√𝑥 find dy/dx
1 1 1 1
a) √𝑥 b) 1 c) d)
2√𝑥 √1−𝑥 2√𝑥 1+𝑥

𝑥+𝑦 2 6 2
Q4)If [ ] = [ ] , then the value of x, y,z
5+𝑧 𝑥𝑦 5 8
a)x=4,y=2,z=0 b) x=3,y=2,z=0 c) x=5,y=2,z=0 d) x=4,y=3,z=0
Q5)The number of all possible matrices of order 3x3 with each entry 0 and 1 is
a)27 b)18 c)81 d)512
Q6)If A is square matrix such that A2=A then (I+A)3-7A is
a) 0 b)1 c)1/A d) A2

Q7) If A is invertible matrix of order 3 and |A| =5 find |adjA|


a)3 b)5 c)15 d) 25
4 𝑥+2
Q8) If [ ] is symmetric matrix then x is
2𝑥 − 3 𝑥+1
a) 0 b) 3 c)2 d) 5
𝑥−𝑦 2 2 2
Q9)If [ ] = [ ] , then the value of y
𝑥 5 3 5
a)1 b)3 c)2 d)5
Q10)Let A be a square matrix of order 3x3, then |kA| is equal to
a)k|A| b)k2|A| c)3k|A| d) k3|A|
Q11)Find the rate of change of the area of circle with respect to its radius r when r=3
a)3π b) 18𝜋 c) π d)6π

Q12)The radius of sphere increase at the rate 0.2 cm/sec find the rate of change of volume at
r= 15 cm
a) 12π b) 180π c) 3π d)225π

Q13)The sides of an equilateral triangle are increasing at the rate of 2cm/sec, find the rate at which
the area increases ,when side is 10 cm
a)10 b)10√2 c)2 d)10√3

Q14) Find the point of inflection for the function f  x   x3 .


a)(0,0) b) (0,1) c) (1,0) d)(1,-1)
Continuity and Differentiability
MCQ
Choose correct answer from the given alternatives and write it. [Each carries 1 Mark]
13.

14.
20.

ANSWERS

1. (D) 2. (C) 3. (B) 4. (A) 5. (A) 6. (A) 7. (B) 8. (A) 9. (A) 10. (B)

11. (B) 12. (C) 13. (B ) 14. (B) 15. (C) 16. (A) 17. (B) 18. (C) 19. (B) 20. (B)
Case – based / Data – based Questions

1. An architect designs a building whose lift (elevator) is from outside of the building attached to
the walls. The floor (base) of the lift (elevator) is in semicircular shape .

The floor of the elevator (lift) whose circular edge is given by the equation 𝑥 2 + 𝑦 2 = 4 and the
straight edge (line) is given by the equation y = 0

(i) Find the point of intersection of the circular edge and straight line edge.
(ii) Find the length of each vertical strip of the region bounded by the given curves.
(iii) (a) Find the area of a vertical strip between given circular edge and straight edge .
(b) Find the area of a horizontal strip between given circular strip and straight
OR
(iv) Find the area of the region of the floor of the lift of the building (in square units)
2. A student designs an open air Honey nest on the branch of a tree , whose plane figure is
parabolic and the branch of tree is given by a straight line

(i) Find point of intersection of the parabola and straight line


(ii) Find the area of each vertical strip.
(iii)(a)Find the length of each horizontal strip of the bounded region.
(b) Find the length of each vertical strip.
OR
(iv) Find the area of region bounded by parabola 𝑥 2 = 4y and line y = 4 (in square units)
3. Using integration ,find the area of region bounded by line y = √3 𝓍 , the curve y = √4 − 𝑥 2 and
y- axis in first quadrant.
4. Find the area of the region using integration :
{ (x,y) : 𝑦 2 ≤ 2x and y ≥ x -4 }
5. A farmer has a triangular shaped field . His son, a science student observes the triangular field
has three edges and can be drawn on a plain paper with three lines given by its equatins.
(i) Find the area of the shaded region in the figure shown below.

(ii) Find the area of the triangle ABC given below.


ConCEPT
MCQ’S (INTEGRALS)
𝑪𝒐𝒔 √𝒙
1.The value of ∫ 𝒅𝒙 is
√𝒙

𝐜𝐨𝐬 𝒙
(a) 2 cos √𝒙 + C (b) √ +𝑪 (c) sin √𝒙 + C (d) 2 sin √𝒙 + C
𝒙

𝒙+𝟑
2. ∫ 𝒆𝒙 𝒅𝒙 =
(𝒙+𝟒)𝟐

𝒆𝒙 𝒆𝒙
(a) +C (b) +C
𝒙+𝟒 𝒙+𝟑
𝟏 𝒆𝒙
(c) +C (d) +C
(𝒙+𝟒)𝟐 (𝒙+𝟒)𝟐

𝑪𝒐𝒔 𝟐𝒙−𝑪𝒐𝒔 𝟐𝜽
3. ∫ 𝒅𝒙 is equal to
𝐂𝐨𝐬 𝒙−𝑪𝒐𝒔𝜽

(a) 2 (Sin x + x Cos 𝜽) + C (b) 2 (Sin x - x Cos 𝜽) + C


(c) 2 (Sin x +2 x Cos 𝜽) + C (d) 2 (Sin x - 2 x Cos 𝜽) + C
𝟑
4. ∫ 𝒙𝟐 𝒆𝒙 𝒅𝒙 equals
𝟏 𝟑 𝟏 𝟐
(a) 𝒆𝒙 + 𝑪 (b) 𝒆𝒙 + 𝑪
𝟑 𝟑
𝟏 𝟑 𝟏 𝟐
(c) 𝒆𝒙 + 𝑪 (d) 𝒆𝒙 + 𝑪
𝟐 𝟐

5. ∫ √𝒂𝟐 − 𝒙𝟐 dx
𝟏 𝒂+𝒙 𝒙
(a) 𝐥𝐧 | | +C (b) 𝑺𝒊𝒏 −𝟏 +C
𝟐𝒂 𝒂−𝒙 𝒂
𝟏 𝟏 𝒙 𝟏 𝟏
(c) 𝒙 √𝒂𝟐 − 𝒙𝟐 + 𝒂𝟐 𝑺𝒊𝒏 −𝟏 (d) 𝒙 √𝒂𝟐 − 𝒙𝟐 + 𝒂𝟐 ln
𝟐 𝟐 𝒂 𝟐 𝟐
|𝒙+ √𝒙𝟐 − 𝒂𝟐 | + C
𝒆 𝐥𝐧 𝒙
6. ∫𝟏 𝒅𝒙 is equal to
𝒙

𝟏 𝒆𝟐
(a) (b) (c) 1 (d) ∞
𝟐 𝟐
𝟏
7. ∫𝟎 𝐭𝐚𝐧(𝑺𝒊𝒏−𝟏 𝒙) 𝒅𝒙
(a) 2 (b) 0 (c) -1 (d) 1
𝝅
(𝑺𝒊𝒏 𝒙+𝑪𝒐𝒔 𝒙)𝟐
8 . The value of I = ∫𝟎𝟐 𝒅𝒙 , is
√𝟏+𝑺𝒊𝒏 𝟐𝒙

(a) 3 (b) 1 (c) 2 (d) 0


𝟐𝒂
9. ∫𝟎 𝒇(𝒙) 𝒅𝒙 is equal to
𝒂
(a) 2 ∫𝟎 𝒇(𝒙) 𝒅𝒙 (b) 0
𝒂 𝒂 𝒂 𝟐𝒂
(c) ∫𝟎 𝒇(𝒙) + ∫𝟎 𝒇 (𝟐𝒂 − 𝒙) 𝒅𝒙 (d) ∫𝟎 𝒇(𝒙) + ∫𝟎 𝒇(𝒙)
𝟏
𝟏 (𝒙− 𝒙𝟑 )𝟑
10. The value of the integral ∫𝟏 dx is
𝟑
𝒙𝟒

(a) 6 (b) 0 (c) 3 (d) 4


DEFINITE INTEGRSLS (MCQ)

0∫
2
1. x2 dx =

a. 2
b. ⅔
c. 8/3
d. None of these
Answer: (c) 8/3

0∫
2
2. (x2 + 3)dx equals

a. 24/3
b. 25/3
c. 26/3
d. None of the above.
Answer: (c) 26/3

1∫
2
3. dx/x2 equals

a. 1
b. -1
c. 2
d. ½
Answer: (d) ½

π
4. 0∫ sin2 x dx =

a. π/2
b. π/4
c. 2π
d. 4π
Answer: (a) π/2

0∫
4
5. 3x dx equals

a. 12
b. 24
c. 48
d. 86
Answer: (b) 24
6. Integrate 0∫ 2 (x2+x+1) dx

a. 15/2
b. 20/5
c. 20/3
d. 3/20
Answer: (c) 20/3

7. If f(a + b + 1 – x) = f(x) ∀ x, where a and b are fixed positive real numbers, then the below
expression is equal to

SOLUTION

ANS: C
𝑏 𝐼𝑥𝐼
8. Let l= ∫𝑎 𝑑𝑥, 𝑎 < 𝑏
𝑥

What is l equals to if 𝑎 < 𝑏 < 0


a) a+b b) a-b
𝑎+𝑏
c) b-a d) 2

solution: a-b
𝑏 𝐼𝑥𝐼
9. Let l= ∫𝑎 𝑑𝑥, 𝑎 < 𝑏
𝑥
What is l , if a< 0 < 𝑏
a) a+b b) a-b
𝑎+𝑏
c) b-a d) 2

solution : a+b
10.

then value of a is equal to


(a) 3
(b) 6
(c) 9
(d) 1
Solution: c

11.

SOLUTION: a

12.

Solution: c
13.

(a) I1 > I2
(b) I2 > I1
(c) I1 = I2
(d) I1 > 2I2
Solution: b

PREPARED BY
AISWARYA S (PGT M ATHEMATICS, KVRHE)
INDEFINITE INTEGRALS
MCQ’s
1) If (d/dx) f(x) is g(x), then the antiderivative of g(x) is
a) f(x)
b) f’(x)
c) g’(x)
d) None of the above
Answer: (a) f(x)
Given: (d/dx) f(x) = g(x)
We know that the integration is the inverse process of differentiation, then the antiderivative of
g(x) is f(x).
Hence, option (a) f(x) is the correct answer.

2) ∫cot²x dx equals to
a) cot x – x + C
b) -cot x – x + C
c) cot x + x + C
d) -cot x + x + C
Answer: (b) -cot x – x + C
Explanation:
We know that cot2 x = cosec2x – 1
∫cot²x dx = ∫ (cosec2x – 1) dx = -cot x -x + C. [Since, ∫cosec2x dx = -cot x + c]
Hence, the correct answer is option (b) -cot x – x + C.

3) If ∫ sec²(7 – 4x)dx = a tan (7 – 4x) + C, then value of a is


a. -4
b. -¼
c. 3
d. 7
Answer: (b) -¼
Explanation:
Given: ∫ sec²(7 – 4x)dx = a tan (7 – 4x) + C
∫ sec²(7 – 4x)dx ={[tan (7-4x)]/-4} + C
∫ sec²(7 – 4x)dx = (-¼) tan (7-4x) + C
Hence, the value of a is -¼.
Answer: (a)

Explanation:

5.

(a) log |sin x + cos x|


(b) x
(c) log |x|
(d) -x

Answer: d
Explaination:
6. If ∫ sec²(7 – 4x)dx = a tan (7 – 4x) + C, then value of a is
(a) 7
(b) -4
(c) 3
(d) −14
Answer: d
Explaination:
(d), ∫sec²(7 – 4x)dx = tan(7−4x)−4 + C = –14 tan (7 – 4x) + C.

7.Given ∫ 2x dx = f(x) + C, then f(x) is

Explaination:

8.
a) 2(sin x + x cos θ) + C
(b) 2(sin x – x cos θ) + C
(c) 2(sin x + 2x cos θ) + C
(d) 2(sin x – 2x cos θ) + C
Answer : (a)

Explaination:
9.

(a) log |sin x + cos x|


(b) x
(c) log |x|
(d) –x
Answer: d
Explaination:

10.

Answer: c
Explaination:

Prepared by
Mrs. Suchita Ahirrao(PGT CME Pune)
Mrs. Kavya Patil (PGT DIAT GIRINAGAR)
TOPIC – DIFFERENTIAL EQUATIONS CLASS – XII
MCQ TYPE QUESTIONS:

Q1. Solution of differential equation x.dy – y.dx = Q represents:


A. a rectangular hyperbola
B. parabola whose vertex is at the origin
C. straight line passing through the origin
D. a circle whose centre is at the origin
𝑑𝑦
Q6: The integrating factor of the differential equation 𝑥 − 𝑦 = 2𝑥 2 𝑖𝑠:
𝑑𝑥
−𝑥
a)𝑒
−𝑦
b) 𝑒
1
c)𝑥

d)x
𝑑2 𝑥
Q7: Consider the differential equation + 4 𝑑𝑥 = 4𝑦 = 0 . What is the general
𝑑𝑦
𝑑𝑦 2
solution of this second-order linear homogeneous differential equation?

a) y = 𝐶1 𝑒 2𝑥 + 𝐶2 𝑒 −2𝑥

b) y = 𝐶1 𝑒 2𝑥 + 𝐶2 𝑥

c) y = 𝐶1 𝑒 −2𝑥 + 𝐶2 𝑥

d) y = 𝐶1 𝑒 −2𝑥 + 𝐶2 𝑒 2𝑥

𝑑𝑦
Q8: The solution of the differential equation = 3𝑥 2 is:
𝑑𝑥

a) y = 𝑥 3 + C

b) y = 𝑥 3 + 3

c) y = 3𝑥 3 + C

d) y = 3𝑥 3
𝑑𝑦
Q9: The general solution of the differential equation + 2𝑦 = 4 is:
𝑑𝑥

a) y = 2x + C

b) y = 2x - 2

c) y = 2𝑥 2 + C

d) y = 2𝑥 2 – 2
𝑑2 𝑦 𝑑𝑦
Q10: The particular solution of the differential equation − = 2𝑥 with initial
𝑑𝑥 2 𝑑𝑥
conditions y (0) = 1 and y'(0) = 0 is:

a) y = 𝑥 2 + 1

b) y = 𝑥 2 + x + 1

c) y = 𝑥 2 + x d) y = 𝑥 2
𝑑𝑦
Q11: The solution of the differential equation + 2𝑦 = 𝑒 −2𝑥 is:
𝑑𝑥

𝑒 −2𝑥
a) y = +C
2

𝑒 −2𝑥
b) y = -C
2

𝑒 −2𝑥
c) y = +1
2

𝑒 −2𝑥
d) y =
2

𝑑2 𝑦
Q12: The general solution of the differential equation 𝑑𝑥 2 − 9𝑦 = 0 is:

a) y = 𝐶1 𝑒 3𝑥 + 𝐶2 𝑒 −3𝑥

b) y = 𝐶1 sin(3x) + 𝐶2 cos(3x)

c) y = 𝐶1 𝑒 3𝑥 + 𝐶2 sin(3x)

d) y = 𝐶1 sin(3x) + 𝐶2 e^(-3x)
𝑑𝑦
Q13: The particular solution of the differential equation = 2𝑥 + 3 with initial condition
𝑑𝑥
y(1) = 5 is:

a) y = 𝑥 2 + 3x + 2

b) y = 𝑥 2 + 3x + 4

c) y = 𝑥 2 + 2x + 4

d) y = 𝑥 2 + 2x + 2
𝑑𝑦
Q14: The solution of the differential equation + 𝑦 = 𝑥 2 + 1 is:
𝑑𝑥

𝑥2
a) y = 2
+x+C

𝑥2
b) y = 2
- x+C

𝑥2
c) y = 2
+x

𝑥2
d) y = 2
–x

𝑑2 𝑦
Q15: The general solution of the differential equation + 4𝑦 = 0 is:
𝑑𝑥 2

a) y = 𝐶1 𝑒 2𝑥 +𝐶2 sin(2x)

b) y = 𝐶1 𝑒 2𝑥 + 𝐶2 cos(2x)

c) y = 𝐶1 𝑒 2𝑥 + 𝐶2 𝑒 −2𝑥

d) y = 𝐶1 sin(2x) + 𝐶2 cos(2x)
VALUE BASED QUESTIONS:

Q1: It is known that, if the interest is compounded continuously, the principal changes at the
rate equal to the produd' of the rate of bank interest per annum and the principal. Let P
denotes the principal at any time t and rate of interest be r % per annum.

Based on the above information, answer the following questions.


(i) If the interest is compounded continuously at 5% per annum, in how many years will
Rs.100 double itself?
(a) 12.728 years (b) 14.789 years (c) 13.862 years (d) 15.872 years
(ii) At what interest rate will Rs.100 double itself in 10 years? (loge2 = 0.6931).
(a) 9.66% (b) 8.239% (c) 7.341% (d) 6.931%
(iii) How much will Rs.1000 be worth at 5% interest after 10 years? (e0.5 = 1.648).
(a) Rs.1648 (b) Rs 1500 (c) Rs 1664 (d) Rs 1572

Q2: The rate of increase in the number of bacteria in a certain bacteria culture is
proportional to the number present. Given that the number triples in 5 hours.

(i) The value of: ∫1Kxdx=

(A)
(B)
(C)
(D)

(ii) The general solution is ____________.


(A)

(B)
(C)
(D)

(iii) If N0 is the initial count of bacteria, after 10 hours the count is ________.

(A)
(B)
(C)
(D)

Q3: Radium is a radioactive compound. Radioactivity is a property of some elements in which


the decomposes continuously in modern time the carbon dating of fossils and old things are
used to determine their life. Radium decomposes at the rate proportional to the quantity of
radium present. It is found that in 25 years approximately 1.1% of a certain quantity radium
has decomposed.

(i) Determine how long it will take for one half of the original amount.

(a) 1567 yrs

(b) 1528 yrs

(c) 50 yrs

(d) 100 yrs

(ii) Determine how long it will take for one fourth of the original?

(a) 1567 yrs

(b) 3133 yrs

(c) 784 yrs

(d) 25 yrs

Q4: Read the passage given below and answer the following questions: The rate of change of
temperature of a body is proportional to the difference between the temperature of the
body itself and that of the surroundings. This Law is known as Newton’s Law of Cooling. Let S
be the constant temperature of surroundings. Let T be the temperature of the body at any
time t.
(i)The temperature of a body in a room is 800 F. After five minutes the temperature of the
body becomes 600 F. After another 5 minutes the temperature becomes 500 F. What is the
temperature of surroundings?
(A) 600 F
(B) 200 F
(C) 800 F
(D) 400 F

(ii) Which of the following is a differential equation?


(A) x2 – 3x + 3 = 0
(B) sin x + cosx= 0
(C) x + y = 7
(D) dy/ dx = e^x
Q5: Polio drops are delivered to 50K children in a district. The rate at which polio drops are
given is directly proportional to the number of children who have not been administered the
drops. By the end of 2nd week half the children have been given the polio drops. How many
will have been given the drops by the end of 3rd week can be estimated using the solution
to the differential equation dy/dx = (𝟓𝟎 − 𝐲) where x denotes the number of weeks and y the
number of children who have been given the drops.

(i) Which method of solving a differential equation can be used to solve dy/dx = (𝟓𝟎 − 𝐲).?
(a). Variable separable method
(b). Solving Homogeneous differential equation
(c). Solving Linear differential equation
(d). all of the above
(ii) The solution of the differential equation dy/dx = 𝐤(𝟓𝟎 − 𝐲) is given by,
(a). log | 50 – y| = kx + C
(b). - log | 50 – y| = kx + C
(c). log | 50 – y| = log| kx |+ C
(d). 50 – y = kx + C

Answers: 1: (c) 2: (a) 3: (a) 4: (c) 5: (d) 6: (c) 7: (a) 8: (a) 9: (a) 10: (c) 11: (a)
12: (a) 13: (c) 14: (a) 15: (b)
DIFFERENTIAL EQUATIONS
MCQ , ASSERTION AND REASONING, CASE BASED QUESTIONS.

Choose and write the correct option in the following questions:


2
𝑑𝑦 3 𝑑2𝑦
1) The degree of the differential equation (1 + 𝑑𝑥 ) = (𝑑𝑥 2 )
a) 1 b) 2 c) 3 d) 4
𝑑4 𝑦 𝑑𝑦 4
2) The order and degree of the differential equation = 𝑦 + ( ) are respectively
𝑑𝑥 4 𝑑𝑥
a) 4,1 b) 4,2 c) 2,2 d) 2,4
𝑑𝑦
3) The integrating factor of the differential equation x𝑑𝑥 – y = 2 x2
a) e-x b) e-y c) 1/x d) x
2 2
𝑑2 𝑦 𝑑𝑦
4) find the product of the order and degree x( ) + ( ) + y2 = 0
𝑑𝑥 2 𝑑𝑥
a) 1 b) 2 c) 3 d) 4
𝑑𝑦
5) Integrating factor of (𝑥 𝑙𝑜𝑔𝑥) + 𝑦 = 2 𝑙𝑜𝑔𝑥.
𝑑𝑥
a) x b) 2x c) logx d) 2logx
𝑑𝑦
6) The general solution of differential equation (𝑒 )𝑑𝑥 = x2
a) y = 2(xlogx -x) + c b) y = 2(xlogx -2x) + c c) y = 2(2xlogx -x) + c d) y = 2(xlogx +x) + c
4
𝑑2𝑦 𝑑𝑦
7) Order and degree of the differential equation ( ) + cos = 0
𝑑𝑥 2 𝑑𝑥
a) O= 1, D=1 b) O=2 , D=1 c) O = 2, D = not defined d) O=2, D=4
1
1
𝑑2 𝑦 𝑑𝑦 3
8) Assertion: Order of the differential equation 𝑑𝑥 2 + (𝑑𝑥 ) +𝑥 4 =0
Reason: The order of a differential equation is the order of highest derivative occurring in the
differential equation.
(a) Both assertion and reason are true and reason is the correct explanation of assertion.
(b) Both are true but reason is not the correct explanation of assertion.
(c) Assertion is true reason is false.
(d) Assertion is false, reason is true.

9) Assertion: Differential equation dy /dx = xy + y / xy + x is homogeneous equation.

Reason: A function(x,y) is called homogeneous of degree n if f(kx,ky)=kn f(x,y).

(a) Both assertion and reason are true and reason is the correct explanation of assertion.

(b) Both are true but reason is not the correct explanation of assertion.

(c) Assertion is true reason is false.

(d) Assertion is false, reason is true


10) Assertion: Integrating factor of (dy/dx) – y tan x = - y2 sec x is cos x

Reason: Integrating factor I.F. = 𝑒 ∫ 𝑃 𝑑𝑥. for linear differential equation (dy/dx) + Py = Q.

(a) Both assertion and reason are true and reason is the correct explanation of assertion.

(b) Both are true but reason is not the correct explanation of assertion.

(c) Assertion is true reason is false.

(d) Assertion is false, reason is true.

11) Assertion : The number of arbitrary constants in theParticular solution of a differential equation of
third order is 3

Reason : To solve the Differential equation of third order we have to integrate thrice

(A) Both Assertion and Reason is true and Reason is correct explanation of the Assertion

(B) Both Assertion and Reason is true But Reason is not correct explanation of the Assertion

(C) Assertion is true ,Reason is false

(D) Assertion is false, Reason is true

12) Assertion: y = a sin x + b cos x is a general solution of y '' + y = 0

Reason: y = a sin x + b cos x is a trigonometric function.

(A) Both Assertion and reason are true and reason is correct explanation of assertion.

(B) Assertion and reason both are true but reason is not the correct explanation of assertion.

(C) Assertion is true, reason is false.

(D) Assertion is false, reason is true.

CASE BASED QUESTIONS

1) A Veterinary doctor was examining a sick cat brought by a pet lover. When it was brought to
the hospital, it was already dead. The pet lover wanted to find its time of death. He took the
temperature of the cat at 11.30 pm which was 94.6 F. He took the temperature again after one
hour; the temperature was lower than the first observation. It was 93.4 F. The room in which
the cat was put is always at 70 F. The normal temperature of the cat is taken as 98.6 F when it
was alive. The doctor estimated the time of death using Newton law of cooling which is
governed by the differential equation: dT/ dt ∝ (𝑇 − 70), where 70 F is the room temperature
and T is the temperature of the object at time t. Substituting the two different observations of T
and t made, in the solution of the differential equation dT/ dt = k (𝑇 − 70) where k is a constant
of proportion, time of death is calculated.
(i) Which method of solving a differential equation helped in calculation of the time of
death?

a. Variable separable method


b. Solving Homogeneous differential equation
c. Solving Linear differential equation
d. all of the above
(ii) The solution of the differential equation dT/ dt = (𝑇 − 70)is given by,
a. log | T – 70| = kt + C
b. log | T – 70| = log |kt |+ C
c. T – 70 = kt + C
d. T – 70 = kt C
Q2) Polio drops are delivered to 50K children in a district. The rate at which polio drops are
given is directly proportional to the number of children who have not been administered the
drops. By the end of 2nd week half the children have been given the polio drops. How many will
have been given the drops by the end of 3rd week can be estimated using the solution to the
differential equation dy/ dx= (𝟓𝟎 − 𝐲) where x denotes the number of weeks and y the number
of children who have been given the drops. 25
i) Which method of solving a differential equation can be used to solve dy dx = (𝟓𝟎 − 𝐲).?
a. Variable separable method

b. Solving Homogeneous differential equation

c. Solving Linear differential equation


d. all of the above

ii) The solution of the differential equation dy/ dx = (𝟓𝟎 − 𝐲) is given by,

a. log | 50 – y| = kx + C

b. - log | 50 – y| = kx + C

c. log | 50 – y| = log| kx |+ C

d. 50 – y = kx + C

iii) The value of c in the particular solution given that y(0)=0 and k = 0.049 is.

a. log 50

b. log 1 50

c. 50

d. -50
MCQ DIFFERENTIAL EQUATIONS

Mr AVINASH NAGDEOTE

KV AMBAJHARI NAGPUR

Q.1: What is the order of differential equation y’’ + 5y’ + 6 = 0?

A. 0

B. 1

C. 2

D. 3

Answer: C. 2

Explanation: Given, differential equation y’’ + 5y’ + 6 = 0.

The highest order derivative present in the differential equation is y’’. Hence, the
order is 2.

Q.2: What is the degree of differential equation (y’’’)2 + (y’’)3 + (y’)4 + y5 = 0?

A. 2

B. 3

C. 4

D. 5

Answer: A. 2

Explanation: The degree is the power raised to the highest order derivative. Therefore,
in the given differential equation, (y’’’)2 + (y’’)3 + (y’)4 + y5 = 0, the degree will be
power raised to y’’’.

Q.3: The number of arbitrary constants in the particular solution of a


differential equation of third order is:

A. 3
B. 2

C. 1

D. 0

Answer: D. 0

Explanation: The solution free from arbitrary constants i.e., the solution obtained from
the general solution by giving particular values to the arbitrary constants is called a
particular solution of the differential equation.

Q.4: What is the differential equation of the family of circles touching the y-axis
at the origin?

A. 2xyy’ + x2 = y2

B. 2xyy’’ + x’ = y2

C. 2xyy’ – x2 = y2

D. xyy’ + x2 = y2

Answer: A. 2xyy’ + x2 = y2

Explanation: Let the center of the circle touch the y- axis at origin lies on the x-axis.

Say, (k, 0) be the center of the circle.

Hence, it touches the y – axis at origin, its radius is p.

Now, the equation of the circle with center (p, 0) and radius (p) is

⇒ (x – k)2 + y2 = k2

⇒ x2 + k2 – 2xk + y2 = p2

Shifting k and – 2xk to RHS then it becomes: k2 and 2xk

⇒ x2 + y2 = k2 – k2 + 2kx

⇒ x2 + y2 = 2kx ….(i)

Differentiating equation on both sides, we have,


⇒ 2x + 2yy’ = 2k

⇒ x + yy’ = k

Now, on substituting the value of ‘k’ in the equation (i), we get,

⇒ x2 + y2 = 2(x + yy’)x

⇒ 2xyy’ + x2 = y2

Therefore, 2xyy’ + x2 = y2 is the required differential equation.

Q.5: Solution of differential equation x.dy – y.dx = Q represents:

A. a rectangular hyperbola

B. parabola whose vertex is at the origin

C. straight line passing through the origin

D. a circle whose centre is at the origin

Answer: C. straight line passing through the origin

Q.6: Which of the following is a second-order differential equation?

A. (y’)² + x = y²

B. y’y” + y = sin x

C. y”’ + (y”)² + y = 0

D. y’ = y²

Answer: B. y’y” + y = sin x

Q7)Explanation: The order of y’y” + y = sin x is 2. Thus, it is a second-order


differential equation.

Q8). What is the order of the differential equation given by dy/dx+4y=sinx?


a) 0.5
b) 1
c) 2
d) 0
Answer: b
Explanation: Since the order of a differential equation is defined as the order of the highest
derivative occurring in the differential equation, i.e for nth derivative dnydxn if n=1.
It has order 1→ differential equation contains only dydx derivative with variables and
constants.

2. Given the differential equation dy/dx=x4−y4(x2+y2)xy the degree of differential equation is


_____________
a) 1
b) 4
c) 0
d) 2

Answer: a
Explanation: The degree of a differential equation is the degree of the highest order
derivative when differential coefficients are free from radicals and fraction above
differential i.e having first order is free from radical and a fraction has a power of 1 thus it
has a degree of 1.

9). The process of formation of the differential equation is given in the wrong order, select the
correct option from below given options.
1) Eliminate the arbitrary constants.
2) Differential equation which involves x,y,dy/dx.
3) Differentiating the given equation w.r.t x as many times as the number of arbitrary
constants.
a) 1,2,3
b) 3,2,1
c) 3,1,2
d) 2,1,3

Answer: c
Explanation: The correct order of forming differential equation is given by option 3,1,2,
even the given differential equation can be solved by other order given in the option but the
task becomes more tedious.

10) A racer accelerates from a stop so that its speed is 10t m/s t second after starting how far
will the car go in 4 seconds?
a) 80m
b) 60m
c) 40m
d) 160m
KENDRIYA VIDYALAYA SANGATHAN
MUMBAI REGION
VECTOR ALGEBRA
MULTIPLE CHOICE QUESTIONS
1. If 𝑎⃗⃗⃗ is a nonzero vector of magnitude ‘a’ and λ a nonzero scalar, then
λ𝑎
⃗⃗⃗ is unit vector if
(A) λ = 1 (B )λ = – 1 (C) a = |λ| (D) a = 1/|λ|
2. Let the vectors 𝑎⃗⃗⃗ and ⃗⃗⃗⃗ ⃗⃗⃗ | = √2
𝑏 be such that |𝑎 ⃗⃗⃗ | = 3 𝑎𝑛𝑑 |𝑏 ,
3
then 𝑎 ⃗⃗⃗ × ⃗⃗⃗⃗
𝑏 is a unit vector, if the angle between 𝑎 ⃗⃗⃗ and ⃗⃗⃗⃗
𝑏 is
(A)π/6 (B)π/4 (C)π/3 (D) π/2

3. Area of a rectangle having vertices A, B, C and D with position vectors

,
respectively is
1 (B)1 (C)2 (D)4
(A)
2

4. If θ is the angle between two vectors 𝑎⃗⃗⃗ and ⃗⃗⃗⃗


𝑏 , then 𝑎 . 𝑏⃗ ≥ 0
only when
𝜋 𝜋
(A) 0 < 𝜃 < (B) 0≤ 𝜃 ≤ (C) 0 < 𝜃 < 𝜋 (D) 0 ≤ 𝜃 ≤ 𝜋
2 2

5. Let 𝑎⃗⃗⃗ and ⃗⃗⃗⃗


𝑏 be two unit vectors and θ is the angle between them.
Then 𝑎 + 𝑏⃗ is a unit vector if
𝜋 𝜋 𝜋 2𝜋
(A) 𝜃 = (B) 𝜃 = (C) 𝜃 = (D) 𝜃 =
4 3 2 3

6.
(A) 0 (B) – 1 (C) 1 (D) 3

7. If θ is the angle between any two vectors 𝑎⃗⃗⃗ and ⃗⃗⃗⃗


𝑏 , then |𝑎 . 𝑏⃗ | =
⃗⃗⃗ × ⃗⃗⃗⃗
|𝑎 𝑏 | when θ is equal to
(A) 0 (B)
𝜋 𝜋 (𝐷) 𝜋
4 (𝐶)
2
8. If |𝑎⃗⃗⃗ | = 10 , |𝑏⃗⃗⃗ | = 2 and 𝑎 . 𝑏⃗ = 12 , then value of ⃗⃗⃗ × ⃗⃗⃗⃗
|𝑎 𝑏 | is :
(A) 5 (B) 10 (𝐶) 14 (𝐷) 16

Compiled by KRISHNAMURTHY AGGU , PGT (MATHS) - KV BSF CHAKUR


9. The angle between vectors 𝑎⃗⃗⃗ and ⃗⃗⃗⃗
𝑏 with magnitudes √3 and 4
respectively and 𝑎 . 𝑏⃗ = 2√3 is :
𝜋 𝜋 𝜋 5𝜋
(A) (B) (𝐶)
6 3
2 (𝐷)
2
̂
10. The area of the parallelogram whose adjacent sides are 𝑖̂ + 𝑘 and
2𝑖̂ + 𝑗̂ + 𝑘̂ is
(A) 3 (B) √2 (C) 4 (D) √3

ANSWERS OF MCQ
1. (D) a = 1/|λ|
2. (B)π/4
3. (C)2
𝜋
4 (B) 0≤ 𝜃 ≤
2

2𝜋
5 (D) 𝜃 =
3

6 (C)1

7
𝜋
(𝐵)
4
8. (𝐷) 16
9. 𝜋
(𝐵 )
3

10. (D) √3

Compiled by KRISHNAMURTHY AGGU , PGT (MATHS) - KV BSF CHAKUR


CASE - STUDY BASED QUESTIONS
[This section comprises of 3 case- study/passage based questions of 4
marks each with sub parts. The first two case study questions have three
sub parts (i), (ii), (iii) of marks 1,1,2 respectively. The third case study
question has two sub parts of 2 marks each.)
1. Read the following passage and answer the questions given below:
Teams A B C , , went for playing a tug of war game. Teams A B C, , have
attached a rope to a metal ring and is trying to pull the ring into their own
area.

Team A pulls with force F1 = 6 𝑖̂ + 0 𝑗̂ kN,


Team B pulls with force, F2 = ‒ 4 𝑖̂ + 4 𝑗̂ kN,
Team C pulls with force, F3 = ‒ 3 𝑖̂ ‒ 3 𝑗̂ kN,

i) What is the magnitude of the force of Team A ?

ii) Which team will win the game?

iii ) Find the magnitude of the resultant force exerted by the teams.
OR
In what direction is the ring getting pulled?

Compiled by KRISHNAMURTHY AGGU , PGT (MATHS) - KV BSF CHAKUR


2. Read the following passage and answer the questions given below:

Solar Panels have to be installed carefully so that the tilt of the roof, and
the direction to the sun, produce the largest possible electrical power in
the solar panels.
A surveyor uses his instrument to determine the coordinates of the four
corners of a roof where solar panels are to be mounted. In the picture ,
suppose the points are labelled counter clockwise from the roof corner
nearest to the camera in units of meters P1 (6,8,4) , P2 (21,8,4), P3
(21,16,10) and P4 (6,16,10)

(i) What are the components to the two edge vectors defined by 𝐴 =
PV of P2 – PV of P1 and 𝐵⃗ = PV of P4 – PV of P1? (where PV stands
for position vector)
(ii) What are the magnitudes of the vectors 𝐴 and 𝐵 ⃗ and in what
units ?
(iii) What are the components to the vector ⃗⃗⃗⃗
𝑁 , perpendicular to 𝐴 and
⃗ and the surface of the roof ?
𝐵

OR
What is the angle between vectors ⃗⃗⃗⃗
𝑁 and ⃗⃗⃗𝑆 ? What is the
elevation angle of the sun above the plane of the roof ?
(Cos 510 =0.629 )

Compiled by KRISHNAMURTHY AGGU , PGT (MATHS) - KV BSF CHAKUR


3. Read the following passage and answer the questions given below:
A class XII student appearing for a competitive examination was asked
to attempt the following questions.

Let 𝑎 , 𝑏⃗ and 𝑐⃗⃗⃗ 𝑏𝑒 𝑡ℎ𝑟𝑒𝑒 non zero vectors.

(i)

(ii) If 𝑎 and 𝑏⃗ are unit vectors and  be the angle between them then

⃗⃗⃗ ‒ 𝑏⃗ | is ?
|𝑎

(iii) What is the area of the parallelogram formed by 𝑎 and 𝑏⃗ as


diagonals ?
OR

4. Read the following passage and answer the questions given below:

Ritika starts walking from her house to shopping mall. Instead of going to
the mall directly , she first goes to a ATM , from there to her daughter`s
school and then reaches the mall . In the diagram, A, B , C and D
represent the coordinates of House , ATM , School and Mall respectively.

Compiled by KRISHNAMURTHY AGGU , PGT (MATHS) - KV BSF CHAKUR


(i) What is the distance between House (A) and ATM (B) ?
(ii) What is the distance between School (C) and Shopping mall (D) ?
(iii)What is the total distance travelled by Ritika ?
OR
What is the extra distance travelled by Ritika in reaching the
shopping mall ?
5. Read the following passage and answer the questions given below:
A barge is pulled into harbour by two tug boats as shown in the
figure :

(i) What is the Position vector of A ?


(ii) ⃗⃗⃗⃗⃗⃗ in terms of 𝑖̂ and 𝑗̂ .
Find the vector 𝐴𝐶
(iii) 𝐴 = 𝑖̂ + 2 𝑗̂ + 3𝑘̂ then what is its unit vector ?
If ⃗⃗⃗
OR
If ⃗⃗⃗
𝐴 = 4 𝑖̂ + 3 𝑗̂ and ⃗⃗⃗ 𝐵 = 3 𝑖̂ + 4 𝑗̂ , then what is the
value of |𝐴| + |𝐵⃗| ?

Compiled by KRISHNAMURTHY AGGU , PGT (MATHS) - KV BSF CHAKUR


KENDRIYA VIDYALAYA SANGATHAN- MUMBAI REGION

LINEAR PROGRAMMING PROBLEMS

1. Maximise Z = 3x + 4y subject to the constraints: x + y ≤ 4, x ≥ 0, y ≥ 0.

Solution: Objective function is Z = 3x + 4 y ……(1).


The given constraints are : x + y ≤ 4, x ≥ 0, y ≥ 0.
The corner points obtained by constructing the line x+ y= 4, are (0,0),(0,4) and
(4,0).

Corner points Z = 3x +4y

O ( 0 ,0 ) Z = 3(0)+4(0) = 0

A(4,0) Z = 3(4) + 4 (0) = 12

B(0,4) Z = 3(0) + 4 ( 4) = 16 …( Max. )


Therefore Z = 16 is maximum at (0 , 4).

2. Maximise Z = x + y subject to x + 4y ≤ 8, 2x + 3y ≤ 12, 3x + y ≤ 9, x ≥ 0, y ≥ 0.

Solution:

Here , Maximise Z = x + y subject to x + 4y ≤ 8, 2x + 3y ≤ 12, 3x + y ≤ 9, x ≥


0, y ≥ 0.

Corner points Z=x+y

P( 0 , 0 ) 0

Q(3 , 0) 3

R( 0, 2 ) 2

S(28/11 , 15/11 ) 43/11.(Max.)


Hence the maximum value is 43/11

3. Minimise Z = 13x – 15y, subject to the constraints: x+y≤7


,2x–3y+6≥0,x≥0,y≥0
Solution:
Considerx+y=7
When x=0, then y=7x=0, then y=7 and
when y=0, then x=7y=0, then x=7
So, A(0, 7) and B(7, 0) are the points on line
x+y=7

Consider 2x–3y+6=02x–3y+6=0
When x = 0, then y = 2 and when y = 0, then x = – 3, So C(0, 2) and D(-3, 0)
are the points on line 2x–3y+6=02x–3y+6=0
Also, we have x > 0 and v > 0.
The feasible region OBEC is bounded, so, minimum value will obtain at a
comer point of this feasible region.
Corner points are O(0, 0), B(7, 0), E(3, 4) and C(0, 2)
Z=13x–15y
At O(0,0),Z=0O(0,0),Z=0
At B(7,0),Z=13(7)–15(0)=91B(7,0),Z=13(7)–15(0)=91
At E(3,4),Z=13(3)–15(4)=−21E(3,4),Z=13(3)–15(4)=−21
At C(0,2),Z=13(0)–15(2)C(0,2),Z=13(0)–15(2)
= -30 (minimum)
Hence , the minimum value is -30 at the point (0, 2).
4. The feasible solution for an LPP is shown in Figure. Let Z = 3x – 4y be the objective
function. Minimum of Z occurs at

Solution:

Corner points Z = 3x – 4y

(0, 0) 0

(5,0) 15

(6,8) -14

(6 ,5) -2

(4,10) -28

(0,8) -32……………..(Min.)
The minimum value occurs at (0,8)

5. Solve the system of linear inequations: x + 2y ≤ 10; 2x + y ≤ 8.


Solution:
Draw the st. lines x + 2y = 10 and 2x + y = 8.
These lines meet at E (2,4).
Hence, the solution of the given linear inequations is shown as shaded in the
following figure :

6. Find the linear constraints for which the shaded area in the figure below is the
solution set:

Solution:
From the above shaded portion, the linear constraints are :
2x + y ≥ 2,x – y ≤ 1,
x + 2y ≤ 8, x ≥ 0, y ≥ 0

7. Minimize Z = 50x+70y …(Z is cost , x and y Food Type1 and Type 2 ) (1)
Subject to 2x + y ≥ 8 …(2)
x + 2y ≥ 10 ….(3)
and x ≥ 0, y ≥0 ……..(4)
For the solution, we draw the lines :
x = 0,y = 0,2x + y = 8 and x + 2y = 10
The feasible regin is as shown with vertices C(10,0), E(2,4) and B(0,8).
Applying Corner Point Method, we have

Corner Point Z = 50x + 70y

C: (10,0) 500

E:(2,4) 380 (Minimum)

B: (0,8) 560

Thus minimum cost is ₹380 when 2 kg of Food I and 4 kg of Food II are mixed.

8. maximize when p is profit , x and y are Toy Type 1 and Toy Type 2
P = 50x + 60y …(1)
Subject to constraints :
20x +10v ≤ 180 …(2)
10x + 20y ≤ 120 …….(3)
10x + 30y ≤ 150 ……(4)
and x ≥ 0, y ≥ 0 …(5)
Applying Corner Point Method we have :

Corner point P = 50jc + 60y

O: (0,0) 0

F:(0,5) 300

G: (6,3) 480

H: (8,2) 520 (Maximize)

A: (9,0) 450

Hence, maximum profit is ₹520 when x = 8 and y = 2. i.e., when 8 toys of type A and
2 toys of type B are made

9.

.
10.

.
LINEAR PROGRAMMING
MULTIPLE CHOICE QUESTIONS

1 The corner points of the feasible region determined by the system of linear
constraints are (0, 0), (0,40), (20,40),(60,20),(60,0).The objective function is
Compare the quantity in Column A and Column B

Column A Column B
Maximum of Z 325

(a) The quantity in column A is (b)The quantity in column B is


greater greater
(c) The two quantities are equal. (d) The relationship cannot be
determined on the basis of the
information supplied.

2 The feasible solution for a LPP is shown in given figure. Let Z=3x-4y be
the objective function. Minimum of Z occurs at

(4, 10)

(0, 8) (6, 8)

(6, 5)

(o,o) (5,0)

(a) (0,0) (b) (0,8)


(c) (5,0) (d) (4,10)

3 Corner points of the feasible region determined by the system of linear


constraints are (0,3),(1,1) and (3,0). Let Z= px+qy, where p, q>0. Condition on
p and q so that the minimum of Z occurs at (3,0) and (1,1) is
(a) p=2q (b) p=q/2
(c) p=3q (d) p=q
4 The set of all feasible solutions of a LPP is a ____ set.
(a) Concave (b) Convex
(c) Feasible (d) None of these
5 Corner points of the feasible region for an LPP are (0,2), (3,0), (6,0), (6,8) and
(0,5). Let F=4x+6y be the objective function. Maximum of F – Minimum of F =
(a) 60 (b) 48
(c) 42 (d) 18
6 In a LPP, if the objective function Z = ax+by has the same maximum value on
two corner points of the feasible region, then every point on the line segment
joining these two points give the same……….value.
(a) minimum (b) maximum
(c) zero (d) none of these
7 In the feasible region for a LPP is ………, then the optimal value of the
objective function Z = ax+bymayormaynot exist.
(a) bounded (b) unbounded
(c) in circled form (d) in squared form
8 A linear programming problem is one that is concerned with finding the …A …
of a linear function called …B… function of several values (say x and y),
subject to the conditions that the variables are …C… and satisfy set of linear
inequalities called linear constraints.
(a) Objective, optimal value, (b) Optimal value, objective,
negative negative
(c) Optimal value, objective, non- (d) Objective, optimal value, non-
negative negative
9 Maximum value of the objective function Z = ax+by in a LPP always occurs at
only one corner point of the feasible region.
(a) true (b) false
(c) can’t say (d) partially true
10 Region represented by x≥0,y≥0 is:
(a) First quadrant (b) Second quadrant
(c) Third quadrant (d) Fourth quadrant
11 Z =3x + 4y,
Subject to the constraints x+y 1, x,y ≥0.
the shaded region shown in the figure as OAB is bounded and
thecoordinatesof corner points O, A and B are (0,0),(1,0) and (0,1),
respectively.

The maximum value of Z is 2.


(a) true (b) false
(c) can’t say (d) partially true

12 The feasible region for an LPP is shown shaded in the figure. Let Z = 3x-4y be
objective function. Maximum value of Z is:

(a) 0 (b) 8
(c) 12 (d) -18
13 The maximum value of Z = 4x+3y, if the feasible region for an LPP is as
shown below, is

(a) 112 (b) 100


(c) 72 (d) 110
14 The feasible region for an LPP is shown shaded in the figure. Let Z = 4x-3y be
objective function. Maximum value of Z is:

(a) 0 (b) 8
(c) 30 (d) -18
15 In the given figure, the feasible region for a LPP is shown. Find the maximum
and minimum value of Z = x+2y.

(a) 8, 3.2 (b) 9, 3.14


(c) 9, 4 (d) none of these
16 The linear programming problem minimize Z= 3x+2y,subject to constraints
x+y8, 3x+5y 15, x,y ≥0, has
(a) One solution (b) No feasible solution
(c) Two solutions (d) Infinitely many solutions
17 The graph of the inequality 2x+3y > 6 is:
(a) half plane that contains the (b) half plane that neither
origin contains the origin nor the points of
the line 2x+3y =6
(c) whole XOY-plane excluding the (d) entire XOY-plane
points on the line 2x+3y =6
18 Of all the points of the feasible region for maximum or minimum of objective
function the points
(a) Inside the feasible region (b) At the boundary line of the
feasible region
(c) Vertex point of the boundary of (d) None of these
the feasible region
19 The maximum value of the object function Z = 5x + 10 y subject to the constraints
x + 2y ≤ 120, x + y ≥ 60, x – 2y ≥ 0, x ≥ 0, y ≥ 0 is
(a) 300 (b) 600
(c) 400 (d) 800
20 Z = 6x + 21 y, subject to x + 2y ≥ 3, x + 4y ≥ 4, 3x + y ≥ 3, x ≥ 0, y ≥ 0. The
minimum value of Z occurs at
(a) (4, 0) (b) (28, 8)
(c) (2,2/7 ) (d) (0, 3)

21 Shape of the feasible region formed by the following constraints x + y ≤ 2,


x + y ≥ 5, x ≥ 0, y ≥ 0
(a) No feasible region (b) Triangular region
(c) Unbounded solution (d) Trapezium
22 Maximize Z = 4x + 6y, subject to 3x + 2y ≤ 12, x + y ≥ 4, x, y ≥ 0.
(a) 16 at (4, 0) (b) 24 at (0, 4)
(c) 24 at (6, 0) (d) 36 at (0, 6)
23 Feasible region for an LPP shown shaded in the
following figure. Minimum of Z = 4x+3y occurs at the
point:

(a) (0,8) (b) (2,5)


(c) (4,3) (d) (9,0)
24 The region represented by the inequalities
x ≥ 6, y ≥ 2, 2x + y ≤ 0, x ≥ 0, y ≥ 0 is
(a) unbounded (b) a polygon
(c) exterior of a triangle (d) None of these

25 Minimize Z = 13x – 15y subject to the constraints : x + y ≤ 7, 2x – 3y + 6 ≥ 0 , x ≥


0, y ≥ 0.

(a) -23 (b) -32


(c) -30 (d) -34
Answer Key:-
Q: 1 b Q: 2 b Q: 3 b Q: 4 a Q: 5 a
Q: 6 b Q: 7 b Q: 8 c Q: 9 b Q: 10 a
Q: 11 b Q: 12 a Q: 13 a Q: 14 c Q: 15 b
Q: 16 b Q: 17 b Q: 18 c Q: 19 b Q: 20 c
Q: 21 a Q: 22 d Q: 23 b Q: 24 d Q: 25 c
ASSERTION AND REASONING TYPE QUESTIONS

1. Assertion (A): Feasible region is the set of points which satisfy all of the
given constraints.
Reason (R): The optimal value of the objective function is attained at the points
on X-axisonly.
A. Both A and R are true and R is the correct explanation ofr A
B. Both A and R are true but R is NOT the correct explanation of A
C. A is true but R is false.
D. A is false but R is true.
E. Both A and R are false.
2. Assertion (A): It is necessary to find objective function value at every
point in the feasible region to find optimum value of the objective function.
Reason(R):For the constrains2x+3y≤6, 5x+3y≤15, x≥0 and y≥0 cornner points
of the feasible region are (0,2), (0,0) and (3,0).
A. Both A and R are true and R is the correct explanation of A
B. Both A and R are true but R is NOT the correct explanation of A
C. A is true but R is false.
D. A is false but R is true.
E. Both A and R are false.

3. Assertion (A): It is necessary to find objective function value at every point


in the feasible region to find optimum value of the objective function.
Reason(R):For the constrains2x+3y≤6, 5x+3y≤15, x≥0 and y≥0 cornner points of
the feasible region are (0,2), (0,0) and (3,0).
A. Both A and R are true and R is the correct explanation of A
B. Both A and R are true but R is NOT the correct explanation of A
C. A is true but R is false.
D. A is false but R is true.
E. Both A and R are false.

4. Assertion (A) : For the constraints of linear optimizing function Z = x 1+ x2


given by x1+ x2≤1, 3x1 + x2 ≥1,x≥0 and y≥0 there is no feasible region.
Reason (R): Z = 7x + y, subject to 5x + y ≤5, x + y ≥3, x ≥ 0, y≥ 0. The
1 5
corner points of the feasible region are (2 , 2) (0,3)𝑎𝑛𝑑 (0,5).
A. Both A and R are true and R is the correct explanation of A
B. Both A and R are true but R is NOT the correct explanation of A
C. A is true but R is false.
D. A is false but R is true.
E. Both A and R are false.
5. Assertion (A): For the constraints of a LPP problem given by x1 + 2x2≤2000,
x1 + x2 ≤ 1500, x2 ≤600 and x1, x2≥0 the points (1000, 0), (0, 500), (2, 0) lie in the
positive bounded region, but point (2000, 0) does not lie in the positive
boundedregion.
Reason (R):

A. Both A and R are true and R is the correct explanation of A


B. Both A and R are true but R is NOT the correct explanation of A
C. A is true but R is false.
D. A is false but R is true.
E. Both A and R are false.

6. Assertion (A):The graph of x≤2 and y≥2 will be situated in the first and
second quadrants.
Reason (R):

A. Both A and R are true and R is the correct explanation of A


B. Both A and R are true but R is NOT the correct explanation of A
C. A is true but R is false.
D. A is false but R is true.
E. Both A and R are false.
7. Assertion (A): The maximum value of Z = 11x+7y
Subject to the constraints are
2x+y≤6,
x≤2,x,y≥0. Occurs at the point (0,6).
Reason (R): If the feasible region of the given LPP is bounded, then the maximum
and minimum values of the objective function occurs at corner points.
A. Both A and R are true and R is the correct explanation of A
B. Both A and R are true but R is NOT the correct explanation of A
C. A is true but R is false.
D. A is false but R is true.
E. Both A and R are false.

8. Assertion (A):If an LPP attains its maximum value at two corner points of the
feasible region then it attains maximum value at infinitely many points.
Reason (R): if the value of the objective function of a LPP is same at two corners
then it is same at every point on the line joining two corner points.
A. Both A and R are true and R is the correct explanation of A
B. Both A and R are true but R is NOT the correct explanation of A
C. A is true but R is false.
D. A is false but R is true.
E. Both A and R are false.

9. Consider, the graph of constraints


stated as linear inequalities as
below:
5x+y≤100,
x+y≤60,
x,y≥0.

Assertion (A): The points (10,50),


(0,60) , (10,10) and (20,0) are feasible
solutions.
Reason (R): Points within and on the
boundary of the feasible region represent
feasible solutions of the constraints.

A. Both A and R are true and R is the correct explanation of A


B. Both A and R are true but R is NOT the correct explanation of A
C. A is true but R is false.
D. A is false but R is true.
E. Both A and R are false.
10. Consider, the graph of constraints stated as linear inequalities as below:
5x+y≤100,
x+y≤60,
x,y≥0.

Assertion (A): (25,40) is an infeasible solution of the problem.


Reason (R):Any point inside the feasible region is called an infeasible solution.

A. Both A and R are true and R is the correct explanation of A


B. Both A and R are true but R is NOT the correct explanation of A
C. A is true but R is false.
D. A is false but R is true.
E. Both A and R are false.

11. Assertion (A): The region represented by the set {(x,y): 4≤x2+y2≤9} is a
convex set.
Reason (R): The set {(x,y): 4≤x2+y2≤9} represents the region between two
concentric circles of radii 2 and 3.

A. Both A and R are true and R is the correct explanation of A


B. Both A and R are true but R is NOT the correct explanation of A
C. A is true but R is false.
D. A is false but R is true.
E. Both A and R are false.

12. Assertion (A):For an objective function Z= 15x + 20y, corner points are (0,0),
(10,0), (0,15) and (5,5). Then optimal values are 300 and 0 respectively.
Reason (R):The maximum or minimum value of an objective function is known as
optimal value of LPP. These values are obtained at corner points.

A. Both A and R are true and R is the correct explanation ofr A


B. Both A and R are true but R is NOT the correct explanation of A
C. A is true but R is false.
D. A is false but R is true.
E. Both A and R are false.

13. Assertion (A): For the LPP Z= 3x+2y, subject to the constraints
x+2y≤2; x≥0; y≥ 0 both maximum value of Z and Minimum value of Z can be
obtained.
Reason (R):If the feasible region is bounded then both maximum and minimum
values of Z exists.

A. Both A and R are true and R is the correct explanation of A


B. Both A and R are true but R is NOT the correct explanation of A
C. A is true but R is false.
D. A is false but R is true.
E. Both A and R are false.

14. Assertion (A):The linear programming problem, maximize Z = x+2y subject to


constraints x-y≤10,2x + 3y≤20 and x≥0; y≥ 0. It gives the maximum value of Z as
40/3.
Reason (R):To obtain maximum value of Z, we need to compare value of Z at all
the corner points of the shaded region.

A. Both A and R are true and R is the correct explanation of A


B. Both A and R are true but R is NOT the correct explanation of A
C. A is true but R is false.
D. A is false but R is true.
E. Both A and R are false.

15. Assertion (A):Consider the linear programming problem. Maximise Z=4x+y


Subject to constraints x+ y≤50; x+ y≥100 and x, y ≥0. Then, maximum value of
Z is 50.
Reason (R):If the shaded region is bounded then maximum value of objective
function can be determined.

A. Both A and R are true and R is the correct explanation of A


B. Both A and R are true but R is NOT the correct explanation of A
C. A is true but R is false.
D. A is false but R is true.
E. Both A and R are false.
CASE STUDY
CS 1 A train can carry a maximum of 300
passengers. A profit of Rs. 800 is
made on each executive class and
Rs. 200 is made on each economy
class. The IRCTC reserves at least 40
tickets for executive class. However,
atleast 3 times as many passengers
prefer to travel by economy class,
than by executive class. It is given
that the number of executive class
ticket is Rs. x and that of economy
class ticket is Rs. y.
Optimize the given problem.

Based on the above information, answer the following questions.

1 The objective function of the LPP is:

(a) Maximise Z = 800x + 200y (b) Maximise Z = 200x + 800y

(c) Minimise Z = 800x + 200y (d) Minimise Z = 200x + 800y

2 Which among these is a constraint for this LPP?

(a) x+y≥300 (b) y≥3x

(c) x≤40 (d) y≤3x

3 Which among these is not a corner point for this LPP?

(a) (40,120) (b) (40, 260)

(c) (30, 90) (d) (75, 225)

4 The maximum profit is:

(a) Rs.56000 (b) Rs. 84000

(c) Rs. 205000 (d) Rs. 105000

5 Which corner point the objective function has minimum value?

(a) (40,120) (b) (40, 260)

(c) (30, 90) (d) (75, 225)

CS 2 A manufacturing company makes two models X and Y of a product. Each piece of


model X requires 9 labour hours for fabricating and 1 labour hour for finishing.
Each piece of model Y requires 12 labour hours of fabricating and 3 labour hours
for finishing, the maximum labour hours available for fabricating and finishing are
180 and 30 respectively. The company makes a profit of Rs. 8000 on each piece of
model X and Rs. 12000 on each piece of model Y. Assume x is the number of
pieces of model X and y is the number of pieces of model Y.

Based on the above information, answer the following questions.

1 Which among these is not a constraint for this LPP?

(a) 9x+12y≥180 (b) 3x+4y≤60

(c) x+3y≤30 (d) None of these

2 The shape formed by the common feasible region is:

(a) Triangle (b) Quadrilateral

(c) Pentagon (d) hexagon

3 Which among these is a corner point for this LPP?

(a) (0,20) (b) (6,12)

(c) (12,6) (c) (10,0)

4 Maximum of Z occurs at

(a) (0,20) (b) (0,10)

(c) (20,10) (d) (12,6)

5 The sum of maximum value of Z is:

(a) 168000 (b) 160000

(c) 120000 (d) 180000

CS 3 Suppose a dealer in rural area wishes to purchase a


number of sewing machines. He has only Rs. 5760 to
invest and has space for atmost 20 items for storage. An
electronic sewing machine costs him Rs. 360 and a
manually operated sewing machine Rs. 240. He can sell
an electronic sewing machine at a profit of Rs. 22 and a
manually operated sewing machine at a profit of Rs.18.
Based on the above information, answer the following
questions.
1 Let x and y denote the number of electronic sewing machines and manually
operated sewing machines purchased by the dealer. If it is assumed that the
dealer purchased atleast one of the given machines then:

(a) x+y≥ 0 (b) x+y< 0


(c) x+y> 0 (c) x+y≤ 0
2 Let the constraints in the given problem is represented by the following
inequalities:
x+y≤20; 360x+240y≤5760 and x,y≥0. Then which of the following point lie in its
feasible region.

(a) (0,24) (b) (8,12)


(c) (20,2) (d) None of these
3 If the objective function of the given problem is maximize Z = 22x+18y, then its
optimal value occur at:

(a) (0,0) (b) (16,0)


(c) (8,12) (d) (0,2)
4 Suppose the following shaded region APDO, represent the feasible region
corresponding to mathematical formulation of the given problem.
Then which of the following represent the coordinates of one of its corner points.

(a) (0,24) (b) (12,8)


(c) (8,12) (d) (6,14)
5 If an LPP admits optimal solution at two consecutive vertices of a feasible region,
then

(a) The required optimal solution (b) The optimal solution occurs at
is at a mid pointof the line joining two every point on the line joining these two
points. points.
(c) The LPP under consideration (d) The LPP under consideration
is not solvable. must be reconstructed.
ANSWERS:

ASSERTION AND REASONING


1 C 2 D 3 D 4 A 5 A
6 A 7 A 8 A 9 A 10 C
11 D 12 A 13 A 14 A 15 D
CASE STUDY
CS-1 1) A 2) B 3) C 4) D 5) A
CS-2 I)A 2) B 3) C 4) D 5) A
CS-3 I)C 2) B 3) C 4) C 5) B
PROBABILITY
MULTIPLE CHOICE QUESTIONS

1) A problem in Mathematics is given to three students whose chances of solving it


are 1/2, 1/3, 1/4 respectively. If the events of their solving the problem are
independent then the probability that the problem will be solved, is
A. 1/4 B. 1/3 C. 1/2 D. ¾
2) P(A ∩ B) is equal to:
A. P(A) . P(B|A) B. P(B) . P(A|B) C. Both A and B
D. None of these

Answer: C. Both A and B


Explanation: By multiplication theorem of probability.

3) If P (A) = 0.8, P (B) = 0.5 and P (B|A) = 0.4, what is the value
of P (A ∩ B)?
A. 0.32 B. 0.25 C. 0.1 D. 0.5

Answer: A. 0.32
Explanation: Given, P (A) = 0.8, P (B) = 0.5 and P (B|A) = 0.4
By conditional probability, we have;
P (B|A) = P(A ∩ B)/P(A)
⇒ P (A ∩ B) = P(B|A). P(A) = 0.4 x 0.8 = 0.32

4) If P (A) = 6/11, P (B) = 5/11 and P (A ∪ B) = 7/11, what is the


value of P(B|A)?
A. ⅓ B. ⅔ C. 1 D. None of the above

Answer: B. ⅔
Explanation: By definition of conditional probability we know;
P(B|A) = P(A ∩ B)/P(A) …(i)
Also,
P(A ∩ B) = P(A) + P(B) – P(A U B)
= 6/11 + 5/11 – 7/11
= 4/11
Now putting the value of P(A ∩ B) in eq.(i), we get;
P(B|A) = (4/11)/(6/11) = 4/6 = ⅔

5) Find P(E|F), where E: no tail appears, F: no head appears, when


two coins are tossed in the air.
A. 0 B. ½ C. 1 D. None of the above

Answer: A. 0
Explanation: Given,
E: no tail appears
And F: no head appears
⇒ E = {HH} and F = {TT}
⇒ E ∩ F = ϕ
As we know, two coins were tossed;
P(E) = ¼
P(F) = ¼
P(E ∩ F) = 0/4 = 0
Thus, by conditional probability, we know that;
P(E|F) = P(E ∩ F)/P(F)
= 0/(¼)
= 0

6) If P(A ∩ B) = 70% and P(B) = 85%, then P(A/B) is equal to:


A. 17/14 B. 14/17 C. ⅞ D. ⅛
Answer: B.14/17
Explanation: By conditional probability, we know;
P(A|B) = P(A ∩ B)/P(B)
= (70/100) x (100/85)
= 14/17
7) If P(A) = 0.4, P(B) = 0.7 and P(B/A) = 0.6. Find P(A ∪ B).
A. 0.46 B. 0.86 C. 0.76 D. 0.54
Answer: B. 0.86
Explanation: Given,
P(A) = 0.4, P(B) = 0.7 and P(B/A) = 0.6
By conditional probability, we know;
P(B|A) = P(A ∩ B)/P(A)
⇒ 0.6 × 0.4 = P (A ∩ B)
⇒ P(A ∩ B) = 0.24
Now, P(A ∪ B) = P(A) + P(B) – P(A ∩ B)
= 0.4 + 0.7 – 0.24
= 0.86
8) An urn contains 10 black and 5 white balls. Two balls are drawn
from the urn one after the other without replacement. What is the
probability that both drawn balls are black?
A. 3/7 B. 7/3 C. 1/7 D. ⅓
Answer: A. 3/7
Explanation: Let E and F denote the events that the first and
second ball drawn is black, respectively.
We need to find P(E ∩ F) or P (EF).
P(E) is the probability of black ball first drawn.
P(E) = 10/15
Now, 9 black balls are left in the urn.
P(F|E) = 9/14
By multiplication rule;
P (E ∩ F) = P (E) P(F|E)
= 10/15 x 9/14
= 3/7
9) If E and F are independent events, then;
A. P(E ∩ F) = P(E)/ P(F) B. P(E ∩ F) = P(E) + P(F)
C. P(E ∩ F) = P(E) . P(F) D. None of the above
Answer: C. P(E ∩ F) = P(E) . P(F)
Explanation: Two events E and F are said to be independent if;
P(F|E) = P (F) given P (E) ≠ 0
and
P (E|F) = P (E) given P (F) ≠ 0
Now, by the multiplication rule of probability, we have
P(E ∩ F) = P(E) . P (F|E) … (1)
If E and F are independent, then;
P(E ∩ F) = P(E) . P(F)
10) If A and B are two independent events, then the probability
of occurrence of at least one of A and B is given by:
A. 1+ P(A′) P (B′) B. 1− P(A′) P (B′) C. 1− P(A′) + P (B′)
D. 1− P(A′) – P (B′)
Answer: B. 1− P(A′) P (B′)
Explanation: P(at least one of A and B) = P(A ∪ B)
= P(A) + P(B) − P(A ∩ B)
= P(A) + P(B) − P(A) P(B)
= P(A) + P(B) [1−P(A)]
= P(A) + P(B). P(A′)
= 1− P(A′) + P(B) P(A′)
= 1− P(A′) [1− P(B)]
= 1− P(A′) P (B′)
11) The probability of solving the specific problems
independently by A and B are 1/2 and 1/3 respectively. If both try
to solve the problem independently, find the probability that
exactly one of them solves the problem.
A. 1 B. ½ C. ⅓ D. ¼
Answer: B. ½
Explanation: P(A) = ½
P(B) = ⅓
Since, A and B are independent events, therefore;
⇒ P (A ∩ B) = P (A). P (B)
⇒ P (A ∩ B) = ½ × 1/3 = 1/6
P (A’) = 1 – P (A) = 1 – 1/2 = 1/2
P (B’) = 1 – P (B) = 1 – 1/3 = 2/3
Now the probability that exactly one of them solved the problem is
either the problem is solved by A and not B or vice versa.
P (A).P (B’) + P (A’).P (B)
= ½ (2/3) + ½ (1/3)
= 1/3 + 1/6 = 3/6
⇒ P (A).P (B’) + P (A’).P (B) = ½

3 MARKS QUESTIONS
1. The random variable X has a probability distribution P X of the following form, where 'k' is
some real number:

(i) Determine the value of k.


(ii) Find P  X  2
(iii) Find P  X  2
2. Let X denote the number of colleges where you will apply after your results and P(X = x)
denotes your probability of getting admission in x number of colleges. It is given that
where k is a positive constant. Find the value of k. Also, find the probability that you
will get admission in
(i) exactly one college
(ii) at most 2 colleges
(iii) at least 2 colleges.
3. Suppose a girl throws a the. If she gets 1 or 2, she tosses a coin three times and notes the
number of tails. If she gets 3, 4, 5 or 6, she tosses a coin once and notes whether a ‘head’
or ‘tail’ is obtained. If she obtained exactly one ‘tail’, what is the probability that she threw
3, 4, 5 or 6 with the die?
4. Two numbers are selected at random (without replacement) from the first five positive
integers. Let X denotes the larger of the two numbers obtained. Find the mean and
variance of X.
5. There are 4 cards numbered 1, 3, 5 and 7, one number on one card. Two cards are drawn
at random without replacement. Let X denote the sum of the numbers on the two drawn
cards. Find the mean and variance of X.
6. A bag X contains 4 white balls and 2 black balls, while another bag Y contains 3 white
balls and 3 black balls. Two balls are drawn (without replacement) at random from one of
the bags and were found to be one white and one black. Find the probability that the balls
were drawn from bag Y.
CASE BASED STUDY QUESTIONS
Q1 The right car insurance can help protect you, your family members, your passengers and
other drivers. By investing a small amount now in your own protection, you can help avoid costly
expenses in the future. With appropriate coverage, you can avoid negotiating with other drivers
and property owners involved in the accident and managing the costs of repairing or replacing
your vehicle. An insurance company insured 2000 scooter drivers, 4000 car drivers and 6000
truck drivers. The probability of an accident is 0.01, 0.03 and 0.15 respectively. One of the
insured persons meets with an accident.

Based on the above information answer the following questions.

(a) What is the probability that he is a scooter driver?


(b) What is the probability that he is a Truck driver?

Q2 Read the following passage and answer the questions given below:
In an Office three employees Jayant, Sonia and Oliver process incoming copies of a certain
form. Jayant processes 50%of the forms, Sonia processes 20% and Oliver the remaining
30% of the forms. Jayant has an error rate of 0.06 , Sonia has an error rate of 0.04 and Oliver
has an error rate of 0.03 . Based on the above information, answer the following questions.

(i) Find the probability that Sonia processed the form and committed an error.
(ii) Find the total probability of committing an error in processing the form.
iii) The manager of the Company wants to do a quality check. During inspection, he selects a
form at random from the days output of processed form. If the form selected at random has
an error, find the probability that the form is not processed by Jayant.

Q3 Read the following passage and answer the questions given below . Consider the experiment
of tossing a coin . If the coin shows head, toss it again but if it shows tail, then throw a die
A is the event that ’ the die shows a number greater than 4 ‘ and
B is the event ‘ the coin is showing at least one tail ‘,
(i) Find Probability that die shows a number greater than 4 and the coin is showing at
least one tail
(ii) (ii) Find Probability of the event that the die shows a number greater than 4 given that
there is at least one tail
Sol A ={ (T,5),(T,6)}
B = { ( H ,T), ( T,1),(T,2),(T,3),(T,4),(T,5),(T,6)}
A ∩ B = { (T,5),(T,6)}
P(B) = P( H ,T)+P ( T,1)+P(T,2)+P(T,3)+P(T,4)+P(T,5)+P(T,6)
= ¼ + 1/12 +1/12+1/12+1/12+1/12+1/12
= 3/4

P(A ∩B) =P (T,5)+P(T,6)


= 1/12 +1/12 = 1/6
P(A/B) = (1/6)/(3/4) =2/9

Q4 A coach is training 3 players. He observes that the player A can hit a target 4 times in 5
shots, player B can hit 3 times in 4 shots and the player C can hit 2 times in 3 shots From this
situation answer the following:

i). Let the target is hit by A, B: the target is hit by B and, C: the target is hit by A and C.
Then, the probability that A, B and, C all will hit, is
a. 4/5
b. 3/5
c. 2/5
d. 1/5

ii). Referring to (i), what is the probability that B, C will hit and A will lose?

a. 1/10 b. 3/10 c. 7/10 d. 4/10

iii) With reference to the events mentioned in (i), what is the probability that ‘any two of A,
B and C will hit?
1. 1/30 2. 11/30 3. 17/30 4. 13/30

iv) What is the probability that ‘none of them will hit the target’?
a. 1/30 b. 1/60 c. 1/15 d. 2/15

v) . What is the probability that at least one of A, B or C will hit the target?
a. 59/60 b. 2/5 c. 3/5 d. 1/60
Answers: i). (c) 2/5 ii). (a) 1/10 iii) . (d) 13/30 iv. (b) 1/60 v). (a) 59/60

Q 5) The reliability of a COVID PCR test is specified as follows: Of people having COVID,
90% of the test detects the disease but 10% goes undetected. Of people free of COVID,
99% of the test is judged COVID negative but 1% are diagnosed as showing COVID
positive. From a large population of which only 0.1% have COVID, one person is
selected at random, given the COVID PCR test, and the pathologist reports him/her as
COVID positive. Based on the above information, answer the following

i. What is the probability of the ‘person to be tested as COVID positive’ given that ‘he is
actually having COVID?
a. 0.001 b. 0.1 c. 0.8 d. 0.9
ii. What is the probability of the ‘person to be tested as COVID positive’ given that ‘he is
actually not having COVID’?
a. 0.01 b. 0.99 c. 0.1 d. 0.001

iii. What is the probability that the ‘person is actually not having COVID?
a. 0.998 b. 0.999 c. 0.001 d. 0.111
iv. . What is the probability that the ‘person is actually having COVID given that ‘he is
tested as COVID positive’?
a. 0.83 b. 0.0803 c. 0.083 d. 0.089
v. What is the probability that the ‘person selected will be diagnosed as COVID positive’?
a. 0.1089 b. 0.01089 c. 0.0189 d. 0.189

Answers
i). (d) 0.9 (a) 0.01 iii) . (b) 0.999 iv). (c) 0.083 v). (b) 0.01089

Q 6) In answering a question on a multiple choice test for class XII, a student either knows
the answer or guesses. Let 3/5 be the probability that he knows the answer and 2/5 be
the probability that he guesses. Assume that a student who guesses at the answer will
be correct with probability 1/3. Let E1, E2, E be the events that the student knows the
answer, guesses the answer and answers correctly respectively. Based on the above
information, answer the following
i. What is the value of P(E1)?
a. 2/5 b. 1/3 c. 1 d. 3/5

ii. Value of P(E | E1) is


a. 1/3 b. 1 c. 2/3 d. 415

iii. ∑ P(E|𝐸𝑘 ) P(𝐸𝑘) 𝑘=2 𝑘=1 Equals


a. 11/15 b. 4/15 c. 1/5 d. 1

iv. Value of ∑ P(𝐸𝑘) 𝑘=2 𝑘=1


a. 1/3 b. 1/5 c. 1 d. 3/5

v. What is the probability that the student knows the answer given that he answered it
correctly?
a. 2/11 b. 5/3 c. 9/11 d. 13/3

Answers I). (d) 3/5 ii) (b) 1 iii. (a) 11/15 iv. (c) 1 v). (c) 9/1
ASSERTION AND REASONING TYPE QUESTIONS OF MATRICES

1. Assertion (A) 3 𝑥 3
2
The value of x for which | | =| | is ± 2√2
𝑥 1 4
1
Reason(R) 𝑎 𝑏]
The determinant of a matrix A order 2x2, A= [ is = ad – bc
𝑐 𝑑
A Both A and R are true and R is the correct explanation of A
B Both A and R are true but R is NOT the correct explanation of A.
C A is true but R is false
D A is false but R is true
E Both A and R are false

2. Assertion (A) 𝑥 2 6 2
The value of x for which | | =| | is ± 6
18 𝑥 18 6
Reason(R) 𝑎 𝑏
The determinant of a matrix A order 2 x 2 , A= [ ] is = ab–
𝑐 𝑑
dc

A Both A and R are true and R is the correct explanation of A


B Both A and R are true but R is NOT the correct explanation of A.
C A is true but R is false
D A is false but R is true
E Both A and R are false

3. Assertion (A) 1 0 1
If A= [0 1 2] then |3𝐴| =9|𝐴|
0 0 4
Reason(R) If A is a square matrix of order n then |𝑘𝐴| =kn|𝐴|
A Both A and R are true and R is the correct explanation of A
B Both A and R are true but R is NOT the correct explanation of A.
C A is true but R is false
D A is false but R is true
E Both A and R are false

4. Assertion (A) If A is a non singular square matrix of order 3x3 and|𝐴| =5


then |𝑎𝑑𝑗𝐴| is equal to 125
Reason(R) |𝑎𝑑𝑗𝐴| =(|𝐴| )n-1where n is order of A.

A Both A and R are true and R is the correct explanation of A


B Both A and R are true but R is NOT the correct explanation of A.
C A is true but R is false
D A is false but R is true
E Both A and R are false

5. Assertion (A) 5 −7]and -1 [7 6]then 23 31


Let A-1= [ B = (AB) -1 =[ ]
−2 3 8 7 26 35
Reason(R) (AB) -1 = A-1B-1
A Both A and R are true and R is the correct explanation of A
B Both A and R are true but R is NOT the correct explanation of A.
C A is true but R is false
D A is false but R is true
E Both A and R are false

6. Assertion (A) 1 2 0
Value of x for which the matrix [ 0 1 2] is singular is 5
−1 2 𝑥
Reason(R) A square matrix is singular if |𝐴| =0
A Both A and R are true and R is the correct explanation of A
B Both A and R are true but R is NOT the correct explanation of A.
C A is true but R is false
D A is false but R is true
E Both A and R are false

7. Assertion (A) 2 3 1
The minor of the element 3 in the matrix [0 −2 4]is 8.
2 1 5
Reason(R) : Minor of an element aij of a matrix is the determinant
obtained by deleting its jth row and ith column
A Both A and R are true and R is the correct explanation of A
B Both A and R are true but R is NOT the correct explanation of A.
C A is true but R is false
D A is false but R is true
E Both A and R are false
8. Assertion (A) For two matrices A and B of order 3, |𝐴|=2|𝐵| = -3 then
if|2𝐴𝐵| is -48.
Reason(R) For a square matrix A, A(adj A)=(adj A)A= |𝐴| I
A Both A and R are true and R is the correct explanation of A
B Both A and R are true but R is NOT the correct explanation of A.
C A is true but R is false
D A is false but R is true
E Both A and R are false

9. Assertion (A) Values of k for which area of the triangle with vertices (2, -6),
(5,4) and (k,4) is 35 sq units are 12, 2.
Reason(R) Area of a triangle with vertices A (x1, y1),B (x2, y2) and C (x3,
𝑥1 𝑦1 1
1
y3) is |𝑥2 𝑦2 1|
2
𝑥3 𝑦3 1

A Both A and R are true and R is the correct explanation of A


B Both A and R are true but R is NOT the correct explanation of A.
C A is true but R is false
D A is false but R is true
E Both A and R are false

10. Assertion (A) The points A(a, b+c), B(b, c+a) and C(c, a+b) are collinear.
Reason(R) Three points A (x1, y1) , B(x2, y2) and C(x3, y3) are collinear
if area of a triangle ABC is zero.
A Both A and R are true and R is the correct explanation of A
B Both A and R are true but R is NOT the correct explanation of A.
C A is true but R is false
D A is false but R is true
E Both A and R are false

11. Assertion (A) 1 −1 2


Inverse of the matrix [0 2 −3] is the
3 −2 4
−2 0 1
matrix[ 9 2 −3]
6 1 −2
Reason(R) 1
: Inverse of a square matrix A, if it exits is given by A-1 =𝐼𝐴𝐼
adjA
A Both A and R are true and R is the correct explanation of A
B Both A and R are true but R is NOT the correct explanation of A.
C A is true but R is false
D A is false but R is true
E Both A and R are false
12. Assertion (A) 2 −1 4 0
For a matrix A=[ ], A. adj A =[ ]
−3 4 0 4
Reason(R) For a square matrix A , A( adj A) = (adj A)A= |𝐴| I
A Both A and R are true and R is the correct explanation of A
B Both A and R are true but R is NOT the correct explanation of A.
C A is true but R is false
D A is false but R is true
E Both A and R are false
13. Assertion (A) In a square matrix of order 3 the minor of an element a 22 is
6 then cofactor of a22is -6.
Reason(R) Cofactor an element aij = AIJ= ( -1)i+jMij
A Both A and R are true and R is the correct explanation of A
B Both A and R are true but R is NOT the correct explanation of A.
C A is true but R is false
D A is false but R is true
E Both A and R are false

14. Assertion (A) 2 3 2 −3


Inverse of a matrix A =[ ] is the matrix A-1 =[ ]
1 2 −1 2
Reason(R) 𝑎 𝑏 𝑑 −𝑏
: Inverse of a square matrix ( )is ( ).
𝑐 𝑑 −𝑐 𝑎
A Both A and R are true and R is the correct explanation of A
B Both A and R are true but R is NOT the correct explanation of A.
C A is true but R is false
D A is false but R is true
E Both A and R are false

15. Assertion (A) If A is an invertible matrix of order 2, and det A= 3 then


1
det( A-1)is equal to 3
Reason(R) If A is an invertible matrix of order 2 then det (A-1) =
det 𝐴
A Both A and R are true and R is the correct explanation of A
B Both A and R are true but R is NOT the correct explanation of A.
C A is true but R is false
D A is false but R is true
E Both A and R are false

16. Assertion (A) The equation of the line joining (1,2) and (3,6) using
determinants is y= 3x.
Reason(R) The area of ∆PAB is zero if P(x, y) is a point on the line
joining a A and B.
A Both A and R are true and R is the correct explanation of A
B Both A and R are true but R is NOT the correct explanation of A.
C A is true but R is false
D A is false but R is true
E Both A and R are false

Answers
ASSERTION AND REASONING
1 A 2 C 3 D 4 D 5 E

6 D 7 E 8 B 9 D 10 A

11 A 12 D 13 D 14 C 15 C

16 D
General Instructions :
Read the following instructions very carefully and strictly follow them:
(i) This question paper contains 38 questions. Allquestions are compulsory.
(ii) This question paper is divided into five Sections - A, B, C, D and E.
(iii) In Section A, Questions no. I to 18 are multiple choice questions (MCQs) and
questions number 19 and 20 are Assertion-Reason based questions of 1 mark
each.
(iv) In Section B, Questions no. 21 to 25 are very short answer (VSA) type
questions, carrying 2 marks each.
(v) In Section C, Questions no. 26 to 31 are short answer (SA) type questions,
carrying 3 marks each.
(vi) In Section D, Questions no. 32 to 35 are long answer (LA) type questions
carrying 5 marks each.
(vii) In Section E, Questions no. 36 to 38 are case study based questions carrying
4 marks each.
(viii) There is no overall choice. However, an internal choice has been provided in
2 questions in Section B, 3 questions in Section C, 2 questions in Section Dand
2 questions in Section E.
(ix) Use of calculators is not allowed.
SECTION A
This section comprises multiple choice questions (MCQs) of l mark each.
IfA is a square matrix of order 3 and A = 6, then the value of adj A|
is:

(a) 6 (b) 36

(c) 27 (d) 216


T/6

The value of sin 3x dx is:

(a)
V3 1
(b) -

2 3
Hl 1
(c) (d)
2 3

65/C/1 Page 3 PT.O.


’ ’ angle between a
If a and (a + b)are all unit yectors and 0 is the
and b, then the value of 0is:
T
2 (c) (d)
(a) (b) 3 6
3 6

+j+2k is :
The projection of vector i on the vector i
2 3
1 (c) (d)
(a) (b)

child is a girl. The probability that


A family has 2 children and the elder
both children are girls is : 3
1 1 (d)
(a) (b) (c) 2 4
4 8

through the point (2, 4, 5)


8. The vector equation of a line which passesz+8
x+3 4- y is:
and is parallel to the line 3 6

(a) r =(-2i +4j -5k)+ A(31 +2j +6k )


r = (2i-4j +5k) +A(3i 2j +6k)
A
(b)

(c) T =(2í -4i +5k)+ A(3i+2j +6k)


(d) r =-2i +4fj-5k)+ (3i 2j - 6k)
2x -3 10 1
determinants and
For which value of x, are the 5 X 3 2

equal ?
-3 (c) ±2 (d) 2
(a) ±3 (b)
row and third column
The value of the cofactor of the element of second
4 3 2
in the matrix2 -1 0 is:
1 2 3

-5 (c) -11 (d) 11


(a) 5 (b)
PT.O.
65/C/1 Page 5
DD
D
the differential equation
The difference of the order and the degree of
2 3
d'y +
dy + x=0 is :
dx2 dx

(c) -1 (d) 0
(a) 1 (b) 2

1 -1 value ofk is:


19. If matrix A= and A= kA, then the
-1 1
2 (d) -1
(a) 1 (b) -2 (c)
COs 2x
-dx is equal to
2 X. COs2 X
sin
- cot x tan X + C
(a) tan x cot x + C (b)
cot x + tan x + C (d) tan x - cot x C
(c)
dx = X is
(3x + y)
The integrating factor of the differential equation dy
2 1
1 (d)
(a) 1 (b) (c) X
2 X

The point which lies in the half-plane 2x + y-4s0 is:


(a) (0, 8) (b) (1, 1)

(c) (5, 5) (d) (2, 2)


dy
14. If (cos x = (cos y*, then dx is equal to:

y tan x + log (cos y)


(a) x tan y - log (cos x)
x tan y + log (cos x)
(b)
y tan x + log (cos y)
y tan x- log (cos y)
(c)
x tan y- log (cos x)

y tan x + log (cOs y)


(d)
x tan y + log (cos x)
65/C/1 Page 7 P.T.O.
1
Then matrix X is:
15. It is given that X 3
1 01 -1|
(a) (b) 1
1 1
0
-1|
(c) (d)
-1|
16 ABCD is a parallelogram and AC and BD are its diagonals, then
If
AC + BD is :

2DA (b) 2 AB (c) 2BC (d) 2BD


(a)
0 -b cos 0, then which one of the
4. If x =a cos + b sin 0, y = a sin
following is true?
+X dy +y=0
(a) X dy +y=0 (b)
dx2 dx dñ2 dx

dy 2dy dy -y= 0
(c) +X -y= 0 (d) dy2
X
dx
dx
region of an LPP are O(0, 0),
18 The corner points of the bounded feasible
value of the objective
A(250, 0),B(200, 50) and C(0, 175). If the maximum B(200, 50), then
function Z= 2ax + by occurs at the points A(250, 0) and
the relation between a and b is :

250%
200+
(0,175C
150

100
B(200,50)
50
250,0> 300
50 100 150 200 250

a =b a = 2b
2a = b (b) 2a = 3b (c) (d)
(a)
Page 9 P.T.O.
65/C/1
Questions number 19 and 20 are Assertion and Reason based questions carrying
labelled Assertion (A) and the other
Imark each. Two statenents are given, one
(a), (6), (c) and (d)
labelled Reason (R). Select the correct answer from the codes
as given below.
Reason (R) is the
(a) Both Assertion (A) and Reason (R) are true and
correct exxplanation of the Assertion (A).
and Reason (R) is not
(b) Both Assertion (A) and Reason (R) are true
the correct explanation of the Assertion (A).
(c) Assertion (A) is true, but Reason (R) isfalse.

(d) Assertion (A) is false, but Reason (R) is true.

cot(J3)is
.19. Assertion (A): The principal value of 6

Reason (R) : Domain of cot- xis R-{-1, 1).

A(0, 0, 0), B(3, 4, 5),


L20. Assertion (A): Quadrilateral formed by vertices
C(8, 8, 8) and D(5, 4, 3) is a rhombus.
Reason (R): ABCD is a rhombus if AB=BC = CD = DA, AC BD.
SECTION B

type questions of2 marks each.


This section comprises very short answer (VSA)

If three non-zero vectors are a ,


b and c such that a . b= . C
21. -’ ’
and a xb = a x c, then show that b = c .

22. (a) Simplify :


COS X
tan-l
sin x

OR

Prove that the greatest integer function f:R’ R, given by


f(x) = x], is neither one-one nor onto.
P.T.0.
65/C/1 Page l1
\23. Function fis defined as
2x + 2, if X<2
f(x) = k, if X =2
3x, if X> 2
Find the value of k for which the function fis continuous at x = 2.

Find the intervals in which the function fx) = x - 4x + 4x + 15,


is strictly increasing.


25. (a) If a and e are three vectors such that | a | =7, | b| =24,
| c| =25 and a + b + c = 0, then find the value of
’ ’
a.b+ b.c+ c.a .
OR

(b) If a line makes angles oa, B and Ywith x-axis, y-axis and Z-axis
respectively, then prove that sin a + sin ß+ sin y = 2.

SECTION C

This section comprises short answer (SA) type questions of 3 marhs each.

26. (a) Evaluate:


/2
X sin x cOS X
dx
4
sin x + cos X
0

OR

Evaluate:
3

(|x-1| +|x-2|) dx
1

65/C/1 Page 13 P.T.0.


37. Find the particular solution of the differential equation
dy Xy
given that y = 1 when x =0.
dx +y2
OR

(b) Find the particular solution of the differential equation


(1+x) dy + 2xy = 1
given that y = 0 when x = 1.
dx 1+x2

bag B
28: (a Out of two bags, bag A contains 2 white and 3 red balls and
from
contains 4 white and 5 red balls. One ball is drawn at random
it
one of the bags and is found to be red. Find the probability that
was drawn from bag B.
OR
truth. Two
(b) Out of a group of 50 people, 20 always speak the
(without
persons are selected at random from the group
replacement). Find the probability distribution of number of
selected persons who always speak the truth.

29 Find:
COS
do
V3- 3sin - cos

graphically:
30: Solve the following Linear Programming Problem
Minimise z = 3x + 8y
subject to the constraints
3x + 4y 8
5x + 2y 11
x>0, y >0

Find:
2x +1 dx
(x + 4)
P.T.O.
65/C/1 Page 15
SECTION D
This section comprises long answer type questions (LA) of 5 marks each.
[3 2 1

32. If matrix A = 4 1 3 . fnd A-1 and hence solve the following


1 1 1

system of linear equations:


3x +2y +z= 2000
4x + y + 3z = 2500
X + y +z= 900
|
X+1 y+3 z +5
33. (a) Show that the lines and
3 5 7
X2 y-4 z6
intersect. Also find their point of
1 3 5
intersection.
OR

Find the shortest distance between the pair of lines


X-1
2
y+1
3
=Z and x+1_y-2:z=
5 1
2.
Find the area of the triangle ABC bounded by the lines represented by
the equations 5x - 2y - 10 = 0, X- y -9 = 0 and 3x 4y 6 = 0, using
integration method.

35. Show that the relation S in set R of real numbers defined by


S= ((a, b) :asb', ae R, be R}
is neither reflexive, nor symmetric, nor transitive.
OR

(b) Let R be the relation defined in the set A -(1, 2, 3, 4, 5, 6, 7}by


R= ((a, b): both a and bare either odd or even). Show that
R is an equivalence relation. Hence, find the elements of
equivalence class [1].
65/C/1 Page 17 P.T.O.
mANDAK

SECTION E

This section comprises 3 case studybased guestions of 4 marks each.

Case Study - 1

B6. In a group activity class, there are 10 students whose ages are 16, 17, 15,
14, 19, 17, 16, 19, 16 and 15 years. One student is selected at random
such that each has equal chance of being chosen and age of the student is
recorded.

On the basis of the above information, answer the following questions :


(i) Find the probability that the age of the selected student is a
composite number.

(ii) Let X be the age of the selected student. What can be the value
of X? 1

(iii) (a) Find the probability distribut:ion of random variable X and


hence find the mean age. 2

OR

(iii) (b) Astudent was selected at random and his age was found to be
greater than 15 years. Find the probability that his age is a
prime number. 2
65/C/1 Page 19 P.T.O.
Case Study - 2
A housing society wants to commission a swimming pool for its residents.
For this, they have to purchase a square piece of land and dig this to such
a depth that its capacity is 250 cubic metres. Cost of land is ? 500 per
Square metre. The cost of digging increases with the depth and cost for
the whole pool is 4000 (depth)2.

Suppose the side of the square plot is x metres and depth is h metres.
On the basis of the above information, answer the following questions :
(i) Write cost C(h) as a function in terms of h.
1
(ii) Find critical point.
(iiü) (a) Use second derivative test to find the value of h for which cost
of constructing the pool is minimum. What is the minimum
cost of construction of the pool ? 2

OR

65/C/1 Page 21 P.T.O.


(iii) (b) Use first derivative test to find the depth of the pool so that
x
cost of construction is minimum. Also, find relation between
2
and h for minimum cost.

Case Study - 3

varieties of
In an agricultural institute, scientists do experiments with
plants
seeds to grow them in different environments to produce healthy
and get more yield.
fast after
A scientist observed that a particular seed grew very
germination. He had recorded growth of plant since germination and he
said that its growth can be defined by the function
fx) = xß 4x' + 15x + 2, 0<x< 10
3

where x is the number of days the plant is exposed to sunlight.

On the basis of the above information, answer the following questions :


() What are the critical points of the function f(x) ? 2

(ii) Using second derivative test, find the minimum value of the
function. 2

65/C/1 Page 23
CBSE Mathematics Class 12 Sample Paper 8 Page 1

Sample Paper 8
Mathematics (Code-041)
Class XII Session 2023-24
Time Allowed: 3 Hours Maximum Marks : 80
General Instructions :
1. This Question Paper contains - five sections A, B, C, D and E. Each section is compulsory. However, there are
internal choices in some questions.
2. Section A has 18 MCQs and 02 Assertion-Reason based questions of 1 mark each.
3. Section B has 5 Very Short Answer (VSA)-type questions of 2 marks each.
4. Section C has 6 Short Answer (SA)-type questions of 3 marks each.
5. Section D has 4 Long Answer (LA)-type questions of 5 marks each.
6. Section E has 3 source based/case based/passage based/integrated units of assessment (4 marks each) with sub-
parts.

Section - A
Multiple Choice Questions each question carries 1 mark.

1. The order of the differential equation of all circles of radius a is


(a) 2 (b) 3

(c) 4 (d) 1

2. The derivative of log x is


(a) 1, x 2 0 (b) 1 , x ! 0
x x
(c) 1, x ! 0 (d) None of these
x

3. If A = {1, 2, 3, 4} and B = {1, 2, 3, 4, 5, 6} are two sets and function f : A " B is defined by f (x) = x + 2 , 6x d A,
then the function f is
(a) bijective (b) onto

(c) one-one (d) many-one

4. cos−1 9cos a− π kC is equal to


3

(a) −π (b) π
3 3
(c) 2π (d) 5π
3 3

1 x −3 1 1 0
If A = > 2
x 4yH
, B=>
1 0H
5. and adj (A) + B = > H , then the values of x and y are
0 1
(a) 1, 1 (b) ! 1, 1

(c) 1, 0 (d) None of these

Install NODIA App to See the Solutions.


Click Here To Install
Page 2 Sample Paper 8 CBSE Mathematics Class 12

ax + 3, x # 2
6. If f (x) = * 2 , then the values of a for which f is continuous for all x are
a x − 1, x 2 2

(a) 1 and - 2 (b) 1 and 2

(c) - 1 and 2 (d) - 1 and - 2

7. The function f ^x h = x2 , for all real x , is


(a) decreasing (b) increasing

(c) neither decreasing nor increasing (d) none of the above

8. The function f ^x h = x3 has a


(a) local minima at x = 0 (b) local maxima at x = 0

(c) point of inflexion at x = 0 (d) none of the above

9. # 1+x+ x + x2 dx is equal to
x + 1+x

(a) 1 1+x +C (b) 2 (1 + x) 3/2 + C


2 3

(c) 1+x +C (d) 2 (1 + x) 3/2 + C

10. #0
π/2 sin x − cos x dx is equal to
1 + sin x cos x

(a) 0 (b) π
4
(c) π (d) π
2

11. Find the area of a curve xy = 4 , bounded by the lines x = 1 and x = 3 and X -axis.
(a) log 12 (b) log 64

(c) log 81 (d) log 27

12. Solution of (x + 2y3) dy = y dx is

(a) x = y3 + cy (b) x + y3 = cy

(c) y2 − x = cy (d) none of these

dy
13. Integrating factor of differential equation cos x + y sin x = 1
dx
(a) cos x (b) tan x

(c) sec x (d) sin x

Install NODIA App to See the Solutions.


Click Here To Install
CBSE Mathematics Class 12 Sample Paper 8 Page 3

14. Two vectors av and bv are parallel and have same magnitude, then
(a) they have the same direction (b) they are equal

(c) they are not equal (d) they may or may not be equal

15. If the position vectors of the vertices A, B, C of a triangle ABC are 7tj + 10kt, − it + 6tj + 6kt and − 4it + 9tj + 6kt
respectively, then triangle is
(a) equilateral (b) isosceles

(c) scalene (d) right angled and isosceles also

16. If α, β, γ are the angles which a half ray makes with the positive directions of the axes, then sin2 α + sin2 β + sin2 γ
is equal to
(a) 2 (b) 1

(c) 0 (d) - 1

17. If P ^A j B h = 0.83 , P ^Ah = 0.3 and P ^B h = 0.6 , then the events will be
(a) dependent (b) independent

(c) cannot say anything (d) None of the above

18. Two dice are thrown n times in succession. The probability of obtaining a doublet six atleast once is

b 36 l (b) 1 - b 35 l
(a) 1 n n

36

b 12 l
(c) 1 n (d) None of these

19. Assertion: Let A = {− 1, 1, 2, 3} and B = {1, 4, 9} where f : A " B given by f (x) = x2 , then f is a many-one
function.
Reason: If x1 ! x2 & f (x1) ! f (x2), for every x1, x2d domain then f is one-one or else many
(a) Assertion is true, Reason is true; Reason is a correct explanation for Assertion.

(b) Assertion is true, Reason is true; Reason is not a correct explanation for Assertion.

(c) Assertion is true; Reason is false.

(d) Assertion is false; Reason is true.

R − 2 2 VW
1 S1
20. Assertion: If A = S− 2 1 2 W , then (AT ) A = I
3S
S− 2 − 2 − 1WW
T X
Reason: For any square matrix A, (AT )T = A
(a) Assertion is true, Reason is true; Reason is a correct explanation for Assertion.

(b) Assertion is true, Reason is true; Reason is not a correct explanation for Assertion.

(c) Assertion is true; Reason is false.

(d) Assertion is false; Reason is true.

Install NODIA App to See the Solutions.


Click Here To Install
Page 4 Sample Paper 8 CBSE Mathematics Class 12

Section - B
This section comprises of very short answer type-questions (VSA) of 2 marks each.

21. Let R is the equivalence relation in the set A = {0, 1, 2, 3, 4, 5} given by R = {(a, b): 2 divides (a - b)} . Write the
equivalence class [0].

22. Find # sin6 x dx .


cos8 x
O
Evaluate # dx .
sin2 x cos2 x

23. Write the direction ratios of the vector 3av + 2bv , where av = it + tj − 2kt and bv = 2it − 4tj + 5kt.
 O
Find the unit vector in the direction of the sum of vectors 2it + 3tj − kt and 4it − 3tj + 2kt.

24. Find the vector equation of the line which passes through the point (3, 4, 5) and is parallel to the vector 2it + 2tj − 3kt.

25. Prove that if E and F are independent events, then the events E and F' are also independent.

Section - C
This section comprises of short answer-type questions (LA) of 3 marks each.

26. Write the value of cos-1 b 1 l - 2 sin-1 b- 1 l .


2 2

1 1 1
27. Find the maximum value of 1 1 + sin θ 1
1 1 1 + cos θ

Find the value of k , so that the following functions is continuous at x = 2 . f (x) = *


x3 − x2 − 16x + 20 , x ! 2
28. (x − 2) 2
k, x=2

 O

dy
Find at x = 1, y = π if sin2 y + cos xy = K .
dx 4

29. The volume of a sphere is increasing at the rate of 8 cm3/s. Find the rate of which its surface area is increasing
when the radius of the sphere is 12 cm.
 O

Show that the function f (x) = x3 − 3x2 + 6x − 100 is increasing on R .

Install NODIA App to See the Solutions.


Click Here To Install
CBSE Mathematics Class 12 Sample Paper 8 Page 5

30. Evaluate # 2 cos x dx .


3 sin2 x

31. Write a unit vector perpendicular to both the vectors av = it + tj + kt and bv = it + tj .

Section - D
This section comprises of long answer-type questions (LA) of 5 marks each.

32. Show that the line lines


rv = (it + tj − kt) + λ (3it − tj ) and
rv = (4it − kt) + µ (2it + 3kt) intersect. Also, find their point of intersection.

33. Find the value of # (1 + sincos x


x) (2 + sin x)
dx .

O
Prove that #0
a
f (x) dx = #0
a
f (a − x) dx . and hence evaluate #0
π/2 x dx .
sin x + cos x

Show that the differential equation :x sin2 a k − yD dx + x dy = 0 is homogeneous.


y
34.
x
Find the particular solution of this differential equation, given that y = π , when x = 1.
4

35. Maximise Z = 5x + 3y subject to the constraints: 3x + 5y # 15 ; 5x + 2y # 10 , x $ 0 , y $ 0 .


 O
Minimize Z = 3x + 5y such that x + 3y $ 3 , x + y $ 2 , x , y $ 0 .

Section - E
Case study based questions are compulsory.

36. A market analysis is a quantitative and qualitative assessment of a market. It looks into the size of the market
both in volume and in value, the various customer segments and buying patterns, the competition, and the
economic environment in terms of barriers to entry and regulation.

Based on the past marketing trends and his own experience, marketing expert suggested to the concerned the
segments of market for their products as follows:
The first segment consisted of lower income class, the second segment that of middle income and the third segment

Install NODIA App to See the Solutions.


Click Here To Install
Page 6 Sample Paper 8 CBSE Mathematics Class 12

that of high income. The data based on the income of the consumers was readily available. During a particular
month in particular year, the agent reported that for three products of the company the following were the sales:
There were 200 customers who bought all the three products, 240 customers who bought I and III, 60 customers
only products II and II and 80 customers only products only III regardless of the market segmentation groups.
Based on the market segmentation analysis, for product I, the percentage for the income groups are given as (40%,
20% and 40%), for product II (30%, 20% and 50%), for product III (10%, 50% and 40%).
(i) Taking the suitable variable form the system of equation that represent given problem.
(ii) Using matrix method, find out the number of persons in the lower income, middle income and higher income
class in the region referred.

37. In apparels industries retailers have an interesting conundrum facing them. On one hand, consumers are more
drawn to hot promotional deals than ever before. The result of this is that they sell more units (of product) for
less money, and this adversely impacts comp store sales.

Arvind Fashions knows that the it can sell 1000 shirts when the price is ` 400 per shirt and it can sell 1500 shirts
when the price is ` 200 a shirt. Determine
(i) the price function
(ii) the revenue function
(iii) the marginal revenue function.

38. The U.S. Constitution directs the government to conduct a census of the population every 10 years. Population
totals are used to allocate congressional seats, electoral votes, and funding for many government programs. The
U.S. Census Bureau also compiles information related to income and poverty, living arrangements for children,
and marital status. The following joint probability table lists the probabilities corresponding to marital status and
sex of persons 18 years and over.14

Sex Marital Status


(R) (N) (W) (D)
(M) 0.282 0.147 0.013 0.043
(F) 0.284 0.121 0.050 0.060
R & Married
N & Never married
W & Widowed
D & Discovered or separated
M & Male
F & Male
Suppose a U.S. resident 18 years or older is selected at random.
(i) Find the probability that the person is female and widowed.
(ii) Suppose the person is male. What is the probability that he was never married?
(iii) Suppose the person is married. What is the probability that the person is female?

 ******

Install NODIA App to See the Solutions.


Click Here To Install
KENDRIYA VIDYALAYA SANGATHAN
Class XII
Session 2023-24
Mathematics (Code-041)
Time Allowed: 3 Hours Maximum Marks: 80

General Instructions :
1. This Question paper contains - five sections A, B, C, D and E. Each section is
compulsory. However, there are internal choices in some questions.
2. Section A has 18 MCQ’s and 02 Assertion-Reason based questions of 1 mark each.
3. Section B has 5 Very Short Answer (VSA)-type questions of 2 marks each.
4. Section C has 6 Short Answer (SA)-type questions of 3 marks each.
5. Section D has 4 Long Answer (LA)-type questions of 5 marks each.
6. Section E has 3 source based/case based/passage based/integrated units of
assessment(4 marks each) with sub parts.

SECTION A
Multiple Choice Questions of 1 mark each
1 0 a  3 1
 
If 2 0  1 is a skew-symmetric matrix, then the values of a and b,
b 1 0 

respectively, are
(a) -2, -3 (b) -2, 3 (c) 2, -3 (d) -3, 3
2 8 0 1
If A(𝑎𝑑𝑗𝐴)=[ ], then |𝑎𝑑𝑗𝐴| is:
0 8
(a) 8 (b) 64 (c) 512 (d) 16
3 2 −1 3 1
The matrix [ 𝜆 0 7] is not invertible for
−1 0 7
(a) λ = –1 (b) λ = 0 (c) λ = 1 (d) λ = R – {1}
4 𝑘𝑥 + 1, 𝑥 ≤ 5 1
If 𝑓(𝑥 ) = { is continuous at x = 5, then value of k is
3𝑥 − 5, 𝑥 > 5
6 7 9 5
(a) (b) (c) 5 (d) 9
7 6

5 1
∫ 𝑒 𝑥 (𝑡𝑎𝑛𝑥 + log 𝑠𝑒𝑐𝑥)𝑑𝑥 =

(a) 𝑒 𝑥 𝑡𝑎𝑛𝑥 + C (b) 𝑒 𝑥 𝑙𝑜𝑔𝑠𝑒𝑐𝑥 + C


(c) 𝑒 𝑥 𝑠𝑒𝑐 𝑥 + C (d) none of these

6 𝑑2 𝑦 𝑑𝑦 1
The sum of the order and degree of the differential equation + 3 𝑑𝑥 = 𝑒 𝑥 is
𝑑𝑥 2

1
(a) 2 (b) 3 (c) 5 (d) 4
7 The feasible region for LPP is shown shaded in the figure. Let Z = 3 x – 4 y be 1
the objective function, then the maximum value of Z is:

(a) 12 (b) 8 (c) 0 (d) –18


8 𝐼𝑓 |𝑎⃗ + 𝑏̅| = |𝑎⃗ − 𝑏̅| , then the angle between 𝑎
⃗⃗⃗⃗ and 𝑏⃗⃗ is 1

(a) 90° (b)60° (c)110° (d)180°


9 𝑠𝑖𝑛 2 𝑥−𝑐𝑜𝑠 2𝑥 1
∫ 𝑠𝑖𝑛 2 𝑥𝑐𝑜𝑠 2𝑥
dx is equal to:

(a) tan 𝑥+cot 𝑥 +c (b) tan 𝑥+ cosec𝑥 +c


(c) -tan 𝑥+cot 𝑥 +c (d) tan 𝑥+ sec 𝑥 + c
10 If A is a square matrix such that A2 = A, then the simplified value of (I – A)3 + A 1
is equal to
(a) A (b) A2 (c) I (d) A3
11 Consider the following system of linear inequalities: 1
2x + y ≤ 10 , x + 3y ≤ 15, x, y ≥ 0
If the corner points of the feasible region are (0, 0), (5, 0), (3, 4) and (0, 5).
Let Z = p x +q y, where p, q > 0. Condition on p and q, so that the maximum
value of Z occurs at both (3, 4) and (0, 5) is:
(a) p = q (b) p =2 q (c) p = 3q (d) q = 3p
12 3x 7 8 7 1
If | |=| |, the possible value(s) of x are
−2 4 6 4
(a) 2 (b) 3 (c) 2, 3 (d) -2
13 If A is a square matrix of order 3 and |A| = 5, then |𝑎𝑑𝑗𝐴| = 1
1
(a) 5 (b) 25 (c) 125 (d) 5

14 Let A and B are two events such that P(A) = 0.6, P(B) = 0.2 and P(A/B) = 0.5, 1
then P(A'/B') =
1 3 3 6
(a) 10 (b) 10 (c) 8 (d) 7

15 𝑑𝑦 1
The integrating factor of the differential equation 𝑑𝑥 + 𝑦𝑡𝑎𝑛𝑥 − 𝑠𝑒𝑐𝑥 = 0

2
𝑡𝑎𝑛 2 𝑥
(a) tan 𝑥 (b) 𝑠𝑒𝑐 2 𝑥 (c) sec 𝑥 (d) 2

16 Derivative of 𝑒 𝑙𝑜𝑔𝑥 𝑖𝑠 1
(a) 𝑒 𝑙𝑜𝑔𝑥 (b) log 𝑥 (c) 0 (d) 1
17 A man is watching an aero plane which is at the coordinate point A (4, -1, 3), 1
assuming the man is at O (0, 0, 0). At the same time, he saw a bird at the coordinate
⃗⃗⃗⃗⃗⃗⃗ is
point B (2, 0, 4). The unit vector along 𝐴𝐵
2 1 1 −2 1 1
(a) 𝑖̂ + 6 𝑗̂ + 6 𝑘̂ (b) 𝑖̂ + 6 𝑗̂ + 6 𝑘̂
6 √6
−2 1 1 4 2 3
(c) 𝑖̂ + 𝑗̂ + 𝑘̂ (d) 𝑖̂ + 𝑗̂ + 𝑘̂
√6 √6 √6 √6 √6 √6

18 P is a point on the line joining the points (0,5, −2) and 𝐵(3, −1,2). If the x- 1
coordinate of P is 6, then its z-coordinate is
(a) 10 (b) 6 (c) -6 (d) -10
ASSERTION-REASON BASED QUESTIONS
In the following questions, a statement of assertion (A) is followed by a statement
of Reason (R). Choose the correct answer out of the following choices.
(a) Both A and R are true and R is the correct explanation of A.
(b) Both A and R are true but R is not the correct explanation of A.
(c) A is true but R is false.
(d) A is false but R is true.

19 2𝜋 1
Assertion (A): 𝑡𝑎𝑛 −1 (tan )
3
𝜋 𝜋 𝜋 𝜋
= 𝑡𝑎𝑛−1 (tan( 𝜋 − )) = 𝑡𝑎𝑛−1 (− tan ) = − 𝑡𝑎𝑛−1 (tan ) = −
3 3 3 3
2𝜋 2𝜋
Reason (R): 𝑡𝑎𝑛−1 (tan )=
3 3

20 Assertion (A): The triangle ABC, with vertices A(1,2,3), B(2,-1,4) and C(4,5,-1) 1
√274
has the area = square units.
2
1
Reason(R): The area of the triangle ABC is ⃗⃗⃗⃗⃗⃗ × 𝐴𝐶
|𝐴𝐵 ⃗⃗⃗⃗⃗⃗ |
2

SECTION B
Very Short Answer Type-Questions (VSA) of 2 marks each
21 Find the domain of the function defined by 𝑓(𝑥) = 𝑠𝑖𝑛−1 √𝑥 − 1 2
OR
cos 𝑥−sin 𝑥 𝜋 3𝜋
Write in the simplest form tan−1 cos 𝑥+sin 𝑥 , − 4 < 𝑥 < .
4

3
22 A man 1.6 m tall walks at the rate of 0.3 m/sec away from a street light that is 4 2
m above the ground. At what rate is the tip of his shadow moving? At what rate
is his shadow lengthening?
23 Using direction ratios show that the points A(2, 3, 4), B(-1, -2, 1) and C(5, 8, 7) 2
are collinear.
OR
If a⃗⃗ = 2î + 2ĵ + 3k̂ , ⃗⃗
b = −î + 2ĵ + k̂ and c⃗ = 3î + ĵ are such that a⃗⃗ + λb
⃗⃗ is

perpendicular to c⃗, then find the value of λ.


24 dy cos2 (a+y) 2
If cos y = x cos (a + y), with cos a ≠ ±1 prove that =
dx sin a

25 Show that the line through points (4, 7, 8) and(2, 3, 4) is parallel to the line 2
through the points (−1, −2, 1) and (1, 2, 5).
SECTION C
Short Answer Type Questions (SA) of 3 marks each
26 (x2 +1)(x2 +4) 3
Evaluate: ∫ (x2 +3)(x2 −5) dx

27 Three friends go for coffee. They decide who will pay the bill, by each tossing a 3
coin and then letting the “odd person” pay. There is no odd person if all three
tosses produce the same result. If there is no odd person in the first round, they
make a second round of tosses and they continue to do so until there is an odd
person. What is the probability that exactly three rounds of tosses are made?
OR
Find the mean number of defective items in a sample of two items drawn one-
by-one without replacement from an urn containing 6 items, which include 2
defective items. Assume that the items are identical in shape and size .
28 𝑥 2+1 3
𝐸𝑣𝑎𝑙𝑢𝑎𝑡𝑒: ∫ (𝑥−1)2 (𝑥+3)dx

OR
Evaluate ∫ 𝑥 sin−1 𝑥 𝑑𝑥
29 dy 3
Show that the differential equation (x − y) dx = x + 2y is homogeneous and

solve it.
OR
Find the general solution of the differential equation
dy
x dx + y − x + xy cotx = 0 (x ≠ 0)

30 Solve the linear programming problem graphically: 3


Maximize: Z = 8000 x + 12000 y
Subject to the constraints:
3x + 4y ≤ 60, x + 3y ≤ 30, x ≥ 0, y ≥ 0.

4
31 𝑥 3 + 𝑥 +1 3
𝐸𝑣𝑎𝑙𝑢𝑎𝑡𝑒: ∫ (𝑥−1)2
dx

SECTION D
Long Answer-Type Questions (LA) of 5 marks each

32 Make a rough sketch of the region {(𝑥, 𝑦): 0 ≤ 𝑦 ≤ 𝑥 2, 0 ≤ 𝑦 ≤ 𝑥, 0 ≤ 𝑥 ≤ 2} and 5


find the area of the region using integration.
33 Define the relation R in the set 𝑁 × 𝑁 as follows: 5
For (a, b), (c, d) ∈ 𝑁 × 𝑁, (a, b) R (c, d) iff ad = bc. Prove that R is an equivalence
relation in 𝑁 × 𝑁.
OR
Show that 𝑓: 𝑅 → 𝑅, given by 𝑓(𝑥 ) = 1 + 𝑥 2 is neither one-one nor onto.
34 Find the shortest distance between two lines whose vector equations are 5
r⃗ = (1 − t)î + (t − 2)ĵ + (3 − 2t)k̂
r⃗ = (s + 1)î + (2s − 1)ĵ − (2s + 1)k̂
OR
Find the equation of the line passing through the points A(0, 6, -9) and B(-
3, -6, 3). If D is the foot of perpendicular drawn from a point C(7, 4, -1) on the
line AB, then find coordinates of the point D and the equation of line CD.
35 2 −3 5 5
If A= [3 2 −4] find A-1.. Hence,
1 1 −2
solve the system of equations:
2x - 3y + 5z = 11, 3x + 2y - 4z = – 5 and x + y – 2z = −3
SECTION E
This section comprises of 3 case-study/passage-based questions of First two case study
questions have three sub-parts (i), (ii), (iii) of marks 1, 1, 2 respectively. The third case
study question has two sub-parts of 2 marks each.

36 Case-Study 1: 4
Read the following passage and answer the questions given below.
Three students Rohan, David and Leena are finding the derivative of the function
𝑓(𝑥 ) = (𝑥 2 − 5𝑥 + 8)(𝑥 3 + 7𝑥 + 9).
They followed three ways mentioned below:
(i) Rohan differentiated the given function by using product rule
(ii) David differentiated the given function by expanding the product to
obtain a single polynomial
(iii) Leena differentiated the given function by logarithmic differentiation.
Did they all give the same answer?

5
37 Case-Study 2: 4
Read the following passage and answer the questions given below.
A wire of length 28 m is to be cut into two pieces. One of the pieces is to be made
into a square and the other into a circle.

In order to find the lengths of the two pieces, so that the combined area be
minimum, two steps are followed:
(i) Find the area of the circle.
(ii) Find the area of the square.
(iii) What should be the lengths of the two pieces so that the combined area
of the circle and the square is minimum?
38 Case-Study 3: 4
Read the following passage and answer the questions given below.
Probabilities of solving a specific problem independently by Javed Ali and
1 1
Jashandeep Kaur are and respectively. If both try to solve the problem,
2 3

independently, then find the probability that


(i) The problem is solved
(ii) Exactly one of them solves the problem.

6
Marking Scheme
Q.NO. SECTION A-(Multiple choice questions 1-Mark each) MARKS

1 (b) 1

2 (a) 1

3 (a) 1

4 (c) 1

5 (b) 1

6 (b) 1

7 (c) 1

8 (a) 1

9 (a) 1

10 (c) 1

11 (d) 1

12 (d) 1

13 (b) 1

14 (c) 1

15 (c) 1

16 (d) 1

7
17 (c) 1

18 (b) 1

19 (c)

20 (a) 1

SECTION B (VSA questions of 2-Marks each)

21 f (x) = sin–1 √𝑥 − 1

-1≤ √𝑥 − 1 ≤ 1 ⟹ 0≤ √𝑥 − 1 ≤ 1 1

0≤ 𝑥 − 1 ≤ 1 ⟹ 1≤ 𝑥 ≤ 2, 𝑥 ∈ [1 , 2] 1

OR

1 − tan 𝑥 1/2
−1
tan
1 + tan 𝑥
1/2
𝜋
tan −tan 𝑥
tan−1 4
𝜋
1+tan tan 𝑥
4

𝜋 1/2
tan−1 tan ( 4 − 𝑥)
1/2
𝜋
-𝑥
4

22

2
Finding expression 𝑦 = 3 𝑥 1/2

Where 𝑥 = distance of man at any time t from street light.

𝑦 =length of shadow of man

Getting
𝑑𝑦
= 0.2m/sec 1/2
𝑑𝑡

At any time t the tip of his shadow is at a distance of (𝑥 + 𝑦)m from street light

8
The rate at which his shadow moving

𝑑𝑥 𝑑𝑦 ½
=( 𝑑𝑡 + 𝑑𝑡 )m/s=0.5m/s

The rate at which his shadow lengthening=

𝑑𝑦
m/s=0.2m/s
𝑑𝑡
1/2

23 Finding dr’s of AB i.e -3,-5,-3 1/2

Finding dr’s of BC i.e 6,10, 6 ½

−3 −5 −3 1
1
Showing = = =−2
6 10 6

Or

Finding 𝑎⃗ + 𝜆𝑏⃗⃗ = (2 − 𝜆)𝑖̂ +(2 + 2𝜆)𝑗̂ + (3 + 𝜆)𝑘̂


½

Applying (𝑎⃗ + 𝜆𝑏⃗⃗). 𝑐⃗ =0


½

Getting 𝜆 = 8 1

cos 𝑦
24 Writing 𝑥 = cos(𝑎+𝑦) 1/2

1
sin(𝑎+𝑦) cos 𝑦−cos(𝑎+𝑦) sin 𝑦 𝑑𝑦
Getting, 1 = 𝑐𝑜𝑠 2(𝑎+𝑦) 𝑑𝑥

𝑑𝑦 𝑐𝑜𝑠 2(𝑎+𝑦)
Proving 𝑑𝑥 = ½
sin 𝑎

25 Let AB be the line through the points, (4, 7, 8) and (2, 3, 4), and CD be the line
through the points, (−1, −2, 1) and (1, 2, 5).
½
The directions ratios, a1, b1, c1, of AB are (2 − 4), (3 − 7), and (4 − 8) i.e., −2, −4,
and −4.

The direction ratios, a2, b2, c2, of CD are (1 − (−1)), (2 − (−2)), and (5 − 1) i.e., 2, ½
4, and 4.
½

9
SECTION-C(Short Answer Questions of 3-marks each)

26 (𝑥 2+1)(𝑥 2 +4) 7𝑥 2 +19 ½


(𝑥 2 +3)(𝑥 2 −5)
=1+ 𝑥 4−2𝑥 2 −15

½
7𝑥 2+19 𝐴𝑥+𝐵 𝐶𝑥+𝐷
Let (𝑥 2+3)(𝑥 2−5) = (𝑥 2+3) + 𝑥 2 −5

1 27
Finding value of A= 0,B= 4 ,C= 0,D= 4
1

(𝑥 2+1)(𝑥 2+4) 1 𝑥 27 𝑥−√5


∫ (𝑥 2+3)(𝑥 2−5) 𝑑𝑥 =𝑥 + 4 tan−1 ( 3)+8√5 log 𝑥+√5 + c 1
√3 √

27 P(not obtaining an odd person in a single round) = P(All three of them throw tails or All
1 1 1 1
three of them throw heads) = 2 × 2 × 2 × 2 = 4 1
12
(obtaining an odd person in a single round) = 1 − P(not obtaining an odd person in a
1 3 1/2
single round) = 1 − =
4 4

The required probability =

P(‘In first round there is no odd person’ and ‘In second round there is no odd person’
1 1 3 3 1
and ‘In third round there is an odd person’) = 4 x 4 x 4 = 64

Or

Let X= number of defective items

X assumes value 0,1,2

½
Finding probability distribution

X 0 1 2
1
P(X) 2 8 1 12
5 15 15

2 8 1 10
Finding mean = ∑ 𝑝ixi= 0 x 5 + 1x 15 + 2 x15 = 15

10
2
=3
1

28 𝑥 2 +1 𝐴 𝐵 𝑐 ½
(𝑥−1) 2(𝑥+3)
= 𝑥−1 + +𝑥+3
(𝑥−1)2

1
A =3/8 ,B =1/2 ,c = 5/8

On substituting and finding integration


1
12
3 1 5
= 8log |𝑥 − 1| - 2(𝑥−1) + 8log |𝑥 + 3|+c

Or

𝑑
I = sin−1 𝑥 ∫ 𝑥 𝑑𝑥 − ∫ {(𝑑𝑥 sin−1 𝑥) ∫ 𝑥 𝑑𝑥} 𝑑𝑥
½
𝑥 2 sin −1 𝑥 1 𝑥2
= − ∫ 1
2 2 √1−𝑥 2

𝑥 2 sin−1 𝑥 1 𝑑𝑥 1
+ 2 {∫ √1 − 𝑥 2 𝑑𝑥 − ∫ √1−𝑥 2 }
2
1/2
1 2 −1 𝑥
(2𝑥 − 1) sin 𝑥 + 4 √1 − 𝑥2 +C
4

29 Showing homogeneous ½

Substituting y= vx

1−𝑣 𝑑𝑥 1
And getting 𝑣 2+𝑣+1dv = 𝑥

Solving and getting solution


1
1 𝑥 2 +𝑥𝑦+𝑦 2 −1 2𝑦+𝑥 12
− log + √3 tan ( ) =log 𝑥 + c
2 𝑥2 √3𝑥

Or

Writing in the form

𝑑𝑦 1
+ (𝑥 + cot 𝑥)y=1 ½
𝑑𝑥

1
I.F= 𝑒 ∫(𝑥+cot 𝑥)𝑑𝑥 = xsin 𝑥
½

11
𝑦(𝑥 sin 𝑥 )=∫ 𝑥 sin 𝑥𝑑𝑥 + c ½
1
Finding solution xysin 𝑥 = -xcos 𝑥 + sin 𝑥 + c 12

30 Plotting correct figure

1
12

Shading the feasible region 1

Finding the maximum value of Z ½

Corner Point Corresponding value of Z


(0, 0) 0
(20, 0) 1,60,000
(12, 6) 1,68,000(Max.)
(0, 10) 1,20,000
31 Let 𝑥 = 𝑡 − 1 1/2

Diff wrt.x dx =dt

(𝑡+1)3 +𝑡+1+1
∫ dt
𝑡2
½
𝑡 3 +3𝑡 2 +4𝑡+3
∫ dt 1
𝑡2

½
4 3
∫ (𝑡 + 3 + 𝑡 + 𝑡 2 )dt

(𝑥−1) 2 3
+3(𝑥 − 1)+4log(𝑥 − 1) − +c ½
2 𝑥−1

SECTION-D(Long Answer type (LA) of 5 –marks each)

12
32. The points of intersection of parabola 𝑦 = 𝑥 2 and the line 𝑦 = 𝑥 are (0,0) and ½
(1,1)

For correct sketching


1

1 2
Required area =∫0 𝑥 2 dx + ∫1 𝑥𝑑𝑥
2
𝑥3 1 𝑥2 2
[ ] +[ ]
3 0 2 1
1
11 12
Finding correct answer = sq.unit
6

33 Showing ab=ba ,hence R is symmetric 1

1
Showing cb= da⇒ (𝑐, 𝑑 )R(𝑎, 𝑏),hence R is symmetric

1
(𝑎, 𝑏)R(𝑐, 𝑑 )⇒ad=bc, (𝑐, 𝑑 )R(𝑒, 𝑓)⇒cf=de
1
12
af =be⇒(𝑎, 𝑏)𝑅 (𝑒, 𝑓),hence R is transitive
½
R is reflexive,symmetric,and transitive nence R is equivalence relation

Or

Showing 𝑓 (𝑥 ) = 𝑓 (𝑦) but 𝑥 ≠ 𝑦 for any 𝑥, 𝑦 ∈ 𝑅


1

13
Hence not one-one 1

Showing , Range f= 1 + 𝑥 2 ≥ 1 ⩝𝑥 ∈𝑅 1

Hence not onto 1

1
Hence f(x) is neither one-one nor onto

34 Writing the equation of two lines in standard form

𝑟⃗ = 1𝑖̂ − 2𝑗̂ + 3𝑘̂ + 𝑡(−𝑖̂ + 𝑗̂ − 2𝑘̂) 1


𝑟⃗ = 𝑖̂ − 𝑗̂ − 𝑘̂ + 𝑠(𝑖̂ + 2𝑗̂ − 2𝑘̂)
1
𝑎1
⃗⃗⃗⃗⃗=1𝑖̂ − 2𝑗̂ + 3𝑘̂ , ⃗⃗⃗⃗⃗=
𝑎2 𝑖̂ − 𝑗̂ − 𝑘̂ ⃗⃗⃗⃗
𝑏1 = −𝑖̂ + 𝑗̂ − 2𝑘̂ , ⃗⃗⃗⃗⃗
𝑏2 = 𝑖̂ + 2𝑗̂ − 2𝑘̂
½
𝑎1 = 𝑗̂ − 4𝑘̂
𝑎2 - ⃗⃗⃗⃗⃗
⃗⃗⃗⃗⃗

⃗⃗⃗⃗ 1
𝑏1 𝑋 ⃗⃗⃗⃗⃗
𝑏2 = 2𝑖̂ − 4𝑗̂ − 3𝑘̂
⃗⃗⃗⃗⃗⃗⃗⃗ 𝑎1 ⃗⃗⃗⃗⃗
(𝑎2 − ⃗⃗⃗⃗⃗⃗)(𝑏 1𝑋 ⃗⃗⃗⃗⃗
𝑏2 ) 1
d=| ⃗⃗⃗⃗⃗ ⃗⃗⃗⃗⃗
|
|𝑏 1 𝑋 𝑏 2 |

8
½
d=
√29

or

Finding equation of line AB

𝑥 𝑦−6 𝑧+9
1
= =
−3 −12 12
½
Point D on line,(−3𝑘, −12𝑘 + 6,12𝑘 − 9)

Dr’s of line CD
½
-3k-7,-12k+2,12k-8

Line CD perpendicular to AB
1
-3(-3k-7) + (-12)(-12k+2)+12(12k-8) =0

1
Finding k= 3
1

Finding point D(-1,2,-5)

𝑥−7 𝑦−4 𝑧+1


Finding equation of line CD , = = 1
4 1 2

14
35 Here|𝐴| = -1 , A is a non singular matrix 1

0 −1 2 2
Adj A = [2 −9 23]
1 −5 13

0 1 −2 1
-1
A = [−2 9 −23]
−1 5 −13

AX =B ,x = 1, y= 2 and z = 3
1

SECTION – E (Case Studies/Passage based questions of 4 Marks each)

36 (i)

(ii)

15
1

(iii)

Taking logarithm on both the sides, we obtain

Differentiating both sides with respect to x, we obtain

16
From the above three observations, it can be concluded that all the results

of are same.

37 (i) Area of circle =

196𝜋 1
(𝜋+4)2
sq unit

112 2
(ii) Area of square = (𝜋+4) sq units.
1
28𝜋 112
(iii) Length of circle= 𝜋+4 ,Length of square = 𝜋+4
2

38 Let A =Probability that Javed Ali solved the problem

B= Probability that Jashandeep kaur Ali solved the problem

(i) Probability that the problem is solved = P (A ∪ B)

= P (A) + P (B) − P (AB)

(ii) 2

Probability that exactly one of them solves the problem is given


by,

17
**************************************************************************

18
SAMPLE PAPER

PART A

Section – I
Questions from 1-16 carry 1 mark each.
2 4 2𝑥 4
Q.1 Q.1 Find the values of 𝑥 , if | | = | |.
5 1 6 𝑥
Q.2 If A is a 3x3 invertible matrix, then what will be the value of k , if det(𝐴−1 ) = ( det 𝐴)𝑘 ?
Q.3 A matrix A of order 3x3 has determinant 6. What is the value of |3𝐴|
7𝜋
Q.4 Evaluate 𝑐𝑜𝑠 −1 (cos )
6

Q.5 Find the relation between a and b so that f(x) = ax + 1 for all x ≤ 3
bx + 3 for x < 3 is continuous at x = 3.

Q.6 Find the value of k for which


1−𝑐𝑜𝑠4𝑥
f (x) = { ,if x≠ 0
8𝑥 2

k , if x = 0

sec2 x
Q.7 Evaluate  dx
3  tan x
Q.8 Evaluate  e x  tan x  1 sec x dx

𝑑𝑦
Q.9. Find the integrating factor of the differential equation (x logx) 𝑑𝑥 + y = 2log x

Q.10 The number of arbitrary constants in the general solution of differential equation of fourth
order is ?
̂ − 6𝑘̂ and of magnitude 5.
Q.11 Find the vector along the direction of the vector 3 𝑖̂ − 2𝑗

Q.12 Find the scalar product of the vectors 𝑖̂ − 3𝑗̂ + 5𝑘̂ and 𝑖̂ + 𝑗̂ + 2𝑘̂
     
Q.13. If a  b  a  b , Show that a  b .

Q.14 A line makes angles 45 ° , 60° and acute angle 𝜃 with axes. Find 𝜃.
Q,15 In a LPP, the objective function is always
A. Linear B. Quadratic C. Cubic D. Biquadratic
Q.15 The corner points of the bounded feasible region of a LPP are A(0,50), B(20, 40), C(50, 100)
and D(0, 200) and the objective function is Z = x + 2y. Then the maximum value is
A. 100 B. 400 C. 250 D. 450

Q.16 If A and B are independent events the P(A∩B)= …


Q.17 If area of a triangle is 35 square units with vertices (2,-6),(5,4)and (k,4),then k is
(a) 12 (b) -2 (c) -12,-2 (d) 12,-2
Q.18 If A is any square matrix of order n, then A(adjA) is equal to
(a) |𝐴|I b) I (c) 0 (d) none of these
Assertion – Reason Based Question
In the following question , a statement of Assertion (A) is followed by a statement of Reason
(R) . Choose the correct answer out of the following choices
(a) Both (A) and (R) are true and (R) is the correct explanation of the (A)
(b) Both (A) and (R) are true and (R) is not the correct explanation of the (A)
(c) (A) is true and (R) is false
(d) (A) is false (R) is true

Q.19 Assertion (A) : The relation f : {1,2,3,4} → {x,y,z,p} defined by f = {(1,x) , (2,y) (3,z)}
is a bijective function.
Reason (R) : The function f : {1,2.3} → {x,y,z,p } such that f = {(1,x) , (2,y) (3,z)} is one- one
𝑑
Q.20 Let f(x) be a polynomial function of degree 6 such that 𝑑𝑥 ( f(x)) = (𝑥 − 1)3 (𝑥 − 3)2 ,
then Assertion (A) : f(x) has a minimum at x=1
𝑑
Reason (R) : When ( f(x)) < 0, ∀ x ∈ (a+h , a) ; where ‘h’ is an infinitesimally small
𝑑𝑥
positive quantity , then f(x) has a minimum at x=a ,provided f(x) is continuous at x=a.

SECTION B (2marks each)


Q.21 Check whether the relation R defined in the set {1, 2, 3, 4, 5, 6} as R = {(a, b) : b = a + 1} is
reflexive, symmetric or transitive.
𝑑2 𝑦 𝜋
Q.22 If y = a(1 + cos𝜃) and x = a (𝜃 − 𝑠𝑖𝑛𝜃) , find at 𝜃 = .
𝑑𝑥 2 2

Q.23 A particle moves along the curve 6y = x 3 +2. Find the points on the curve at which the y-
coordinate is changing 8 times as fast as the x-coordinate.

Q.24 If adjacent sides of parallelogram are 3𝑖̂ − 𝑗̂ + 5𝑘̂ and 2𝑖̂ + 3𝑗̂ + 𝑘̂
find the unit vectors along the diagonals of parallelogram.
Q.25 . Show that the points A (2, -1, 3), B (3, -5, 1) and C (-1, 11, 9) are collinear using vectors.

SECTION C
𝑑𝑥
Q.26 𝐸𝑣𝑎𝑙𝑢𝑎𝑡𝑒 ∫
√3𝑥 2+5𝑥+7

cos 2𝑥−cos 2∝
Q.27 𝐸𝑣𝑎𝑙𝑢𝑎𝑡𝑒 ∫ 𝑑𝑥
cos 𝑥−𝑐𝑜𝑠∝

Q.28∫ √1 − cos 3𝑥 dx

Q.29 If y = (𝑡𝑎𝑛−1 𝑥)2 , show that


( 𝑥 2 + 1)2 𝑦2 + 2x (𝑥 2 + 1) 𝑦1 = 2
Q.30 A dealer in rural area wishes to purchase a number of sewing machines. He has only Rs
5,760 to invest and has space for at most 20 items for storage. An electronic sewing machine cost
him Rs 360 and a manually operated sewing machine Rs 240. He can sell an electronic sewing
machine at a profit of Rs 22 and a manually operated sewing machine at a profit of Rs 18.
Assuming that he can sells all the items that he can buy, how should he invest his money in order
to maximize his profit ? Make it as a LPP and solve it graphically.

Q.31 Assume that each born child is equally likely to be a boy or a girl. If a family has two children,
what is the conditional probability that both are girls given that (i) the youngest is a girl, (ii) at
least one is a girl?
SECTION D
Q.32 Show that the relation R in the set A = {x  Z : 0  x  12}, given by R = {(a, b): |a –b| is a
multiple of 4 }is an equivalence relation.
−4 4 4 1 −1 1
Q.33 Determine the product [−7 1 3 ] [1 −2 −2] and use it to solve the system of
5 −3 −1 2 1 3
equarions
X – y + z = 4 , x – 2y – 2z = 9 and 2x + y +3z =1.
Q.34 Find the area of the region included between the curves 4 y = x 2 and x – 4 y + 2 =0.
Q. 35Find the Cartesian equation of the plane passing through the points A(0,0,0) , B (3, -1, 2)
x  4 y  3 z 1
and parallel to line  
1 4 7
SECTION E
Q.36 Read the following passage and answer the following questions .

The monthly income of two sister Reshma and Ritam are in the ratio 3 : 4 and their monthly
expenditures are in the ratio 5 : 7 . Each sister saves Rs. 15000 per month ,

(i)I f monthly income of Reshma and Ritam are Rs. 3x and Rs, 4x and their monthly expenditure
are Rs. 5y and Rs. 7y respectively , then express information provided in problem in system of
linear equations.
(i) Express the system of linear equations in (i) in matrix form AX = B .
(ii) (a) Find 𝐴−1
(b) Find C = A2 – 2I
Q 37.An architect designs a building whose lift (elevator) is from outside of the building attached to
the walls. The floor (base) of the lift (elevator) is in semicircular shape .
The floor of the elevator (lift) whose circular edge is given by the equation 𝑥 2 + 𝑦 2 = 4 and the
straight edge (line) is given by the equation y = 0

(i) Find the point of intersection of the circular edge and straight line edge.
(ii) Find the length of each vertical strip of the region bounded by the given curves.
(iii) (a) Find the area of a vertical strip between given circular edge and straight edge .
(b) Find the area of a horizontal strip between given circular strip and straight
OR
(iv) Find the area of the region of the floor of the lift of the building (in square units)
(a)

FROM
SARITA DOLE
PGT MATHS
KV CRPF NAGPUR
Sample Question Paper
Class XII
Session 2023-24 Mathematics
(Code-041)

Time Allowed: 3 Hours Maximum Marks: 80

General Instructions :

1. This Question paper contains - five sections A, B, C, D and E. Each section is


compulsory. However, there are internal choices in some questions.
2. Section A has 18 MCQ’s and 02 Assertion-Reason based questions of 1 mark each.
3. Section B has 5 Very Short Answer (VSA)-type questions of 2 marks each.
4. Section C has 6 Short Answer (SA)-type questions of 3 marks each.
5. Section D has 4 Long Answer (LA)-type questions of 5 marks each.
6. Section E has 3 source based/case based/passage based/integrated units of
assessment (4 marks each) with sub parts.
--------------------------------------------------------------------------------------------------------------------------

SECTION A
(Multiple Choice Questions) Each
question carries 1 mark

Q1. If A is any square matrix, then which of the following is skew-symmetric?


(a) A + AT (b) A – AT (c) AAT (d) ATA
Q2. If A is square matrix of order 3 such that I A I = 5 , then the value of I 3 adj A I
(a) 45 (b) 375 (c) 675 (d) 3375
Q3. If |𝑎⃗ + 𝑏 ⃗⃗| = |𝑎⃗ − 𝑏⃗⃗| then the vectors a and b are adjacent sides of
(a) a rectangle (b) a square (c) a rhombus (d) None of these
Q4. The value of the derivative of |x – 1| + |x – 3| at x = 2 is :
(a) –2 (b) 0 (c) 2 (d) not defined
y
Q5. If y = log (log x), then the value of e dy/dx is :
1 1 1
(a) ey (b) (c) (d)
𝑥 𝑙𝑜𝑔𝑥 𝑥𝑙𝑜𝑔𝑥
Q6. The order of the differential equation of a family of curves represented by an equation containing four
arbitrary constants, will be
(a) 2 (b) 4 (c) 6 (d) None of these

Q7. L.P.P is a process of finding


(a) Maximum value of objective function (b) Minimum value of objective function
(c) Optimum value of objective function (d) None of these

Q8. If θ is the angle between any two vectors 𝑎⃗ and 𝑏⃗⃗ , then | 𝑎⃗ ⋅𝑏⃗⃗ | = | 𝑎⃗ ×𝑏⃗⃗ | when θ is equal to
(a) 0 (b) 𝜋/2 (c) 𝜋/4 (d) 𝜋

Q9. Value of

(a) 0 (b) 𝜋/2 (c) 𝜋/4 (d) 𝜋


Q10. The probability of obtaining an even prime number on each die, when a pair of dice is rolled is
(a) 0 (b) 1/3 (c) 1/12 (d)1/36
Q11. The general solution of the differential equation 𝑦𝑑𝑥 − 𝑥𝑑𝑦 = 0 𝑖𝑠
(a) 𝑥𝑦 = 𝐶 (b) 𝑥 = 𝐶𝑦2 (c) 𝑦 = 𝐶𝑥 (d) 𝑦 = 𝐶𝑥2
Q12. The number of all possible matrices of order 3 × 3 with each entry 0 or 1 is:
(a) 27 (b) 18 (c) 81 (d) 512

Q13. If two vectors 𝑎⃗ 𝑎𝑛𝑑 𝑏¯ are such that |𝑎⃗| = 2 , |𝑏


¯⃗ | = 3 𝑎𝑛𝑑 𝑎⃗. 𝑏¯ = 4, 𝑡ℎ𝑒𝑛 |𝑎⃗ − 2𝑏¯ | is equal to
(a) √2 (b) 2√6 (c) 24 (d) 2√2
Q14. Feasible region for an LPP is shown shaded in the following figure.

Minimum of Z = 4 x + 3 y occurs at the point.


(a) (0, 8) (b) (2, 5) (c) (4, 3) (d) (9, 0)

Q15. The lines in a space which are neither intersecting nor parallel, are called
(a) Concurrent lines (b) intersecting lines (c) skew lines (d) parallel lines

Q16. If A2 – A + I = O, then the inverse of A is


(a) I – A (b) A – I (c) A (d) A + I

Q17. A square matrix A = [aij]n × n is called a lower triangular matrix if aij = 0 for
(a) i = j (b) i < j (c) i > j (d) None of these
sin x
Q18. If y = (tanx) , then dy/dx is equal to
(a) sec x + cos x (b) sec x + log tan x (c) (tan x)sin x (d) None of these

ASSERTION-REASON BASED QUESTIONS


In the following questions, a statement of assertion (A) is followed by a statement ofReason (R).
Choose the correct answer out of the following choices.

(a) Both A and R are true and R is the correct explanation of A.


(b) Both A and R are true but R is not the correct explanation of A.
(c) A is true but R is false.
(d) A is false but R is true.
Q19. Assertion: The function f(x) = sin x does not possess inverse if x ∈ R.
Reason: The function f(x) = sin x is not one-one onto if x ∈ R.
Q20. Assertion : In ∆ABC , 𝐴𝐵 ⃗⃗⃗⃗⃗⃗ + ⃗⃗⃗⃗⃗⃗
𝐵𝐶 + ⃗⃗⃗⃗⃗⃗⃗⃗⃗⃗⃗
𝐶𝐴 = 0 .
⃗⃗⃗⃗⃗⃗ = 𝑎⃗ , 𝑂𝐵
Reason : If 𝑂𝐴 ⃗⃗⃗⃗⃗⃗ = 𝑏 ⃗⃗ , then 𝐴𝐵 ⃗⃗ (triangle law of addition)
⃗⃗⃗⃗⃗⃗ = 𝑎⃗ + 𝑏

SECTION B
This section comprises of very short answer type-questions (VSA) of 2 marks each
-1
Q21. Find the domain of sin (2x -1).
𝑶𝑹
Q22. Prove that the Greatest Integer Function f : R→R, given by f (x) = [x], is neither
one-one nor onto, where [x] denotes the greatest integer less than or equal to x.

Q23. A ladder 5 m long is leaning against a wall. The bottom of the ladder is pulled along the ground,
away from the wall, at the rate of 2 cm/s. How fast is its height on the wall decreasing when the foot of
the ladder is 4 m away from the wall?

^ 𝑎𝑛𝑑 𝑏¯ = 5𝚤̂ − 𝚥 + 𝜆𝑘^ , then find the value of 𝜆 so that the vectors
Q24. . If ⃗ = 𝚤̂ − 𝚥 + 7𝑘
𝑎⃗ + 𝑏¯ 𝑎⃗ − 𝑏¯ are orthogonal.
𝑶𝑹
Find the image of the point (2, –1, 4) in the YZ-plane

Q25. Find |𝑥⃗| if (𝑥⃗ − 𝑎⃗). (𝑥⃗ + 𝑎⃗) = 12, where 𝑎⃗ is a unit vector.
SECTION C
(This section comprises of short answer type questions (SA) of 3 marks each)

Q26. Find:

Q27. A black die and a red die are rolled together. Find the conditional probability of obtaining a sum
greater than 9 given that the black die resulted in a 5.
𝑶𝑹
Three cards are drawn successively, without replacement from a pack of 52 well shuffled cards.
What is the probability that first two cards are aces and the third card drawn is a king?
1
Q28. Evaluate ∫0 |5𝑥 − 3|
𝑶𝑹

Q29. Solve the differential equation: 𝑦𝑑𝑥 + (𝑥 − 𝑦2) = 0

OR
Solve the differential equation: 𝑥𝑑𝑦 − 𝑦𝑑𝑥 = √ 𝑥 2 + 𝑦2 dx

Q30. Find the maximum value of the objective function Z = 5x + 10 y subject to the constraints
x + 2y ≤ 120, x + y ≥ 60, x – 2y ≥ 0, x ≥ 0, y ≥ 0.

Q31. Find

SECTION D
(This section comprises of long answer-type questions (LA) of 5 marks each)

Q32. Find the area lying in the first quadrant and bounded by the circle x2 + y2 = 4 and the lines x = 0
and x = 2
Q33. Show that the relation R in the set A = {1, 2, 3, 4, 5} given by R = {(a, b) : |a – b| is even}, is an
equivalence relation. Show that all the elements of {1, 3, 5} are related to each other and all the elements
of {2, 4} are related to each other. But no element of {1, 3, 5} is related to any element of {2, 4}.

OR

Check whether the relation R defined in the set {1, 2, 3, 4, 5, 6} as R = {(a, b) : b = a + 1} is reflexive,
symmetric or transitive.

Q34.

OR

Find the vector and cartesian equations of the line passing through the point (1, 2, –4) and
𝑥−8 𝑦+19 𝑧−10 𝑥−15 𝑦−29 𝑧−5
perpendicular to the two lines 3 = −16 = 7 and 3 = 8 = −5

Q35.
SECTION E
(This section comprises of 3 case-study/passage-based questions of 4 marks each with two sub-
parts. First two case study questions have three sub-parts (i), (ii), (iii) of marks 1, 1, 2 respectively.
The third case study question has two sub-parts of 2 marks each.)

Q36. Case-Study 1: Read the following passage and answer the questions given below.

A potter made a mud vessel, where the shape of the pot is based on f(x) = |x-3| + |x-2|, where f(x) represents
height of the pot

i. When x>4 what will be the height in terms of x ?


a. x-2 b. x-3 c. 2x-5 d. 5-2x

ii. What is dy/dx at x=3 ?


a. 2 b. -2 c. 1 d. Function is not differentiable
iii. If the potter is trying to make a pot using the function f(x)= [x] will he get a pot or not ?
a. Yes, because it is a continuous function
b. Yes, because it is not a continuous function
c. No, because it is a continuous function
d. No, because it is not a continuous function

Q37. Case-Study 2: Read the following passage and answer the questions given below.

A tank with rectangular base of length x metre, breath y metre and rectangular side, open at the
top is to be constructed so that the depth is 1 m and volume is 9𝑚3.If building of tank is Rs 70 per
square metre for the base and Rs 45 per square metre for the sides?

(i) What is the cost of the base?


(a) 9xy (b) 70xy (c) xy (d) 50xy

ii) What is the cost of making all the sides?


(a) 90(x+y) (b) 90xy (c) 9(x+y) (d) 40(x +y)
(iii) If ‘C’ be the total cost of the tank , then
for what value of x, C is minimum?
(a) 2 (b) 1 (c) 3 (d)5
Q38. Case-Study 3: Read the following passage and answer the questions given below.

There are two antiaircraft guns, named as A and B. The probabilities that the shell fired from
them hits an airplane are 0.4 and 0.2 respectively. Both of them fired one shell at an airplane at the
same time.
(i) What is the probability that the shell fired from exactly one of them hit the plane?
(ii) If it is known that the shell fired from exactly one of them hit the plane, then what is the
probability that it was fired from A?
Sample Question Paper
Class XII
Session 2023-24
Mathematics

Time Allowed: 3 Hours Maximum Marks: 80

General Instructions :

1. This Question paper contains - five sections A, B, C, D and E. Each


section is compulsory. However, there are internal choices in some
questions.
2. Section A has 18 MCQ’s and 02 Assertion-Reason based questions of 1 mark
each.
3. Section B has 5 Very Short Answer (VSA)-type questions of 2 marks each.
4. Section C has 6 Short Answer (SA)-type questions of 3 marks each.
5. Section D has 4 Long Answer (LA)-type questions of 5 marks each.
6. Section E has 3 source based/case based/passage based/integrated
units ofassessment (4 marks each) with sub parts.

SECTION A
(Multiple Choice Questions)
Each question carries 1 mark
Q1. If A is matrix of order m × n and B is a matrix such that AB′ and B'A are both
defined , then order of matrix B is

(a) m × m (b) n × n (c) n × m (d) m × n

Q2. If A is any square matrix of order 3 × 3 such that |𝐴| = 3 , Then the value of

|𝑎𝑑𝑗𝐴| is

1
(a) 3 (b) (c) 9 (d) 27
3

Q 3. If |𝑎 ⃗ | = 2 and 𝑎
⃗ | = 10 , |𝑏 ⃗ = 12 , then the value of
⃗ .𝑏 |𝑎 ⃗ | is
⃗ × 𝑏

(a) 5 (b) 10 (c) 14 (d) 16

4 −𝑥 2
Q4. The function f(x) = 4𝑥 −𝑥 3 is
(a) Discontinuous at only one point
(b) Discontinuous at exactly two points
(c) Discontinuous at exactly three points
(d) None of these
Q5. ∫ tan−1 √𝑥 𝑑𝑥 is equal to

(a) ( x + 1 ) tan−1 √𝑥 −√𝑥 + C


(b) xtan−1 √𝑥 −√𝑥 + C
(c) √𝑥 − xtan−1 √𝑥 + C
(d) √𝑥 − ( x + 1 ) tan−1 √𝑥 + C
𝑑𝑦
Q6. The integrating factor of differential equation cos 𝑥 + y sin x = 1 is
𝑑𝑥
(a) cos 𝑥 (b) tan x (c) sec x (d) sin x

Q 7. The point which does not lie in the half plane 2 x + 3 y – 12 ≤ 0 is

(a) (1,2) (b) (2,1) (c) (2,3) (d) ( − 3 , 2)

Q8. If 𝜃 is the angle between any two vectors 𝑎 ⃗ , then |𝑎


⃗ and b ⃗ | = |𝑎
⃗ .𝑏 ⃗ |
⃗ × 𝑏
when 𝜃 is equal to
𝜋 𝜋
(a) 0 (b) (c) (d) 𝜋
4 2

𝜋
4 𝑑𝑥
Q9. ∫ is equal to
𝜋 1+cos 2𝑥

4

(a) 1 (b) 2 (c) 3 (d) 4

Q10. Let A be a non - singular matrix of order 3 × 3 , Then |𝑎𝑑𝑗𝐴| is equal to

(a) |𝐴| (b) |𝐴|2 (c) |𝐴|3 (d) 3|𝐴|

Q11. The corner points of the feasible region determined by the system of linear constraints
are (0,0), (0,40),(20,40),(60,20),(60,0). The objective function is 𝑍 = 4𝑥 + 3𝑦.
Compare the quantity in Column A and Column B
Column A Column B
Maximum of Z 325

(a) The quantity in Column A is greater.


(b) The quantity in Column B is greater.
(c) The two quantities are equal
(d) The relationship cannot be determined on the basis of the information supplied.

Q12. If A is a square matrix such that |𝐴| = 5 , Then the value of |𝐴 𝐴𝑇 | is

(a) 25 (b) 26 (c) 27 (d) 28


Q13. If A is a 3 × 3 matrix, |𝐴| ≠ 0 and |3𝐴| = k|𝐴| , then the value of k is

(a) 25 (b) 26 (c) 27 (d) 28


1
Q14. If P(A) = 2 , P(B) = 0 , then 𝑃(𝐴|𝐵) is

1
(a) 0 (b) (c) not defined (d) 1
2

Q15. The general solution of 𝑒 𝑥 𝑐𝑜𝑠 𝑦 𝑑𝑥 − 𝑒 𝑥 𝑠𝑖𝑛 𝑦 𝑑𝑦 = 0 is

(a) 𝑒 𝑥 𝑐𝑜𝑠 𝑦 = 𝑘 (b) 𝑒 𝑥 𝑠𝑖𝑛 𝑦 = 𝑘


(c ) 𝑒 𝑥 = 𝑘 𝑐𝑜𝑠 𝑦 (d) 𝑒 𝑥 = 𝑘 𝑠𝑖𝑛 𝑦

𝑑2 𝑦
Q16. If 𝑥 = 𝑡 2 and 𝑦 = 𝑡 3 then
𝑑𝑥 2

3 3 3 3
(a) (b) (c ) (d) 4
2 4𝑡 2𝑡

Q17. The magnitude of projection (2 𝑖̂ − 𝐽̂ + 𝑘̂) on ( 𝑖̂ − 2𝑗̂ +2 𝑘̂) is

(a) 1 unit (b) 2 units (c ) 3 units (d) 4 units

Q18. If a line makes angles 𝛼 , 𝛽, 𝛾 with the positive direction of co – ordinate axes,
then the value of 𝑠𝑖𝑛2 𝛼 + 𝑠𝑖𝑛2 𝛽 + 𝑠𝑖𝑛2 𝛾 is.

(a) 0 (b) 1 (c) 2 (d) 3

ASSERTION-REASON BASED QUESTIONS


In the following questions, a statement of assertion (A) is followed by a
statement of Reason (R). Choose the correct answer out of the following choices.

(a) Both A and R are true and R is the correct explanation of A.


(b) Both A and R are true but R is not the correct explanation of A.
(c) A is true but R is false.
(d) A is false but R is true.

Q19. Assertion (A): Range of 𝑐𝑜𝑡 −1 𝑥 is (0,π)

Reason (R): Domain of 𝑡𝑎𝑛 −1 𝑥 is R.

Q20. Assertion (A): The acute angle between the line 𝑟 = 𝑖̂ + 𝑗̂ + 2𝑘^ + 𝜆(𝑖̂ − 𝑗̂) and the
X – axis is 𝜋/4
Reason (R): The acute angle 𝜃 between the lines
𝑟= 𝑥1 𝑖̂ +𝑦1 𝑗̂ + 𝑧1 𝑘^ + 𝜆(𝑎1 𝑖̂ +𝑏1 𝑗̂+ 𝑐1 𝑘̂ ) and

𝑟= 𝑥2 𝑖̂ +𝑦2 𝑗̂ + 𝑧2 𝑘^ + 𝜇(𝑎2 𝑖̂ +𝑏2 𝑗̂+ 𝑐2 𝑘̂ ) is given by


SECTION B
This section comprises of very short answer type-questions (VSA) of 2 marks each

Q21. Write the principal value of tan−1 (√3) + cot −1 (−√3).


OR
If A = {1,2,3}, B = {4,5,6,7} and f = {(1,4), (2,5),(3,6)} is a function from A to
B . State whether f is one – one or not.
Q22. The radius of a cylinder is increasing at the rate of 5 cm/min so that its volume is
constant. Find the rate of decreasing of its height when its radius is 5 cm and
height is 3 cm.
Q23. Find a vector in the direction of 𝑎 ⃗ = 𝑖̂ − 2𝑗̂ that has magnitude 7 units.
OR
0 0
If a line makes angles 90 , 60 and 𝜃 with x, y and z axis respectively , where 𝜃
is acute, then find 𝜃.
Q24. Write the derivative of sin x with respect to cos x.
Q25. Write a unit vector in the direction of vector ⃗⃗⃗⃗⃗⃗
𝑃𝑄 where 𝑃 ⃗ and 𝑄⃗ are the points
(1,3,0) and (4,5,6) respectively.
SECTION C
(This section comprises of short answer type questions (SA) of 3 marks each)

Q26. Find : ∫ sin −1 (2𝑥) 𝑑𝑥 .

Q27. A and B throw a pair of dice alternately . A wins the game if he gets a total of 9 and
B wins if he gets a total of 7 . If A starts the game, find the probability of
winning the game by B.
OR
A problem in Mathematics is given to 4 students A, B, C , D . Their chances of

1 1 1 2
solving the problem are , , and respectively. What is the probability
3 4 5 3
that (i) the problem will be solved ? (ii) at most one of them solve the problem ?
𝜋
Q28. Evaluate : ∫04 𝑙𝑜𝑔[1 + 𝑡𝑎𝑛𝑥]𝑑𝑥

OR
3 2
Evaluate : ∫1 |𝑥 − 2𝑥| 𝑑𝑥

𝑑𝑦 𝑦
Q29. Solve the differential equation : 𝑥 𝑑𝑥
= y − 𝑥 𝑡𝑎𝑛 (𝑥 )
OR
𝑑𝑦
Solve the differential equation : (𝑥 − 𝑦) = x + 2y
𝑑𝑥
Q30. Solve the following Linear Programming Problem graphically :

Minimize : z = 6 x + 3 y subject to 4𝑥 + 𝑦 ≥ 80,


x + 5 y ≥ 115,
3 x + 2 y ≤ 150,
x ≥ 0, 𝑦 ≥ 0
𝑥
Q31. Find : ∫ (𝑥 2 +1)(𝑥−1) dx

SECTION D
(This section comprises of long answer – type questions (LA) of 5 marks each)

Q32. Find the area of the region {(𝑥, 𝑦): y2 ≤ 4𝑥, 4 𝑥 2 + 4 𝑦 2 ≤ 9} using method of integration.

Q33. Let A = { x ∈ Z : 0 ≤ x ≤ 12} . Show that R = {(a, b) : a, b ∈ A , |𝑎 – 𝑏| is divisible by 4 }

Is an equivalence relation . Find the set of all elements related to 1 . Also write the equivalence

class [2].

OR
Show that the relation S in the set R of real numbers defined as

S = { ( a, b ) : a , b ∈ R and a ≤ 𝑏3 } is neither reflexive nor symmetric nor transitive .

Q34. Find the shortest distance between the lines :

𝑟 = (t + 1) 𝑖̂ + (2 – t ) 𝑗̂ + (1 + t) 𝑘̂

𝑟 = (2s + 2) 𝑖̂ – (1 – s ) 𝑗̂ + (2 s – 1) 𝑘̂ .

OR

𝒙+𝟐 𝟐𝒚−𝟕 𝟓−𝒛


Find the direction cosines of the line = = . Also find the vector
𝟐 𝟔 𝟔
equation of the line through the point A (−1, 2, 3 ) and parallel to the given line .

1 1 1
Q35. If A =[1 0 2], find 𝐴–1. Use 𝐴–1 to solve the following system of equations
3 1 1

𝑥 + 𝑦 + 𝑧 = 6, 𝑥 + 2𝑧 = 7 , 3𝑥 + 𝑦 + 𝑧 = 12
SECTION E
(This section comprises of 3 case-study/passage-based questions of 4 marks each with
two sub-parts. First two case study questions have three sub-parts (i), (ii), (iii) of
marks 1, 1, 2 respectively. The third case study question has two sub-parts of 2 marks
each.)

Q36. Case-Study 1: Read the following passage and answer the questions given below.

Sonam wants to prepare a sweet box for Diwali at home. For making lower part of box, she
takes a square piece of card board of side 18cm.

(i) If x cm be the length of each side of the square cardboard which is to be cut off
from corner of the square piece of side 18 cm, then x must lie in which interval ?
(ii) Sonam is interested in maximizing the volume of the box. So, what should be the
side of the square to be cut off so that the volume of the box is maximum ?

𝑑𝑉
(iii) The values of x for which 𝑑𝑥 = 0, are

OR

What is the value of maximum volume ?

Q37. Case-Study 2: Read the following passage and answer the questions given below.
The Relation between the height of the plant (y in cm) with respect to exposure to sunlight is
1
governed by the following equation 𝑦 = 4𝑥 − 2 𝑥 2 where x is the number of days exposed to
sunlight.

(i) What is the number of days it will take for the plant to grow to the maximum height?
(ii) What is the maximum height of the plant?
(iii) What will be the height of the plant after 2 days?
OR
7
If the height of the plant is cm, then what is the number of days it has been exposed
2
to the sunlight ?

Q38. Case-Study 3: Read the following passage and answer the questions given below.

A coach is training 3 players. He observes that the player A can hit a target 4 times in 5 shots,
player B can hit 3 times in 4 shots and the player C can hit 2 times in 3 shots

(i) What is the probability that ‘none of them will hit the target’?
(ii) What is the probability that at least one of A, B or C will hit the target?
(iii) What is the probability that the target is hit ?
OR
What is the probability that the target is hit by exactly two persons ?

xxxxxxxxxxxxxxxxxxxxxxxxxxxxxxxxxxxxxxxxxxxxxxxxxxxxxxxxxxxxxxxxxxxxxxx
KENDRIYA VIDYALAYA SANGATHAN MUMBAI REGION
SAMPLE PAPER
SUBJECT: MATHEMATICS(041)
MAX. MARKS : 80 CLASS : XII TIME: 3 Hrs.
General Instructions :
1. This Question paper contains - five sections A, B, C, D and E. Each section is compulsory. However, there are
internal choices in some questions.
2. Section A has 18 MCQ’s and 02 Assertion-Reason based questions of 1 mark each.
3. Section B has 5 Very Short Answer (VSA)-type questions of 2 marks each.
4. Section C has 6 Short Answer (SA)-type questions of 3 marks each.
5. Section D has 4 Long Answer (LA)-type questions of 5 marks each.
6. Section E has 4 source based/case based/passage based/integrated units of assessment (4 marks each) with sub parts.
S.No SECTION - A Marks
Q1 √1 + cos 2𝑥 1
The simplest form of cot – 1 ( ) for 0 < x < /2
√1 − cos 2𝑥
𝜋 𝑥 𝜋 𝑥
a) x b) –x c) 2 d) 2 −
2 2
Q2 The value of k, for which the function f is continuos at x = 0, where 1
𝑠𝑖𝑛𝑥
− 𝑐𝑜𝑠𝑥, 𝑥≠0
𝑓(𝑥) = { 𝑥 at 𝑥 = 0
𝑘 𝑥=0
a) 0 b) 1 c) 2 d) 3
Q3 0, 𝑖 ≠𝑗 1
If A = [𝑎𝑖𝑗 ], where 𝑎𝑖𝑗 = { , then A3 is
−1 , 𝑖 = 𝑗
1 −1 −1 1
a) A b) I c) [ ] d) [ ]
−1 1 1 −1
Q4 1
The area of the region bounded by the curve 𝑦 = |𝑥|, 𝑦 = 0, 𝑥 = −4 𝑎𝑛𝑑 𝑥 = 2 is
____ sq. units (a) 6 (b) 8 (c) 10 (d) 12
Q5 The function 𝑓(𝑥 ) = log 𝑥, x > 0 is ________________ 1
a) always increasing on R b) always decreasing on R
c) Increasing on interval (0, ∞) d) Decreasing on interval (0, ∞)
Q6 For the following problem, the maximum value of Z _____________ 1
Maximize Z = 9x + 15y, subject to 3x + 5y ≤ 15, x + y ≤ 4, x ≥ 0, y ≥ 0
a) exist at only one point b) exist at two points
c) exist at infinite points d) does not exist.
Q7 𝑑𝑦
1
If 𝑦 = 5𝑥.sin 𝑥 , then 𝑑𝑥 =
a) 5𝑥.sin 𝑥 b) 5𝑥.sin 𝑥 log 5 c) 5𝑥.sin 𝑥 log 5 . cos 𝑥 d) 5𝑥.sin 𝑥 log 5 [𝑥 cos 𝑥 + sin 𝑥]

Q8 If A is a square matrix such that A2 = A, then ( I + A )3 – 7A is equal to 1


a) A b) Identity Matrix c) I + A d) I – A
Q9 𝑥 −2 −6 −2 1
If | |=| |, then the possible value of x is
2 2𝑥 − 2 2 −10
a) – 5 b) – 6 c) ±8 d) Both a and b
Q10 1
If A is a matrix of order 3x2 and B is matrix of order m x n such that matrix AB exist, then
a) m = 3 b) n = 3 c) m = 2 d) n = 2

Page 1 of 4
Q11 For what value of ‘p’ the vectors 𝑎⃗ = 2𝑖̂ − 3𝑗̂ + 4𝑘̂ and 𝑏⃗⃗ = 𝑝𝑖̂ + 6𝑗̂ − 8𝑘̂ are collinear. 1
(a) -4 (b)3 (c) -6 (d)9
Q12 𝑑 𝑑𝑦 2 1
The sum of order and degree of D.E. (( ) ) = 0
𝑑𝑥 𝑑𝑥
a) 4 b) 3 c) 2 d) 1
Q13 𝑥−1 𝑦−2 𝑧−3 𝑥−1 𝑦−1 𝑧−6 1
If the lines = = and = = are perpendicular, then the
−3 2𝑘 2 3𝑘 1 −5
value of k is
7 −7 10 −10
a) 10 b) c) d)
10 7 7
Q14 If a line makes angle /6 and /3 with y and z axes respectively, then its angle with 1
positive x – axis is
a) /6 b) /3 c) /2 d) /4
∧ ∧ ∧ ∧ ∧ ∧
Q15 1
If 𝑎⃗ = 𝑖 + 𝑗 + 6𝑘 and 𝑏⃗⃗ = 12𝑖 + 3𝑗 + 4𝑘, then the projection of 𝑎⃗ 𝑜𝑛 𝑏⃗⃗ is
(a) 3 (b)5 (c) 9 (d) 10
Q16 In the given probability distribution, the value of k is 1
X 0 1 2 3 4 5
P(X) k 2k 2k k 3k k

a) 1 b) 0.1 c) 0.2 d) 0
Q17 1
The feasible solution for a LPP is shown in given figure.
(4, 10)
Let Z=3x-4y be the objective function. Minimum of Z occurs
at (0, 8) (6, 8)
a) (0, 0) b) (5, 0)
(6, 5)
c) (0, 8) d) (4, 10)

Q 18 𝜋 4 + 5 sin 𝑥 1
The value of ∫−𝜋 log | | 𝑑𝑥 a) 1 b) 3 c) 2 d) 0
4 − 5 sin 𝑥

ASSERTION-REASON BASED QUESTIONS Question 19 and 20


In the following questions, a statement of assertion (A) is followed by a statement of
Reason (R). Choose the correct answer out of the following choices.
(a) Both A and R are true and R is the correct explanation of A.
(b) Both A and R are true but R is not the correct explanation of A.
(c) A is true but R is false.
(d) A is false but R is true.
Q 19 𝑜
𝑠𝑖𝑛15𝑜 1
Assertion (A) : If A = [𝑐𝑜𝑠15𝑜 ], then |𝐴| = 1
𝑠𝑖𝑛75 𝑐𝑜𝑠75𝑜
Reason (R) : A is an Identity matrix
12 4 3
Q 20 Assertion (A) : The direction cosines of the vector 𝑎̅ = 12𝑖̂ − 4𝑗̂ + 3𝑘̂ are 13 , − 13 , 1
13
Reason (R) : Vector 𝑎̅ is the unit vector.
Page 2 of 4
SECTION B
Q 21 A stone is dropped into a quiet lake. If the produced circular waves increase at a rate of 2
4 cm/s, then find the rate of increase in its area when the radius of circular wave is 7
cm?
Q 22 2
Find a unit vector perpendicular to both the vectors 𝑎̅ = 2𝑖̂ + 𝑗̂ − 𝑘̂ and 𝑏̅ = 𝑖̂ − 2𝑗̂ − 2𝑘̂.
Q 23 Find the value of |𝑥 ⃗⃗⃗| if for a unit vector 𝑎
⃗⃗⃗⃗, ( 𝑥 ⃗⃗⃗⃗) ∙ ( 𝑥
⃗⃗⃗ − 𝑎 ⃗⃗⃗⃗) = 35.
⃗⃗⃗ + 𝑎 2
OR
Let 𝑎⃗ and 𝑏⃗⃗ be two vectors such that |𝑎⃗| = 3 and |𝑏| ⃗⃗⃗⃗ = √2/3 and a  b is a unit vector.
Find the angle between the two vectors.
Q 24 𝑑𝑦 3𝑥 − 𝑥 3 2
Find , if y = tan−1 ( )
𝑑𝑥 1 − 3𝑥 2
Q 25 1 1 2
Find the value of tan−1 (−1) + cos −1 (2) + sin−1 (2)
OR
Show that 𝑓: 𝑅 → 𝑍, given by 𝑓(𝑥) = [𝑥] is onto but not one-one.
SECTION C
Q26 𝑑𝑦 𝑦 3
Solve the differential equation : 𝑥 − 𝑦 + 𝑥. 𝑐𝑜𝑠 2 ( ) = 0
𝑑𝑥 𝑥
OR
𝑑𝑦
Solve the differential equation: 𝑐𝑜𝑠 2 𝑥 + y = tan 𝑥.
𝑑𝑥
Q27 1 3
Evaluate :∫ 2
𝑑𝑥 .
√5 − 4𝑥 − 𝑥
Q28 Minimise Z = 5𝑥 + 10𝑦 3
Such that 𝑥 + 2𝑦 ≤ 120 𝑥 + 𝑦 ≥ 60 𝑥 – 2𝑦 ≥ 0 𝑥 ,𝑦 ≥ 0
Q29 𝜋/2 cos 𝑥 3
Evaluate ∫0 cos 𝑥 + sin 𝑥 𝑑𝑥
OR
5
Evaluate ∫−3|𝑥 − 2| 𝑑𝑥
Q30 A factory has two machines A and B. Past record shows that machine A produced 60% 3
of the items of output and machine B produced 40% of the items. Further, 2% of items
produced by machine A and 1% produced by machine B were defective. All items are
put into one stockpile and then one item is chosen at random from this and is found to
be defective. What is the probability that it was produced by machine B?
Q 31 2𝑥 + 3 3
Solve the integral ∫ 𝑑𝑥
(𝑥 + 1)(𝑥 + 2)
OR
−1
Solve the integral ∫ 𝑥. tan 𝑥 𝑑𝑥
SECTION D
Q 32 𝒙+𝟑 𝒚−𝟏 𝒛+𝟒 5
Find the foot of the perpendicular from the point (0, 2, 3) on the line = = . Also
𝟓 𝟐 𝟑
find the length of the perpendicular
OR
Find the shortest distance between the lines whose vector equations are
𝑟⃗ = (𝑖̂ + 2𝑗̂ + 3𝑘̂ ) + 𝜆(𝑖̂ − 3𝑗̂ − 2𝑘̂) and 𝑟⃗ = (4𝑖̂ + 5𝑗̂ + 6𝑘̂ ) + 𝜇 (2𝑖̂ + 3𝑗̂ + 𝑘̂)

Page 3 of 4
Q33 Solve the following equations using matrix method 5
𝑥 + 2𝑦 + 3𝑧 = 6, 2𝑥 + 4𝑦 + 5𝑧 = 11, 3𝑥 + 5𝑦 + 6𝑧 = 14
Q34 5
Using integrals, find the area of region bounded by the curve 𝑥 2 + 𝑦 2 = 2, the line y = x
and x – axis in the first quadrant.
Q35 Consider the set N x N .Let R be the relation in NXN which is defined by (a,b) R (c,d) if 5
and only if ad = bc. Prove that R is an equivalence relation.
OR
Let 𝑓: [0, ) ⟶ [15, ) be a function defined as 𝑓(𝑥 ) = 4𝑥 2 + 12𝑥 + 15. Show that f is
bijective.
SECTION E (Case Study)
Q36 A husband and a wife appeared for a job interview.
The possibility of Husband’s selection is 4/5, where
as the possibility of wife’s selection is 5/6. Based on
the given information answer the following questions

(i) What are the possibilities that both are selected? 1


(ii) What are the possibilities that none of them is selected? 1
(iii) What are the possibilities that only one of them is selected? 2
Q37
The profit of a company is given by the function
𝑓 (𝑥) = − 2𝑥 2 + 16𝑥 + 12,where x is the production (in crores).
Based on the information answer the following questions.

(i) What will be the production when the profit is maximum? 2


(ii) What will be the maximum profit? 1

Find the interval in which the profit is strictly increasing? 1


Q 38 In a class room activity, a teacher asked all the students to bring a wire of length 100
cm. The wire is then bend to form a rectangle. Let x and y are the length and breadth of
rectangle. Based on above information answer the following question.
(i) Express x in terms of y. 1
(ii) If A is the area of rectangle, expresss A in terms of x. 1

(iii) Find dimensions of rectangle, so that area A is maximum. 2

***********

Page 4 of 4
KENDRIYA VIDYALAYA SANGATHAN MUMBAI REGION
Sample Question Paper
Class XII
Session 2023-24
Mathematics (Code-041)
Time Allowed: 3 Hours Maximum Marks: 80

General Instructions :
1. This Question paper contains - five sections A, B, C, D and E. Each section is
compulsory. However, there are internal choices in some questions.
2. Section A has 18 MCQ’s and 02 Assertion-Reason based questions of 1 mark each.
3. Section B has 5 Very Short Answer (VSA)-type questions of 2 marks each.
4. Section C has 6 Short Answer (SA)-type questions of 3 marks each.
5. Section D has 4 Long Answer (LA)-type questions of 5 marks each.
6. Section E has 3 source based/case based/passage based/integrated units of
assessment(4 marks each) with sub parts.

SECTION A
Multiple Choice Questions of 1 mark each
1 0 a  3 1
 
If 2 0  1 is a skew-symmetric matrix, then the values of a and b,
b 1 0 

respectively, are
(a) -2, -3 (b) -2, 3 (c) 2, -3 (d) -3, 3
2 8 0 1
If A(𝑎𝑑𝑗𝐴)=[ ], then |𝑎𝑑𝑗𝐴| is:
0 8
(a) 8 (b) 64 (c) 512 (d) 16
3 2 −1 3 1
The matrix [ 𝜆 0 7] is not invertible for
−1 0 7
(a) λ = –1 (b) λ = 0 (c) λ = 1 (d) λ = R – {1}
4 𝑘𝑥 + 1, 𝑥 ≤ 5 1
If 𝑓 (𝑥 ) = { is continuous at x = 5, then value of k is
3𝑥 − 5, 𝑥 > 5
6 7 9 5
(a) (b) (c) 5 (d) 9
7 6

5 1
∫ 𝑒 𝑥 (𝑡𝑎𝑛𝑥 + log 𝑠𝑒𝑐𝑥)𝑑𝑥 =

(a) 𝑒 𝑥 𝑡𝑎𝑛𝑥 + C (b) 𝑒 𝑥 𝑙𝑜𝑔𝑠𝑒𝑐𝑥 + C


(c) 𝑒 𝑥 𝑠𝑒𝑐 𝑥 + C (d) none of these

1
6 𝑑2 𝑦 𝑑𝑦 1
The sum of the order and degree of the differential equation + 3 𝑑𝑥 = 𝑒 𝑥 is
𝑑𝑥 2

(a) 2 (b) 3 (c) 5 (d) 4


7 The feasible region for LPP is shown shaded in the figure. Let Z = 3 x – 4 y be 1
the objective function, then the maximum value of Z is:

(a) 12 (b) 8 (c) 0 (d) –18


8 𝐼𝑓 |𝑎⃗ + 𝑏̅| = |𝑎⃗ − 𝑏̅| , then the angle between 𝑎
⃗⃗⃗⃗ and 𝑏⃗⃗ is 1

(a) 90° (b)60° (c)110° (d)180°


9 𝑠𝑖𝑛 2 𝑥−𝑐𝑜𝑠 2𝑥 1
∫ 𝑠𝑖𝑛 2 𝑥𝑐𝑜𝑠 2𝑥
dx is equal to:

(a) tan 𝑥+cot 𝑥 +c (b) tan 𝑥+ cosec𝑥 +c


(c) -tan 𝑥+cot 𝑥 +c (d) tan 𝑥+ sec 𝑥 + c
10 If A is a square matrix such that A2 = A, then the simplified value of (I – A)3 + A 1
is equal to
(a) A (b) A2 (c) I (d) A3
11 Consider the following system of linear inequalities: 1
2x + y ≤ 10 , x + 3y ≤ 15, x, y ≥ 0
If the corner points of the feasible region are (0, 0), (5, 0), (3, 4) and (0, 5).
Let Z = p x +q y, where p, q > 0. Condition on p and q, so that the maximum
value of Z occurs at both (3, 4) and (0, 5) is:
(a) p = q (b) p =2 q (c) p = 3q (d) q = 3p
12 3x 7 8 7 1
If | |=| |, the possible value(s) of x are
−2 4 6 4
(a) 2 (b) 3 (c) 2, 3 (d) -2
13 If A is a square matrix of order 3 and |A| = 5, then |𝑎𝑑𝑗𝐴| = 1
1
(a) 5 (b) 25 (c) 125 (d) 5

14 Let A and B are two events such that P(A) = 0.6, P(B) = 0.2 and P(A/B) = 0.5, 1
then P(A'/B') =
1 3 3 6
(a) 10 (b) 10 (c) 8 (d) 7

2
15 𝑑𝑦 1
The integrating factor of the differential equation 𝑑𝑥 + 𝑦𝑡𝑎𝑛𝑥 − 𝑠𝑒𝑐𝑥 = 0
𝑡𝑎𝑛 2 𝑥
(a) tan 𝑥 (b) 𝑠𝑒𝑐 2 𝑥 (c) sec 𝑥 (d) 2

16 Derivative of 𝑒 𝑙𝑜𝑔𝑥 𝑖𝑠 1
(a) 𝑒 𝑙𝑜𝑔𝑥 (b) log 𝑥 (c) 0 (d) 1
17 A man is watching an aero plane which is at the coordinate point A (4, -1, 3), 1
assuming the man is at O (0, 0, 0). At the same time, he saw a bird at the
⃗⃗⃗⃗⃗⃗⃗ is
coordinate point B (2, 0, 4). The unit vector along 𝐴𝐵
2 1 1 −2 1 1
(a) 𝑖̂ + 𝑗̂ + 𝑘̂ (b) 𝑖̂ + 𝑗̂ + 𝑘̂
6 6 6 √6 6 6
−2 1 1 4 2 3
(c) 𝑖̂ + 𝑗̂ + 𝑘̂ (d) 𝑖̂ + 𝑗̂ + 𝑘̂
√6 √6 √6 √6 √6 √6

18 P is a point on the line joining the points (0,5, −2) and 𝐵(3, −1,2). If the x- 1
coordinate of P is 6, then its z-coordinate is
(a) 10 (b) 6 (c) -6 (d) -10
ASSERTION-REASON BASED QUESTIONS
In the following questions, a statement of assertion (A) is followed by a
statement of Reason (R). Choose the correct answer out of the following choices.
(a) Both A and R are true and R is the correct explanation of A.
(b) Both A and R are true but R is not the correct explanation of A.
(c) A is true but R is false.
(d) A is false but R is true.

19 2𝜋 1
Assertion (A): 𝑡𝑎𝑛 −1 (tan )
3
𝜋 𝜋 𝜋 𝜋
= 𝑡𝑎𝑛−1 (tan( 𝜋 − )) = 𝑡𝑎𝑛−1 (− tan ) = − 𝑡𝑎𝑛−1 (tan ) = −
3 3 3 3
2𝜋 2𝜋
Reason (R): 𝑡𝑎𝑛−1 (tan )=
3 3

20 Assertion (A): The triangle ABC, with vertices A(1,2,3), B(2,-1,4) and C(4,5,-1) 1
√274
has the area = square units.
2
1
Reason(R): The area of the triangle ABC is ⃗⃗⃗⃗⃗⃗ × 𝐴𝐶
|𝐴𝐵 ⃗⃗⃗⃗⃗⃗ |
2

SECTION B
Very Short Answer Type-Questions (VSA) of 2 marks each
21 Find the domain of the function defined by 𝑓(𝑥) = 𝑠𝑖𝑛−1 √𝑥 − 1 2
OR
cos 𝑥−sin 𝑥 𝜋 3𝜋
Write in the simplest form tan−1 cos 𝑥+sin 𝑥 , − 4 < 𝑥 < .
4

3
22 A man 1.6 m tall walks at the rate of 0.3 m/sec away from a street light that is 4 2
m above the ground. At what rate is the tip of his shadow moving? At what rate
is his shadow lengthening?
23 Using direction ratios show that the points A(2, 3, 4), B(-1, -2, 1) and C(5, 8, 7) 2
are collinear.
OR
If a⃗⃗ = 2î + 2ĵ + 3k̂ , ⃗⃗
b = −î + 2ĵ + k̂ and c⃗ = 3î + ĵ are such that a⃗⃗ + λb
⃗⃗ is

perpendicular to c⃗, then find the value of λ.


24 dy cos2 (a+y) 2
If cos y = x cos (a + y), with cos a ≠ ±1 prove that =
dx sin a

25 Show that the line through points (4, 7, 8) and(2, 3, 4) is parallel to the line 2
through the points (−1, −2, 1) and (1, 2, 5).
SECTION C
Short Answer Type Questions (SA) of 3 marks each
26 (x2 +1)(x2 +4) 3
Evaluate: ∫ (x2 +3)(x2 −5) dx

27 Three friends go for coffee. They decide who will pay the bill, by each tossing a 3
coin and then letting the “odd person” pay. There is no odd person if all three
tosses produce the same result. If there is no odd person in the first round, they
make a second round of tosses and they continue to do so until there is an odd
person. What is the probability that exactly three rounds of tosses are made?
OR
Find the mean number of defective items in a sample of two items drawn one-
by-one without replacement from an urn containing 6 items, which include 2
defective items. Assume that the items are identical in shape and size .
28 𝑥 2+1 3
𝐸𝑣𝑎𝑙𝑢𝑎𝑡𝑒: ∫ (𝑥−1)2 (𝑥+3)dx

OR
Evaluate ∫ 𝑥 sin−1 𝑥 𝑑𝑥
29 dy 3
Show that the differential equation (x − y) dx = x + 2y is homogeneous and

solve it.
OR
Find the general solution of the differential equation
dy
x dx + y − x + xy cotx = 0 (x ≠ 0)

30 Solve the linear programming problem graphically: 3


Maximize: Z = 8000 x + 12000 y
Subject to the constraints:
3x + 4y ≤ 60, x + 3y ≤ 30, x ≥ 0, y ≥ 0.

4
31 𝑥 3 + 𝑥 +1 3
𝐸𝑣𝑎𝑙𝑢𝑎𝑡𝑒: ∫ (𝑥−1)2
dx

SECTION D
Long Answer-Type Questions (LA) of 5 marks each

32 Make a rough sketch of the region {(𝑥, 𝑦): 0 ≤ 𝑦 ≤ 𝑥 2, 0 ≤ 𝑦 ≤ 𝑥, 0 ≤ 𝑥 ≤ 2} and 5


find the area of the region using integration.
33 Define the relation R in the set 𝑁 × 𝑁 as follows: 5
For (a, b), (c, d) ∈ 𝑁 × 𝑁, (a, b) R (c, d) iff ad = bc. Prove that R is an
equivalence relation in 𝑁 × 𝑁.
OR
Show that 𝑓: 𝑅 → 𝑅, given by 𝑓 (𝑥 ) = 1 + 𝑥 2 is neither one-one nor onto.
34 Find the shortest distance between two lines whose vector equations are 5
r⃗ = (1 − t)î + (t − 2)ĵ + (3 − 2t)k̂
r⃗ = (s + 1)î + (2s − 1)ĵ − (2s + 1)k̂
OR
Find the equation of the line passing through the points A(0, 6, -9) and
B(-3, -6, 3). If D is the foot of perpendicular drawn from a point C(7, 4, -1) on
the line AB, then find coordinates of the point D and the equation of line CD.
35 2 −3 5 5
If A= [3 2 −4] find A-1.. Hence,
1 1 −2
solve the system of equations:
2x - 3y + 5z = 11, 3x + 2y - 4z = – 5 and x + y – 2z = −3
SECTION E
This section comprises of 3 case-study/passage-based questions of First two case study
questions have three sub-parts (i), (ii), (iii) of marks 1, 1, 2 respectively. The third case
study question has two sub-parts of 2 marks each.

36 Case-Study 1: 4
Read the following passage and answer the questions given below.
Three students Rohan, David and Leena are finding the derivative of the function
𝑓 (𝑥 ) = (𝑥 2 − 5𝑥 + 8)(𝑥 3 + 7𝑥 + 9).
They followed three ways mentioned below:
(i) Rohan differentiated the given function by using product rule
(ii) David differentiated the given function by expanding the product to
obtain a single polynomial
(iii) Leena differentiated the given function by logarithmic differentiation.
Did they all give the same answer?

5
37 Case-Study 2: 4
Read the following passage and answer the questions given below.
A wire of length 28 m is to be cut into two pieces. One of the pieces is to be
made into a square and the other into a circle.

In order to find the lengths of the two pieces, so that the combined area be
minimum, two steps are followed:
(i) Find the area of the circle.
(ii) Find the area of the square.
(iii) What should be the lengths of the two pieces so that the combined
area of the circle and the square is minimum?
38 Case-Study 3: 4
Read the following passage and answer the questions given below.
Probabilities of solving a specific problem independently by Javed Ali and
1 1
Jashandeep Kaur are and respectively. If both try to solve the problem,
2 3

independently, then find the probability that


(i) The problem is solved
(ii) Exactly one of them solves the problem.

Prepared by
Ajaykumar Patole
PGT-MATHS
K.V.NO.1,Ahmednagar

6
KENDRIYA VIDYALAYA SANGATHAN
SAMPLE QUESTION PAPER 2023-2024
SUB. – MATHEMATICS (041)
CLASS – XII
TIME: 3 Hrs. MM – 80
General Instructions:
1. This Question paper contains - five sections A, B, C, D and E. Each
section is compulsory.
However, there are internal choices in some questions.
2. Section A has 18 MCQ’s and 02 Assertion-Reason based questions of
1 mark each.
3. Section B has 5 Very Short Answer (VSA)-type questions of 2 marks
each.
4. Section C has 6 Short Answer (SA)-type questions of 3 marks each.
5. Section D has 4 Long Answer (LA)-type questions of 5 marks each.
6. Section E has 3 source based/case based/passage based/integrated
units of assessment (4
marks each) with sub parts.
SECTION – A
Q. QUESTION MARK
N. S
1 If A = [ aij] is a symmetric matrix of order n, then 1
(a) aij = 1/aij for all i,j (b) aij ≠ 0 for all i,j
(c) aij = aji for all i,j (d) aij = 0 for all i, j

2 If A is a square matrix of order 3, | 𝐴 | = 4, then | 𝐴𝐴′ | = 1


(a) 9 (b) -9 (c) -12 (d) 16

3 The area of a parallelogram with vertices A, B, C and D is given by 1


1 1
(a) ⃗⃗⃗⃗⃗⃗ 𝑥 𝐶𝐷
| 𝐴𝐵 ⃗⃗⃗⃗⃗ | (b) ⃗⃗⃗⃗⃗⃗ 𝑥 𝐶𝐷
| 𝐴𝐵 ⃗⃗⃗⃗⃗ | (c) ⃗⃗⃗⃗⃗⃗ 𝑥 𝐵𝐶
| 𝐴𝐵 ⃗⃗⃗⃗⃗ | (d)
2 2

| ⃗⃗⃗⃗⃗⃗
𝐴𝐵 𝑥 ⃗⃗⃗⃗⃗
𝐵𝐶 |

4 The value of 'k' for which the function f(x)={


𝑠𝑖𝑛𝑠𝑖𝑛 𝑥
+𝑐𝑜𝑠 𝑐𝑜𝑠 𝑥 𝑘 is 1
𝑥

continuous at x = 0 is
(a) 1 (b) - 2 (c) -1 (d) 2

5 If f ' (x) = sec x, the f(x) is 1


(a) sec x tan x (b) sec x + tan x

(c) log (sec x + tan x) (d) log ( sec x -


tan x)
6 If m and n, respectively, are the order and the degree of the 1
𝑑 𝑑𝑦 4
differential equation [( ) ] = 0, then m - n =
𝑑𝑥 𝑑𝑥

(a) 1 (b) -1 (c) 3 (d) 5

7 The scalar projection of the vector 2 𝑖̂ + 3𝑗̂ - 5𝑘̂ on the vector 5 𝑖̂ + 1


5𝑗̂ + 5𝑘̂ is
2
(a) (b) 0 (c) 25 (d) None of these
5√3

8 The value of ∫0
1
𝑥 √1 − 𝑥 dx is 1

(a) -4/15 (b) 4/15 (c) -3/15 (d)


2/15

−𝑎
9 The value of ∫𝑎 𝑠𝑖𝑛3 𝑥 𝑑𝑥 is 1

(a) a (b) a/3 (c) 1 (d) 0

10 If A , B are non - singular square matrices of the same order, then 1


(𝐴−1 𝐵−1 ) −1 is
(a) BA (b) AB (c) 𝐴𝐵−1 (d) 𝐴−1 𝐵

11 If [2 4 5 1 ] = [2𝑥 4 5 𝑥 ] , then the possible value(s) of 'x' is/are 1


(a) 1 (b) -1 (c) 0 (d) a and b
both

12 If A is a square matrix of order 3 and |𝑎𝑑𝑗. 𝐴 | = 25, then | 𝐴 | = 1


(a) 5 (b) -5 (c) 0 (d) a and b both

13 Given two independent events A and B such that P(A) = 0.2, P (A U 1


B) = 0.6, then P(B) is :
(a) 0.5 (b) 0.6 (c) 0.7 (d) None of these

14 The point which does not lie in the half plane 2x + 3y -12 ≤ 0 is: 1
(a) (1, 2) (b) (2, 1) (c) (2, 3) (d) ( -3, 2)

15 The general solution of the differential equation


𝑑𝑦
= 𝑒 𝑥 + 1 is: 1
𝑑𝑥

(a) y = 𝑒 𝑥 + 𝑐 (b) y = x𝑒 𝑥 + c (c) y = x + 𝑒 𝑥 + 𝑐 (d)


None of these
16 If x be the angle between two vectors 𝑎 and 𝑏⃗ , then | 𝑎 . 𝑏⃗| =|𝑎 x 1

𝑏⃗| when x is equal to :


(a) 0 (b) 30 (c) 45 (d) 90

17 Direction ratios of line given by :


𝑥−1
=
2𝑦+1
=
1−𝑧
are: 1
3 10 −7

(a) < 3,10,-7> (b) < 3, 5, 7> (c) < 3, -5, -7>
(d) < -3, 20, -7>

18 If y = log(tan x), then dy/dx is: 1


(a) 1/ tan x (b) 𝑠𝑒𝑐 2 𝑥/𝑡𝑎𝑛 𝑡𝑎𝑛 𝑥 (c) sec x (d) sec x tan
x

ASSERTION-REASON BASED QUESTIONS


In the following questions, a statement of assertion (A) is
followed by a statement of Reason (R). Choose the correct
answer out of the following choices.
(a) Both A and R are true and R is the correct explanation of A.
(b) Both A and R are true but R is not the correct explanation of
A.
(c) A is true but R is false.
(d) A is false but R is true.

19 Assertion (A) : The Domain of the function 𝑠𝑒𝑐 −1 𝑥 is (-∞ , - 1/2] U [ 1


1/2, ∞) .
3𝜋
Reason (R) : 𝑠𝑒𝑐 −1 (−√2) = .
4

20 Assertion (A): The direction cosines of the vector 2 𝑖̂ + 3𝑗̂ - 5𝑘̂ are 1
2 3 5
< , ,− >.
√38 √38 √38

Reason (R) : If < a, b, c> are direction ratios then direction


𝑎 𝑏 𝑐
cosines are < 𝐷 , 𝐷 , 𝐷 >, where D = √𝑎2 + 𝑏2 + 𝑐 2 .

SECTION – B
21 Find the value of 𝑐𝑜𝑠 −1 13𝜋
(𝑐𝑜𝑠 𝑐𝑜𝑠 7 ) . 2

OR

Give an example of a relation, which is :


(i) Reflexive, symmetric but not transitive
(ii) Transitive but neither reflexive nor symmetric

22 Find the rate of change of the area of a circle with respect to its 2
radius 'r' when r = 6 cm.
23 For what value of 'a' the vectors: 2 𝑖̂ - 3𝑗̂ + 4𝑘̂ and a𝑖̂ + 6𝑗̂ - 8𝑘̂ are 2
collinear?
OR

Find the vector equation of the line which passes through the point
(3, 4, 5) and is parallel to the vector 2 𝑖̂ + 3𝑗̂ - 5𝑘̂ .
24 Find dy/dx if 2x + 3y = sin y. 2
25 Find 'k' when (2 𝑖̂ + 6𝑗̂ - 14𝑘̂) 𝑥 ( 𝑖̂ + 𝑘𝑗̂ + 7𝑘̂) = 0
⃗ 2
SECTION – C
26 Evaluate: ∫
2𝑥
𝑑𝑥 3
(𝑥 2+1)(𝑥 2+2)

27 3
x 2
 dy
1  2 xy 
2
x 1
Solve the differential equation , where
2
dx

x   ,  1  1,  .

28 3
3
Evaluate : ∫02 | 𝑥 𝑠𝑖𝑛 𝑠𝑖𝑛 𝜋𝑥| 𝑑𝑥

OR
2
Evaluate∫−1 |𝑥 3 − 𝑥 | 𝑑𝑥.

29 Show that (x - y)dy = (x + 2y) dx is a homogenous differential 3


equation. Also, find the general solution of the given differential
equation.
OR
Solve the following differential equation.
𝑑𝑦
√1 + 𝑥 2 + 𝑦 2 + 𝑥 2 𝑦 2 + 𝑥𝑦 𝑑𝑥 = 0.

30 Solve the following Linear Programming Problem graphically: 3


Maximize Z = 5x + 2y subject to the constraints: x - 2y ≤ 2, 3x
+ 2y ≤ 12, -3x + 2y ≤ 3, x ≥ 0, y ≥ 0.

31 Find : ∫
3𝑥+5
𝑑𝑥 3
𝑥 2+3𝑥−18

SECTION – D
32 Bag I contains 3 red and 4 black ball and Bag II contains 4 red and 5 5
black balls. One ball is transferred from Bag I to Bag II and then a
ball is drawn from Bag II. The ball so drawn is found to be red in
colour. Find the probability that the transferred ball is black .

33 Let f: W → W be defined by: f(n) = {𝑛 − 1, 𝑖𝑓 𝑛 𝑖𝑠 𝑜𝑑𝑑 𝑛 + 1, 𝑖𝑓 𝑛 𝑖𝑠 𝑒𝑣𝑒𝑛 . 5


Show that 'f' is one-one and onto .
OR

Let N denoted the set of natural numbers and R be the relation on N


x N defined by: (a, b) R (c, d) if ad (b + c) = bc (a + d). Show that
R is equivalence relation.
34 Find the shortest distance between the following lines: 5
𝑟 = 2 𝑖̂ + 6𝑗̂ − 14𝑘̂ + 𝑠 (2 𝑖̂ + 6𝑗̂ − 14𝑘̂) and
𝑟 = 2 𝑖̂ + 6𝑗̂ − 14𝑘̂ + 𝑡 (2 𝑖̂ + 6𝑗̂ − 14𝑘̂). Also find the equation of
line joining the points from which the distance is shortest.
OR

Find the coordinates of the foot of perpendicular and the length of


the perpendicular drawn from the point P (5, 4, 2) to the line 𝑟 =
−𝑖̂ +3𝑗̂ + 𝑘̂ + 𝜆 (2𝑖̂ + 3𝑗̂ − 𝑘̂ ). Also find the image of P in this line.

35 The cost of 4kg onion, 3 kg wheat and 2 kg rice is Rs 60. The cost of 5
2 kg onion, 4 kg wheat and 6 kg rice is Rs 90. The cost of 6 kg
onion, 2 kg wheat and 3 kg rice is Rs 70. Find the cost of each item
per kg by matrix method.
SECTION – E
36 A manufacture produces three stationery products Pencil, Eraser and
Sharpener which he sells in two markets. Annual sales are indicated
below.
1
1
2

If the unit Sale price of Pencil, Eraser and Sharpener are Rs.
2.50, Rs. 1.50 and Rs. 1.00 respectively, and unit cost of the
above three commodities are Rs. 2.00, Rs. 1.00 and Rs. 0.50
respectively, then,
Based on the above information answer the following:

(i). Find the total revenue of market A.


(ii). Find the total revenue of market B.
(iii). What is the Cost incurred in market A.
OR
Find the Gross profit in both markets
37 Read the following passage and answer the questions given below:

1
A tank with rectangular base and rectangular sides, open at the 1
2
top is to be constructed so that its depth is 2 m and volume is 8
m3.If the building of tank costs Rs 70 per sq. meter for the base
and Rs 45 per sq. meter for the sides.
(i) What is the length of the tank for which construction cost is
least?
(ii) What is the breadth of the tank for which construction cost is
least?
(iii) What is the least cost of construction?

38 Read the following passage and answer the questions given below:
You want to make two gardens in the shape of square and
circle in front of your house. If you purchase a wire of length
28m to fence these gardens and you have used x meters of
wire to fence circular garden.

2
(i) What is the Radius of the circular garden and side of
2
squared garden?

(ii) If you want to minimize the combined area of both gardens


without wasting the wire of length 34m. Then How much
length of the wire will be needed to fence the circular garden.
And how much length of the wire will be needed to fence the
squared garden?
SAMPLE QUESTION PAPER ANSWER KEY 2023-2024

1 C 1
2 D 1
3 D 1
4 D 1
5 C 1
6 A 1
7 B 1
8 B 1
9 D 1
10 A 1
11 A 1
12 D 1
13 A 1
14 C 1
15 C 1
16 C 1
17 B 1
18 B 1
19 D 1
20 A 1
21 π 1
cos −1 (cos (2π − )
7
π
−1
cos (cos ( ) 1/2
7
Π /7
OR 1/2
(i) any example
(i) any example 1
1
22 A = π r2 0.5
dA/ dr = 2πr 0.5
at r = 6 dA / dr = 12π 1
23 2 3 4
=− =
a 6 −8 1

a= -4 1
OR
r⃗ = 3 î + 4ĵ + 5k + s(2 î + 3ĵ - 5k̂.)
̂ 2
24 2x + 3y = sin y.
2 + 3 dy/dx = cos y dy/dx 1
dy/dx = 2/(cos y - 3) 1
25 2 î + 6ĵ - 14k̂) x ( î + kĵ + 7k̂) = 0
⃗⃗
i j k
|2 6 −14| = 0 ⃗⃗ 1
1 k 7
î 42 + 14k)̂
( ̂
− ĵ(14 + 14) + k(2k − 6) = 0
⃗⃗
0.5
K=3 0.5
26 Put x2 = t and 2x dx = dt 0.5
Using partial fraction 1
Correct answer 2
27 The given differential equation is
x 2
 dy
1  2 xy 
2
dx x 1 2
½
 dy 
2x
y
2
dx x  1 x 1
2 2
 2
  
…………………………………………………….(i)
This is a linear differential equation of the
dy ½
form  Py  Q , where
dx
2x 2
P and Q 
x 1
2
 x2 1
2
  1
2x
 x2 1dx
IF  e 
Pdx
e  e log(x 1)  x 2  1
2

Multiplying both side of (i) by integrating
factor= x2 - 1, we get
x 2
1  dy
dx
 2 xy 
2
x 1 2
1

Integrating both sides, we get


2
y(x2 – 1) =  dx + C
x 12
1
2 x 1
 y(x2 – 1) = log + C  y(x2 – 1) =
2 x 1
x 1
log +C
x 1
This is the required solution.

28
3
1
∫0 | x sin πx| dx + ∫12 | x sin πx| dx
3 1
1
∫0 x sin πx dx - ∫12 x sin πx dx 1
2/π + 1/π2 1
OR
2 0 1
∫−1|𝑥 3 − 𝑥 |𝑑𝑥 = ∫−1(𝑥 3 − 𝑥 )𝑑𝑥 - ∫0 (𝑥 3 − 𝑥 )𝑑𝑥 +
2
∫1 (𝑥 3 − 𝑥 )𝑑𝑥….(1.5 marks)
0 1
𝑥4 𝑥2 𝑥4 𝑥2 𝑥4
=[4 − ] -[ − ] +[ −
2 4
−1 2 04
2
𝑥2
] …………………(1.5 marks)
2 1
1 1 1 1
= [(0 − 0) − (4 − 2)] - [(4 − 2) − (0 − 0)]
1 1
+[(4 − 2) − (4 − 2)]
1 1 1 11
= 4+4+2+4 = ………………………..(1mark)
4

29 F(λx, λy) = λ0F(x, y) 1


Put y = vx and dy/dx = v + x dv/dx 1
Correct answer Or 1
Given differential equation is √1 + 𝑥 2 + 𝑦 2 + 𝑥 2 𝑦 2
𝑑𝑦
+xy𝑑𝑥 =0
√1 + 𝑥 2 √1 + 𝑦 2
𝑑𝑦
= −𝑥𝑦 𝑑𝑥 ……………….(1/2mark)

𝑦
𝑂𝑟, 𝑑𝑦 =
√1+𝑦 2
√1+𝑥 2
− 𝑑𝑥 ………………….(1/2 mark)
𝑥
𝑖𝑛𝑡𝑒𝑔𝑟𝑎𝑡𝑖𝑛𝑔 𝑏𝑜𝑡ℎ 𝑠𝑖𝑑𝑒𝑠
𝑦 √1 + 𝑥 2
∫ 𝑑𝑦 = ∫ − 𝑑𝑥
√1 + 𝑦 2 𝑥
On putting 1+𝑦 2 = t and 1+𝑥 2 =𝑢2
…………….(1/2 mark)

30 Correct graph 1.5


Correct answer 1.5
31 3x + 5
∫ 2 dx
x + 3x − 18
Put 3x + 5 = A d/dx(x 2 + 3x − 18) + B 0.5
Correct value of A and B 0.5
Correct answer 2
32 For correct assumption of events 1
For correct probability 1
For correct formula 1
For correct answer 2
33 For one-one 2.5
For onto 2.5
OR
For reflexive 2
For symmetric 1
For transitive 2
34 For correct formula 1
For correct calculation 1.5
For equation 2.5
OR
For foot of perpendicular 3
For length of perpendicular 1
For image of the point 1
35 For correct equation 1.5
For correct inverse 1.5
For correct answer 2
36 (i)Rs. 46000 1
(ii)Rs. 53000 1
(iii) Rs. 31000 OR Rs. 32000 2

37 (i) 2 1
(ii) 2 1
(iii) Rs 1000 2
38 (i) 2
(ii) 2
SAMPLE QUESTION PAPER
V S MESHRAM VSN NAGPUR
Class XII
Session 2023-24
Mathematics (Code-041)

Time Allowed: 3 Hours Maximum Marks: 80


General Instructions :
1. This Question paper contains - five sections A, B, C, D and E. Each section is
compulsory. However, there are internal choices in some questions.
2. Section A has 18 MCQ’s and 02 Assertion-Reason based questions of 1 mark each.
3. Section B has 5 Very Short Answer (VSA)-type questions of 2 marks each.
4. Section C has 6 Short Answer (SA)-type questions of 3 marks each.
5. Section D has 4 Long Answer (LA)-type questions of 5 marks each.
6. Section E has 3 source based/case based/passage based/integrated units of
assessment (4 marks each) with sub parts.

SECTION A

(Multiple Choice Questions)


Each question carries 1 mark

𝑘 8]
1. Find value of k If A=[ is singular matrix
4 2𝑘
a) 0
b) 4
c) 2
d) ±4

4 𝑥+3
2. Find the value of x, if [ ]. Is symmetric matrix
2𝑥 − 3 𝑥 + 1
a) 3
b) 4
c) 5
d) 2
3
3. If [ x 2][ ] =2 ,then x
4

a) 2
b)-2
c)1
d) 3

4. Find the values of k so that the function f is continuous at the indicated point.
𝑘𝑥 2 , 𝑥 ≤ 2
𝑓 (𝑥) = { at x=2
3 , 𝑥>2
a) K=4/3
b) K= ¾
c) K=-4/3
d) K=-1/4

5. Evaluate .ʃ xsinx dx
(a) cos x + C
(b) sinx +C
(c) tan x + C
(d) –cos x + C

6. Write the degree and order of the differential equation.


𝑑 𝑑𝑦 3
{( ) } =0
𝑑𝑥 𝑑𝑥

(a) order 2, degree 1


(b) order 3, degree 1
(c) order 1, degree 2
(d) order 2, degree 2

7. The solution set of the inequality 3x + 5y < 4 is.


(a) an open half-plane not containing the origin.
(b) an open half-plane containing the origin.
(c) the whole XY-plane not containing the line 3x + 5y = 4.
(d) a closed half plane containing the origin.
8. A vector in the direction of vector 5 𝑖⃗ − 𝑗⃗ + 2𝑘⃗⃗ that has magnitude 8 is.
8
a) (5 𝑖⃗ − 𝑗⃗ + 2𝑘⃗⃗)
√30
8
b) √30
2𝑗 + 2𝑘⃗⃗)
(5 𝑖⃗ − ⃗⃗⃗⃗
8
c) √30
( 𝑖⃗ − 𝑗⃗ + 2𝑘⃗⃗)
8
d) (5 𝑖⃗ − 𝑗⃗ + 3𝑘⃗⃗)
√30

√3 𝑑𝑥
9)Evaluate∫1 1+𝑥 2

a)𝜋/8

b) 𝜋 /12

c)2 𝜋 /3

d) 𝜋 /3

10)If A is invertible matrix of order 2 and |A|=4 find |A-1|


(a)4
(b)0
(c)1
(d) 1/4

11)Z = 8x + 10y, subject to 2x + y ≥ 1, 2x + 3y ≥ 15, y ≥ 2, x ≥ 0, y ≥ 0. The minimum value


of Z occurs at
(a) (4.5, 2)
(b) (1.5, 4)
(c) (0, 7)
(d) (7, 0).

12)The vertices (-2, 5), (2, -3) and (0, 1) are


(a) non collinear
(b) collinear
(c) circuler
(d) none of these

13The area of a triangle with vertices (2, -6), (5, 4) and (k,4) is 35 sq. units. The value of k
will be.
(a) 9
(b) 3
(c) 12
(d) 6
𝐸
14)Two events E and F are independent if P(E)=0.3 abd P(E∪ F) =0.5 then P( 𝐹 ) equals

(a) 1/4
(b) 3/8
(c) 2/7
(d) 1/2.

15)The general solution of the differential equation 𝑦𝑑𝑥 − 𝑥𝑑𝑦 = 0 𝑖𝑠.

a) 𝑥𝑦 = 𝐶
b) 𝑥 = 𝐶𝑦2
c) 𝑦 = 𝐶𝑥
d) 𝑦 = 𝐶𝑥2

𝑑𝑦
16) 𝑡𝑎𝑛−1 (−√3) + 𝑡𝑎𝑛−1 (1), 𝑡ℎ𝑒𝑛 𝑑𝑥 𝑖𝑠𝑒𝑞𝑢𝑎𝑙𝑡𝑜.

𝜋
a) 12
𝜋
b) 4

𝜋
c) 2

𝜋
d)- 12

17) The angle between vectors a and b with magnitude √3 and 4 and a.b =2√3 is
(a) 30°
(b) 45°
(c) 60°
(d) 90°
𝑥+1 𝑦−2 𝑧+3 𝑥 𝑦 𝑧
18)The lines 2
= −4
= 6
and 1
= 2 = 3 are
(a) parallel
(b) perpendiculer
(c) intersecting
(d) coincident
ASSERTION-REASON BASED QUESTIONS
In the following questions, a statement of assertion (A) is followed by a statement of
Reason (R). Choose the correct answer out of the following choices.

(a) Both A and R are true and R is the correct explanation of A.


(b) Both A and R are true but R is not the correct explanation of A.
(c) A is true but R is false.
(d) A is false but R is true.
Π
19)i) Assertion(A) : Principal value of 𝑠𝑖𝑛−1 √2
1
𝑖𝑠
4
−1 Π
Reason (R) : Principal value of 𝑐𝑜𝑡 −1 𝑖𝑠
√3 3

Π
i) Assertion(A) : Principal value of 𝑠𝑖𝑛−1 √2
1
𝑖𝑠
4
−1 Π
Reason (R) : Principal value of 𝑐𝑜𝑡 −1 𝑖𝑠 3
√3

1
20)Assertion(A) : Area of parallelogram with diagonals 𝑎⃗ 𝑎𝑛𝑑 𝑏 is .|axb|
2

𝑎 𝑎𝑛𝑑𝑏⃗⃗ represent the adjacent side of triangle can be obtained evaluating


Reason (R) : If ⃗⃗⃗⃗
|𝑎xb|

This section comprises of very short answer type-questions (VSA) of 2 marks each

SECTION-B
1
21)Find the value of 𝑡𝑎𝑛−1 (−1) + 𝑐𝑜𝑠 −1 (− √2) . OR

State the reason for the relation R in the set {1, 2, 3} given by R = {(1, 2), (2,1)} not to be
transitive.

The radius of the balloon increasing at the rate of 10cm/sec .At what rate surface area of
ballon will increase at radius 15 cm
22)If a⃗ = 4î – ĵ + k̂ and b⃗ = 2î – 2ĵ + k̂, then find a unit vector parallel to the
vector a⃗ +b⃗. OR

Find the angle between the vectors 5i+3j+4k and 6i-8j-k.

23) If x = a(𝜃 − 𝑠𝑖𝑛𝜃) and y =a(1+cos𝜃), then find dy/dx

24)Find the unit vector in the direction of the sum of the vectors 2î + 3ĵ – k̂ and 4î – 3ĵ + 2k̂.
SECTION C
(This section comprises of short answer type questions (SA) of 3 marks each)

sin(𝑥−𝑎)
25)Evaluate : ∫ sin(𝑥+𝑎) 𝑑𝑥

26)A speaks truth in 75% of the cases, while B in 90% of the cases. In what percent of cases
are they likely to contradict each other in stating the same fact? Do you think that statement of
B is true? .

OR A bag contains 3 red and 7 black balls. Two balls are selected at random one by one
without replacement. If the second selected ball happens to be red, what is the probability that
the first selected ball is also red?.

𝜋
π 𝑥𝑠𝑖𝑛 𝑥
27)Evaluate : ∫0 1+𝑐𝑜𝑠 2 𝑥
. OR ∫04 𝑙𝑜𝑔(1 + 𝑡𝑎𝑛𝑥)𝑑𝑥

28)Maximise : Z = x + 2y subject to the constraints


x + 2 y ≥ 100
2x – y ≤ 0
2x+ y ≤ 200
x, y ≥ 0
Solve the above LPP graphically..
29)Solve the differential equation
(1 + x)2dy + 2xydx = cot xdx . OR
Solve the differential equation (x2 – y2) dx + 2xydy = 0.

3𝑥+5
30)Evaluate : ∫ 𝑑𝑥.
(𝑥 2+3𝑥−18)
SECTION D
(This section comprises of long answer-type questions (LA) of 5 marks each)
30 Sketch graph Using integration, find the area of the region y=|x+3| and
1
evaluate∫−6 |𝑥 + 3| 𝑑𝑥

31 Show that the relation R in the set A ={ x∈ 𝑍: 0 ≤ 𝑥 ≤ 12}, 𝑔𝑖𝑣𝑒𝑛 𝑏𝑦

R ={(a,b):|a-b| is multiple of 4 is an equivalence relation

. OR

If R is a relation defined on the set of natural numbers N as follows:


R = {(x, y) : x ∈ N, y ∈ N and 2x + y = 24}, then find the domain and range of the relation R .
Also, find whether R is an equivalence relation or not. .

32)Find the shortest distance between the lines :


𝑥+1 𝑦+1 𝑧+1 𝑥−3 𝑦−5 𝑧−7
= = 𝑎𝑛𝑑 = =
7 −6 1 1 −2 1
OR
𝑥−1 𝑦+1 𝑧+10
Find the perpendicular distance of point (1, 0, 0) from the lines : 2
= −3
= 8

. Also, find the coordinates of foot of perpendicular and equation of perpendicular

1 3 4
33)If 𝐴 = [2 1 2] , find A-1. Use it to solve the system of equations
5 1 1
x +3y + 4z = 8, 2x +y +2z = 5, 5x + y +z = 7

SECTION E

(This section comprises of 3 case-study/passage-based questions of 4 marks eachwith two sub-parts.


First two case study questions have three sub-parts (i), (ii), (iii)of marks 1, 1, 2 respectively. The third
case study question has two sub-parts of 2marks each.)

34). Read the following and answer questions

A rectangular hall is to be developed for the meeting of farmers in an agricultural college to


aware techniques of cultivation if the perimeter is P and length breadths are x and y then answer
the following questions

i) The area of rectangular region A expressed as a function of x is


1 1 1
a) 2 (p+x2) b) 2 (px-2x2) c) 2 (px+2x2) d) (px-2x2)
ii) The maximum area of floor is area of page where cultivation can be done can be
done.
𝑃2 2𝑃 𝑃 𝑃2
a) b) c) d)
8 9 10 16
35). Case-Study 2: Read the following passage and answer the questions given below.

An organization conducted bike race with girls and boys with 250 participants finally three boys and
two girls were selected for race Let B = {b1,b2,b3} and G={g1 ,g2} where B represents boys and G
girls

1) Number of all possible relations from boys to girls


a) 26 b) 25 c) 0 d) 23

ii) number of injective relations from boys to girls

a) 26 b) 25 c) 0 d) 23

36). Case-Study 3:Read the following passage and answer the questions given below.

1 1
The probability of solving a problem by Ravi and Reshma is 2 and 3 respectively if both try
to sole the problem independently
i) find the probability that the problem is solved
ii) Exactly one of them will solve the problem

You might also like